Сохранен 892
https://2ch.hk/math/res/12775.html
24 декабря Архивач восстановлен после серьёзной аварии. К сожалению, значительная часть сохранённых изображений и видео была потеряна. Подробности случившегося. Мы призываем всех неравнодушных помочь нам с восстановлением утраченного контента!

МАТЕМАТИКА ДЛЯ НАЧИНАЮЩИХ, 27: гендерного равенства тред

 Аноним 08/03/17 Срд 18:17:51 #1 №12775 
kovalevskaia.jpg
Marie-Sophie-Germain1.jpg
6b30927fe0a372aa26ec721a29ea03f2.jpg
mamiejohnson.jpg
В этом треде мы изучаем математику, ну или начинаем это делать. Если ты школьник или студент, и тебя есть задача, то в здесь тебе помогут её решить или хотя бы скажут, в каком направлении двигаться для её решения.
Также приветствуется обсуждения самого процесса изучения и учебников/задачников.

Анально модерируемый альтернативный тред для начинающих: https://2ch.hk/math/res/9338.html
Обсуждение вузов и математического образования: https://2ch.hk/math/res/9453.html
Мемасики сюда постите: https://2ch.hk/math/res/7199.html

Список литературы от ОП-а, бывшего тут до меня. Был составлен на протяжении 13 тредов, к ознакомлению обязателен.
http://pastebin.com/4iMjfWAf

Если ты только зашел в тред и хочешь спросить, какую книжку прочитать, то ответ, скорее всего, будет в этих списках, анон.

Список от анона с dxdy. Довольно внушителен, тоже рекомендуется к прочтению. Является дополнением к списку старго ОП-а.
http://pastebin.com/YP1uaUyd
Goodbook.txt список книг с dxdy, рекомендованный тамошними обитателями.
http://pastebin.com/4FngRj6n
Литература - НМУ
https://docs.google.com/spreadsheets/d/1UWwIIAFwSwOQLK3m--LOaMOvHUivFDEz-JAnLa87i7Q/edit#gid=0
ОП-список 2. Составляйте список в реальном времени! Предлагайте в тред книги, критикуйте уже имеющиеся!
http://pastebin.com/szzZfkCM
Форчановский список, книги на английском.
http://4chan-science.wikia.com/wiki/Mathematics
Список с видеолекциями(в разработке):
http://pastebin.com/S3d7Jj6J
Качать книги тут:
http://libgen.io
А статьи тут(в разработке):
http://pastebin.com/3BfHPskz
Мемасы(в разработке):
http://pastebin.com/e38Yuj5V
Архивы тредов
http://pastebin.com/kiRZGVHW
http://arhivach.org/thread/233955/
Аноним 09/03/17 Чтв 15:32:19 #2 №12786 
И открывает новый тред вопрос про математику.

Где можно прочитать про достижения математики прошлого? И я имею ввиду не времена гаусса-ньютона а реальную древность. Например из древней греции/египта/индии/китая.
Аноним 09/03/17 Чтв 17:25:02 #3 №12789 
>>12786
Men of mathematics была такая книжка, но это научпоп про историю вообще, а тебе, видимо, надо искать пейперы, раз хочешь прям углубляться в это. Хотя вообще непонятно, во что там углубляться. Статьи на вики по теме точно недостаточно?
Аноним 09/03/17 Чтв 19:19:10 #4 №12791 
>>12789
Мне интересно как они пришли, например, к выводам о площади круга, тупо меряли ирл или какие то логические уловки уровня теории множеств использовали.
09/03/17 Чтв 19:48:27 #5 №12792 
>>12791
Приближали площадью вписанных и описанных многораников.
Аноним 09/03/17 Чтв 20:45:56 #6 №12793 
Привет, аноны. Примат, 2 курс в треде. За два года изнасилован Демидовичем и огромным количеством упражнений, которые делал по большей части по шаблону. У меня есть шанс стать человеком, или для примата всё в порядке?
Аноним 09/03/17 Чтв 20:51:48 #7 №12794 
>>12793
Норм.
Аноним 09/03/17 Чтв 21:00:27 #8 №12795 
Посоветуйте годные сервесы по изучению математики
Аноним 09/03/17 Чтв 21:07:54 #9 №12796 
>>12795
Сервис это что-то интерактивное, наверное?
Аноним 09/03/17 Чтв 21:29:13 #10 №12797 
>>12796
Ага
Аноним 09/03/17 Чтв 22:51:40 #11 №12805 
>>12797
Типа такого? http://mathlets.org/mathlets/
Только не слушай коэффициенты Фурье в наушниках там.
Аноним 09/03/17 Чтв 23:18:57 #12 №12806 
>>12805
Я послушал. Ничего страшного не произошло.
Аноним 09/03/17 Чтв 23:42:07 #13 №12809 
Открыл я листки эти Давидович мат анализ, там докажи, выведи. Как это делать вообще? Просто сижу и пялюсь в текст.
Аноним 10/03/17 Птн 00:24:04 #14 №12811 
>>12809
Ну например? Какое задание у тебя вызывает трудности?
Аноним 10/03/17 Птн 00:56:45 #15 №12812 
>>12811
Да ладно, это я запаниковал слишком рано, оказалось, что я просто школьную программу позабыл. Пойду наворачивать выше по списку. Хотя с доказательствами у меня всегда были проблемы, непонятно вообще с чего начинать, за что цепляться и как развивать мысль дальше. Сложно это все.
Аноним 10/03/17 Птн 01:16:30 #16 №12814 
>>12812
Доказательство - вещь творческая всё-таки. Никаких алгоритмов в ней нет, поэтому без опыта ты и должен чувствовать себя дезориентированным. Читай учебники, разбирай и прорабатывай доказательства, которые в них приводятся, когда почувствуешь почву под ногами - можешь пытаться читать только формулировки теорем, а доказательства к ним придумывать самостоятельно. Твои доказательства будут громоздкими и уродливыми, но это поначалу - элегантность и чувство прекрасного постепенно придут. Ну и делай упражнения, конечно. Просто бегать глазами по строчками и кивать головой - непродуктивный способ изучения математики.
Аноним 10/03/17 Птн 10:51:57 #17 №12816 
>>12786
история математики в трех томах, том первый. И у Ван-дер-Ваардена была книга по истории древней математики.
Аноним 10/03/17 Птн 10:54:37 #18 №12817 
>>12792
А как они находили площади этих многоугольников? Разбивали многоугольник на треугольники равнобедренные?
Аноним 10/03/17 Птн 12:16:51 #19 №12818 
Подкиньте еще сервисов по интерактивному обучению математике.
10/03/17 Птн 13:07:43 #20 №12819 
>>12817
Да, именно так.
sage[mailto:sage] Аноним 10/03/17 Птн 21:02:06 #21 №12824 
а чё тни вообще для математики успели сделать кроме как разве что замуж выйти за известных математиков, из-за чего в результате гипергамии иногда мы слышим и о существовании их жён
Аноним 10/03/17 Птн 21:52:15 #22 №12825 
192592159921.jpg
>>12824
Ну что же ты...
Аноним 10/03/17 Птн 22:12:52 #23 №12827 
>>12825
Нётер-Хуётер.
Аноним 10/03/17 Птн 23:19:22 #24 №12833 
>>12827
Ковалевская-хуевалевская
Аноним 11/03/17 Суб 00:04:39 #25 №12834 
Безымянный1.png
>>12775 (OP)
аноны, помогите мне пожалуйста. Мне нужно решить задачу по теоретической механике, нужно найти уравнение движения точки M через угол ф, который изменяется по правилу ф=3\pi*t. Для этого же нужно спроецировать на оси х и у точку M(?). То есть, фактически это задача школьной геометрии, но что-то я не могу её решить. Может подскажете, как эти проекции найти? Или хотя бы подскажите, какие свойства, теоремы школьной геометрии мне нужно вспомнить. Или может тут можно составить уравнение движения точки как-то по-другому?
Аноним 11/03/17 Суб 02:50:33 #26 №12837 
>>12775 (OP)
Недавно, кстати, поймал себя на мысли, что одних только польских математиков я знаю примерно столько же, сколько и женщин-ученых во всех областях
Аноним 11/03/17 Суб 12:54:40 #27 №12844 
Вот Фейнман благодаря синестезии возле формул видел цвета, что ему знатно помогало. Как такое развить?
Аноним 11/03/17 Суб 13:08:46 #28 №12845 
>>12844
Фейнман пиздел.
Аноним 11/03/17 Суб 13:26:42 #29 №12846 
>>12845
Почему ты так думаешь?
Аноним 11/03/17 Суб 14:38:53 #30 №12849 
если A ⊂ B, то A + B = B и AB = A.
Первое я еще понимаю, но нет ли у вас житейского примера почему работает второе?
Аноним 11/03/17 Суб 15:21:19 #31 №12852 
Есть книжки с подачей материала, рассчитаной на ПОНИМАНИЕ прежде всего? Возможно с углублением в историю, чтобы было понятно откуда вообще появлялись такие концепции и прочее.
Аноним 11/03/17 Суб 16:18:54 #32 №12853 
изображение.png
>>12849
А содержится в В, поэтому их пересекая получается А.
По другому - AB = {x | A э x и B э х}[множество таких икс, что икс принадлежит А и В(обоим множествам]. Т.к. каждый элемет А лежит и в В, то их пересечение есть А.
Аноним 11/03/17 Суб 17:57:36 #33 №12856 
Какая база нужна чтобы плотно подойти к лямбда-исчеслению и дискретке ?
мимо макак
Аноним 11/03/17 Суб 22:30:23 #34 №12862 
>>12856
Да никакой не надо, берешь сикп\конкрит матматикс\интро ту фп харриса и читаешь.
Аноним 11/03/17 Суб 23:34:43 #35 №12874 
>>12862
Спасибо.
Аноним 12/03/17 Вск 01:09:11 #36 №12876 
Вопрос ни разу не про математику, но все же.
Какие книжки по физике есть чтобы находились между школьным курсом и ланду-лившицем? А то школьные сильно простые, а ландау сразу начинает бросаться "ОЧЕВИДНО ЧТО КАК МОЖНО ЗАМЕТИТЬ ПОСЛЕ НЕБОЛЬШИХ ПРЕОБРАЗОВАНИЙ". Или скажите какой то другой курс физики по хардкору чтобы был, но понятный.
Аноним 12/03/17 Вск 01:31:05 #37 №12877 
Аноны, возможно немного не там задаю вопрос, но ладно. Что интересного можно почитать наподобие статьи про две культуры или предисловия к тривиуму Арнольда? Вроде эссе или статей, где математик (или какой естественнонаучник) рассуждает на тему науки, математики или образования.
Аноним 12/03/17 Вск 02:31:04 #38 №12878 
>>12876
Любой курс общей физики. Сивухин или Иродов, например. Ландавшиц - это всё-таки теорфиз уже.
Аноним 12/03/17 Вск 03:03:19 #39 №12879 
>>12834
ну помогите же пожалуйста
Аноним 12/03/17 Вск 03:16:32 #40 №12880 
>>12879
Словами опиши, что происходит.
Аноним 12/03/17 Вск 08:30:29 #41 №12882 
>>12877
Пол Локхард «Плач математика»
Манифест Дьедонне
Некоторые посты Дмитрия Павлова на тифаретнике
Гротендик - Урожаи и Посевы
Аноним 12/03/17 Вск 10:50:27 #42 №12885 
>>12878
А в чем разница между обычным и теоритической?
Ведь по идее теоритическая лучше для знаний чем общая.
Аноним 12/03/17 Вск 12:43:30 #43 №12888 
>>12882
Спасибо
Аноним 12/03/17 Вск 13:37:19 #44 №12890 
>>12775 (OP)
ЛОГИКА

Я в крайнем раздражении от доступных мне источников о логике предикатов.

Если конткретно: мне нужны примеры натуральной дедукции.
(Чтобы они были максимально точными и аккуратными.)

Что есть хорошего почитать по теме?

Просто сложилось впечатление, что доказывают как хотят, без всякого адекватного придерживания некоторой формальной системе. (На самом деле, просто пропускают очевидные для них вещи, которые мне не очевидны.)



Аноним 12/03/17 Вск 14:18:01 #45 №12891 
>>12852
Нашел что-то?
Аноним 12/03/17 Вск 14:29:09 #46 №12892 
>>12852
https://www.youtube.com/watch?v=4ZQWT2aQ73E
Аноним 12/03/17 Вск 15:07:44 #47 №12893 
Матанач, как называется взаимосвязь между аргументом функции и ее значением? Имеется в виду, что есть линейная функция, где у=ax, а есть степенная y=a^x, при этом вторая растет быстрее, чем первая.
Аноним 12/03/17 Вск 15:09:56 #48 №12894 
>>12893
Вот коэффициент скорости роста меня интересует
Аноним 12/03/17 Вск 15:18:37 #49 №12895 
s14.jpg
s15.jpg
>>12885
Слева - общая физика. Справа - теоретическая.
Аноним 12/03/17 Вск 15:21:27 #50 №12896 
>>12877
Арнольд Что такое метематика. И другие статьи Арнольда.
Аноним 12/03/17 Вск 15:28:42 #51 №12897 
>>12890
>Просто сложилось впечатление, что доказывают как хотят, без всякого адекватного придерживания некоторой формальной системе.
Так, в общем-то, и есть. Настоящая база математики куда глубже формализма. О формальных системах вспоминают, когда нужно разрешить какие-то разногласия, или привлечь к делу компьютер.
Аноним 12/03/17 Вск 15:35:22 #52 №12898 
Я только сегодня узнал, что узнать синус и косинус можно с помощью шаманства над координатной плоскостью и кругом над ней да в чем суть тригонометрии. Где же еще можно сделать такие чудные открытия, ибо сейчас я понимаю на уровне калькулятора большинство вещей?
Аноним 12/03/17 Вск 15:38:50 #53 №12899 
>>12898
Две минуты в гугле.
https://betterexplained.com/articles/intuitive-understanding-of-eulers-formula/
https://betterexplained.com/articles/intuitive-trigonometry/
Аноним 12/03/17 Вск 15:43:03 #54 №12900 
>>12899
А если не только тригонометрию, а вообще?
Аноним 12/03/17 Вск 15:46:50 #55 №12901 
>>12898
Для меня в своё время открытием был самый обычный курс матана в первом семестре. Когда всю школьную математику, которую я тогда воспринимал неким набором вычислительных рецептов, слабо связанных между собой, на моих глазах целиком построили буквально из грязи, из понятия натуральных чисел. Именно тогда я по-настоящему полюбил математику, ибо понял, что это такое на самом деле.
Аноним 12/03/17 Вск 16:29:54 #56 №12902 
https://www.youtube.com/watch?v=dPw0gI08hOk

Аноны, что думаете про этот курс лекций Вавилова? Хочу вспомнить, что забыл из линейной алгебре примат и расширить свои знания по алгебре в целом.
Аноним 12/03/17 Вск 16:30:27 #57 №12903 
>>12902
алгебры*
Аноним 12/03/17 Вск 17:24:13 #58 №12904 
Комплексная кошечка.png
>>12902
>>12903
Посмотрел в целом по курсу, складывается впечатление, что собственно линейной алгебры будет не так уж и много, в отличие от высшей алгебры теории групп, колец, категорий и прочих гомологий.
Из минусов курса - некоторые вещи даются на семинарах и благополучно оставлены за кадром, без книжки в руках эти лекции мало чего дадут, разве что немаленький математический кругозор, потому как Николай Александрович очень любит забегать вперёд и распалять в студентах и зрителях любопытство.

Вообще, лучше бы рассматривать этот курс целиком, вместе с топологией и матанализом а в дальнейшем и с более продвинутыми курсами, ведь эти дисциплины очень уж плотно перекликаются. Но для этого его сперва надо осилить целиком.
Аноним 12/03/17 Вск 17:52:32 #59 №12905 
>>12904
А какие книги можешь посоветовать? Можно и на английском. но я не математик, поэтому можно и какие-то попроще.
Аноним 12/03/17 Вск 18:47:34 #60 №12906 
>>12905
Если тебе интересна конкретно линейная алгебра - есть книга Кострикина и Манина "Линейная алгебра и геометрия". Как не математику она может тебе показаться сложноватой и бросающей тебя сразу с ровного места в воду, так что читать её лучше будет после трехкнижия "Введение в алгебру" того же Кострикина. Будет много повторов, которые напомнят тебе подзабытые понятия, и и немного нового.
На английском особых книг именно по линейной алгебре не подскажу, благо у нас хватает авторов, разбирающихся в тематике.
Аноним 12/03/17 Вск 19:11:48 #61 №12907 
>>12895
Хорошо, для каких целей тогда придуманы эти две физики?
Аноним 12/03/17 Вск 20:39:45 #62 №12908 
Capture.PNG
Аноны помогите понять. В решении вроде бы все логично до того, как в x подставляется число 2. Почему из этого дальше следует, что 2 это корень многочлена Q? Заранее ведь нигде не сказано про многочлен Q ничего.
Аноним 12/03/17 Вск 20:48:19 #63 №12909 
>>12908
P(2) = (2-1) Q(2)
P(2) = 0, потому что 2 - корень P.
Аноним 12/03/17 Вск 20:52:54 #64 №12910 
>>12909
Правильно ли я понял, что раз многочлен P(2) должен быть 0, то раз 2-1 дает 1, значит Q(2) должен быть нулем? Тогда почему именно от 2?
Аноним 12/03/17 Вск 20:54:02 #65 №12911 
>>12910
Я, наверное, слишком тупые вопросы задаю для этой доски, не бейте только сильно. Нужно как-то повторить школьный курс.
Аноним 12/03/17 Вск 21:01:26 #66 №12912 
>>12910
>>12911
Все я понял, лол. Какой же я тупой, раз при 1 оно уже известно обратилось в 0, остается только 2 подставить. А Q = x - 2, значит и при 2 обращается в 0.
Аноним 12/03/17 Вск 22:52:56 #67 №12913 
>>12907
Общая физика больше о том, чтобы что-то конкратное рассчитать, ответить на вопрос в духе "если мы сделаем вот так и измерим вот это, то что мы увидим в результате". Она оперирует наглядными и интуитивными понятиями, её рассуждения легко визуализировать, нарисовать картинку. Поэтому изучать физику начинают именно с неё - она более "плотская" и осязаемая, позволяет выработать какое-то интуитивное ощущение физической реальности, и в то же время не требует серьёзной математической подготовки и привычки к абстрактному мышлению. Кроме того, её изложение больше соответствует тому, как наука развивалась исторически. Она не стесняется апеллировать к эксперименту и каким-то феноменологическим частным законам, вроде закона Ома.

Теорфиз же от конкретики намеренно дистанцируется - уже уровне используемых понятий и единиц. Чтобы конвертировать его выкладки в конкретные предсказания в конкретном эксперименте, нужно немало попотеть. С точки зрения философии теоретической физики, не так уж важно знать, как далеко полетит снаряд, если выстрелить им из пушки. Гораздо важнее понимать, как природа устроена в целом, на структурном уровне. Наглядность и интуитивность здесь только мешают, и их вытесняют математические выкладки (хоть и не всегда совершенно строгие), а изложение подчинено структуре материала, логическим связям между его частями.

Попробую пояснить эту разницу на примере электродинамики и специальной теории относительности.

Суть кинематики СТО, с точки зрения общей физики: время не является абсолютным, при разгоне оно замедляется, длины сокращаются вдоль направления движения, одновременность относительна, передавать сигналы быстрее скорости света нельзя. Есть преобразования Лоренца, которые связывают разные системы отсчёта.
Суть кинематики СТО, с точки зрения теорфиза: время и пространство не существуют по отдельности, а являются разными проявлениями единого объекта - пространства-времени. В нём существует метрика специального вида, и группа преобразований, которая эту метрику сохраняет. Отсюда само собой вытекает требование ковариантности. Все уравнения должны иметь лоренц-ковариантный вид, и все физически реальные сущности должны описываться ковариантными математическими объектами.

Суть электродинамики, с точки зрения общей физики: Существуют электрические и магнитные поля, которые порождаются зарядами и токами, и могут взаимно порождать друг друга, за счёт чего передаются на далёкие расстояния в виде волн, и даже переносят энергию.
Суть электродинамики, с точки зрения теорфиза: Электрическая и магнитная составляющие электромагнитного поля и не могли бы существовать по отдельности, без взаимосвязи друг с другом, поскольку описываются одним и тем же неделимым ковариантным объектом - электромагнитным векторным потенциалом. Магнитное поле можно в первом приближении рассматривать как релятивистскую поправку к электростатике - именно поэтому оно порождается движущимися электрическими зарядами, а не каким-то особым магнитным зарядом.

Взгляд на динамику, с точки зрения общей физики: Существуют уравнения движения (второй закон Ньютона, к примеру, или уравнения Максвелла), которые определяют всю динамику. Свободные тела двигаются равномерно и прямолинейно, а любые отклонения от этого режима определяются действующими силами - силой тяготения, силой реакции опоры, силой трения, и другими - всё, что мы привыкли встречать в обычной жизни. Бывают особые комбинации переменных, которые сохраняются во время движения при определённых условиях - вроде энергии и импульса. Это упрощает расчёты и анализ, и именно поэтому их и ввели. Ещё сохраняется масса, заряды и некоторые другие вещи - так уж нам повезло.
Взгляд на динамику, с точки зрения теорфиза: Вся динамика закодирована в особой величине, называемой лагранжианом, и другой связанной с ним величине - действии. Их вид неслучаен и зачастую может быть выведен из геометрических принципов или каких-то ещё соображений. Ключевое значение имеют симметрии - именно наличием симметрий определяется существование сохраняющихся величин (теорема Нётер). Так, энергия двойственна ко времени и её сохранение связано с однородностью времени, и точно так же импульс двойственен к пространству и его сохранение связано с его однородностью. Таким образом, энергия и импульс гораздо более фундаментальные вещи, чем сила, и, в отличие от неё, сохраняют свой смысл даже в квантовом мире. Единственная сила (читай - взаимодействие), существующая в СТО - это взаимодействие вещества с электромагнитным полем. Единственная сила (читай - взаимодействие), существующая в СТО - это взаимодействие вещества с электромагнитным полем, и оно тоже закодировано в лагранжиане. Другие законы сохранения также имеют соответствующие им симметрии. Таким образом, изучение устройства группы симметрий может многое рассказать о динамике.
Аноним 12/03/17 Вск 22:57:02 #68 №12914 
>>12913
>Единственная сила (читай - взаимодействие), существующая в СТО - это взаимодействие вещества с электромагнитным полем. Единственная сила (читай - взаимодействие), существующая в СТО - это взаимодействие вещества с электромагнитным полем, и оно тоже закодировано в лагранжиане.
Зачем-то два раза скопипастил.
Аноним 12/03/17 Вск 23:19:18 #69 №12915 
Здравствуй, матемач. У меня вопрос по базовому терверу.

Есть классическая ошибка игрока, изучение которой гласит, что важно помнить о независимости испытаний от предыдущего исхода. Если пренебречь внешними факторами, монетке похер, сколько раз она там уже выпадала орлом или решкой, каждый раз она снова выпадает с вероятностью в 50%.

Но при этом есть и закон больших чисел. Который гласит, что при достаточно большой выборке, распределение будет приближаться к математически ожидаемому результату. Так вот, если монетка выпала 99 раз орлом, разве не логично предположить, что выборка достаточно большая, чтобы на сотом броске началось сдвижение в сторону ожидаемого 50/50, и вероятность выпадения решки была бы больше?
Аноним 13/03/17 Пнд 00:28:49 #70 №12917 
>>12915
Закон больших чисел работает не так. У тебя конкретный исход кодируется строкой из 100 битов, причём все эти строки равновероятны. Вероятность появления (для n = 6) 000000, 111111, 110100, 010101, 011110 - одна и та же, они все после шести бросков выпадут с равной вероятностью. А закон больших чисел - это просто выражение того факта, что строк с примерно равным числом нулей и единиц намного больше, чем тех, где их число сильно отличается. Иными словами, всё дело в том, что мы забываем про порядок выпадения орлов и решек и считаем только их количество. На вероятность выпадения орла или решки в конкретном эксперименте закон больших чисел никак не влияет.
Аноним 13/03/17 Пнд 00:41:36 #71 №12919 
>>12913
Значит общая физика это что то в духе "как это работает", а теоретическая "как работает общая физика"?
Другими словами, если обычному физику интересно как снять с коленвала в таких то условиях такой то крутящий момент, то физику-теоретику интересно почему крутящий момент вообще передается?

И наконец, даже если я начну учить ландау-лившица и над одной страницей пять дней сидеть до полного понимания сути написанного, это не сделает меня великим мастером снимания вращательного момента с коленвалов?
Опять же, другими словами, если я смогу в теоретическую физику я буду знать тайны устройства мироздания, но не смогу поменять розетку или выключатель в ванной?
Практический смысл я смогу из теоретической физики вынести? Или только если пойду работать в церн гонять протоны по кругу и доказывать гипотезу что под сабатон протон получает в 400 раз больше энергии?

Ты сам кто по образованию? А то так расписал со знанием. Или это паста?
Аноним 13/03/17 Пнд 00:43:41 #72 №12920 
Планиметрия изучается ведь по 9 класс? Хочу её повторить с самых низов, а у Понарина как-то все резко начинается. Есть ещё какие-нибудь книги?
Аноним 13/03/17 Пнд 00:44:08 #73 №12921 
>>12920
Начала эвклида.
Аноним 13/03/17 Пнд 00:50:35 #74 №12922 
>>12775 (OP)
Почему площадь круга пи радиус квадрат, а площадь кругового сектора угол в радианах на радиус?
Аноним 13/03/17 Пнд 01:06:38 #75 №12923 
Безымянный2.png
>>12880
вот картинка, и мне нужно задать уравнение движения точки М данного механизма. Для этого мне нужно спроецировать на оси Х и У точку М, и длины отрезков, которые должны зависеть от угла /phi. Но у меня не получается выразить эти проекции через /phi. Помогите мне пожалуйста, может я что-то делаю не так? Там угол /alpha равен 30 градусам, а отрезок AM=0.1
Аноним 13/03/17 Пнд 01:25:46 #76 №12924 
>>12919
>И наконец, даже если я начну учить ландау-лившица и над одной страницей пять дней сидеть до полного понимания сути написанного, это не сделает меня великим мастером снимания вращательного момента с коленвалов?
>Опять же, другими словами, если я смогу в теоретическую физику я буду знать тайны устройства мироздания, но не смогу поменять розетку или выключатель в ванной?
Ну, в общем случае так и выйдет, разве что ты какой-то там очень умный человек. Это как местная история с гомологами и ящиком. Вроде бы люди и математику знают, а такую простую прикладную задачу так долго решали. Хотя оптимизацию функции одной переменной преподают в любом односеместровом курсе высшей математики. А еще же и множители Лагранжа есть, например. Но они это не смогли увидеть.
Аноним 13/03/17 Пнд 01:46:29 #77 №12925 
>>12893
Порядки роста?
Аноним 13/03/17 Пнд 12:20:46 #78 №12932 
>>12922
Потому что это не так.
Аноним 13/03/17 Пнд 13:41:39 #79 №12933 
http://sunchildren.narod.ru/matem/horstmeier1.html

что скажите за эту методику?
Аноним 13/03/17 Пнд 13:56:01 #80 №12934 
>>12933
Ты думаешь если тебе обычная книжка не далась то дастся эта?
Разочарую, тебе и эта книжка будет не по зубам.
Аноним 13/03/17 Пнд 13:59:46 #81 №12935 
>>12934
А какую мне взять?
Аноним 13/03/17 Пнд 14:13:07 #82 №12936 
14844427453090.png
>>12933
>формы (двумерные и трехмерные)
Лол блядь. Смотрел по диагонали и слегка прихуел от такого, прежде чем понял, что имеется в виду. Видимо, у меня уже математика головного мозга.
Аноним 13/03/17 Пнд 14:41:24 #83 №12938 
>>12919
Примерно так, да. Хороший теоретик не обязан быть хорошим практиком, знание, как снимать вращательный момент с коленвала, не поможет тебе мастерски орудовать напильником, а если ты умеешь изготавливать великолепные шахматные фигуры из слоновой кости, это ещё не делает тебя великим шахматистом. Как и наоборот. Серебряной пули не существует, специализация возникла не просто так.
>Ты сам кто по образованию?
Я матфизик. Не тот, который краевые задачи решает, а в широком смысле.
Аноним 13/03/17 Пнд 14:45:27 #84 №12939 
>>12935
Если ты даун, то видосики на khan academy должны хорошо зайти, заодно английский выучишь
Аноним 13/03/17 Пнд 15:06:39 #85 №12940 
>>12936
Детки то, с синдромом Дауна, вовсю когомологии де Рама ебашут, а ты всё сидишь как сыч.
Аноним 13/03/17 Пнд 15:11:39 #86 №12941 
>>12938
И что делать, если я больше на практику нацелен и хочу уметь искать разрывы в проводке с помощью радио настроенного на короткие волны? Читать сивухина/иродова а потом что?
Или этих знаний достаточно чтобы на коленке из двух магнитов и проволоки скрутить коллекторный электромотор?
Аноним 13/03/17 Пнд 15:17:44 #87 №12942 
>>12941
Тебе тогда нужно что-нибудь техническое и инженерное. Тут я тебе ничего конкретного уже посоветовать не могу.
Аноним 13/03/17 Пнд 15:39:51 #88 №12943 
дифур: u(x, t)_x`` = u(x, t)_t``
при поиске решения частных дифур, как уравнение волны или heat equation, допускают, что
u(x, t) = X(x)T(t)
фу-я может быть представленна как произведение фу-й. метод разделения переменных.
это вывод из вида дифура? как? или угадали?
Аноним 13/03/17 Пнд 16:26:00 #89 №12945 
>>12943
Угадали. Это более-менее стандартная подстановка.
Аноним 13/03/17 Пнд 16:44:02 #90 №12946 
221
391
=
221 · 713
391 · 713

Всем привет! Откуда взялось число 713, никто не подскажет? А то я понять не могу.
Аноним 13/03/17 Пнд 16:57:48 #91 №12948 
>>12940
Ты их с аутистами перепутал.
Аноним 13/03/17 Пнд 16:58:46 #92 №12949 
>>12946
Откуда это?
Аноним 13/03/17 Пнд 17:11:29 #93 №12950 
>>12949
Действие с дробями
Аноним 13/03/17 Пнд 17:18:45 #94 №12951 
Дроби.png
>>12946
>>12950
Домножили и поделили? Ты же это имел ввиду?
Аноним 13/03/17 Пнд 17:22:06 #95 №12952 
>>12951
Блджад, я совсем запутался. Ладно, хуй с ними!
Аноним 13/03/17 Пнд 17:29:06 #96 №12953 
Есть что-то вроде математического букваря?
Аноним 13/03/17 Пнд 17:48:28 #97 №12958 
>>12953
Учебник арифметики
Аноним 13/03/17 Пнд 18:04:40 #98 №12959 
Мне говорили, что в продвинутых школах наряду с теорией множеств введут теорию категорий в первых классах. Можете пояснить и разъяснить?
13/03/17 Пнд 18:10:54 #99 №12960 
>>12959
Ты ебанулся? Кто тебе такое говорил?
Аноним 13/03/17 Пнд 20:20:25 #100 №12962 
>>12959
Я свою племяшку учу всяким прикольным морфизмам одновременно с таблицей умножения. В школу она пока ещё не ходит.
Аноним 13/03/17 Пнд 20:33:21 #101 №12963 
>>12923
Я не понимаю, что происходит в точке A. Зачем вообще дана длина АЕ?
Аноним 13/03/17 Пнд 21:48:58 #102 №12964 
>>12908
У меня чувство, что в одном из предыдущих тредов ты (а может и не ты) эту задачу постил.
Аноним 13/03/17 Пнд 22:11:50 #103 №12966 
>>12963
ну вроде бы в точке А скользящая заделка, хотя не знаю, а насчёт длины АЕ - наверное с помощью неё можно что-то выразить. Но к сожалению, у меня не выходит.
Аноним 13/03/17 Пнд 23:45:30 #104 №12969 
нецветные розы, нецветные розы...
Неравенство Аноним 13/03/17 Пнд 23:57:18 #105 №12970 
matan.png
Наверняка оно известное. Наверняка как-то называется. Подскажите.
Или, может его можно доказать самостоятельно?
(знак может быть в другую сторону, я запутался)
Аноним 14/03/17 Втр 00:57:31 #106 №12972 
Как в умф расписывается Uxx?
Аноним 14/03/17 Втр 02:30:45 #107 №12973 
CodeCogsEqn.gif
>>12972
Аноним 14/03/17 Втр 02:33:20 #108 №12974 
>>12970
Учитывая, что написанное имеет смысл только при 0 < t < 1, неравенство почти очевидно. Достаточно сравнить выражения в скобках.
Аноним 14/03/17 Втр 08:55:29 #109 №12976 
>>12974
Ой, и правда всё просто. Спасибо.
Аноним 14/03/17 Втр 10:46:32 #110 №12978 
>>12942
Ну хорошо.
Что тогда посоветуешь по общей физике, кроме сивухина и иродова?

И да, расскажи кулстори как докатился до жизни такой? Почему матфизиком стал, какими проблемами занимаешься, почему и т.д.
Аноним 14/03/17 Втр 14:25:15 #111 №12981 
image.jpg
Чтобы построить матрицу ссаного БО надо знать на каком множестве оно задано, да? Но Q это результат операции над двумя БО на двух множествах. На каком множестве задано Q, оправдывайтесь.
Аноним 14/03/17 Втр 16:02:47 #112 №12985 
125993408245319743397471151601288n.jpg
>>12981
Школьник, тут никто не будет разбираться в твоей глупой домашке, - тем более когда ты разговариваешь как унтерменш. Все вопросы задавай своей шараге.
Аноним 14/03/17 Втр 22:14:17 #113 №12989 
Ура! У меня получилось взять интеграл!
Аноним 14/03/17 Втр 23:16:25 #114 №12990 
Ананасы, возможно, не там задаю вопрос, расскажите, пожалуйста, как готовиться к экзаменам по математике, офк? В голове удаётся держать максимум половину семестра. На сессии наступает кромешный пиздец, когда между экзаменами 3-4 дня, а вопросов 100-120 матан, и почти во многих требуется доказательство а в билете минимум один вопрос с ним. Причём хорошо, если это какие-то простенькие выводы из свойств, дающихся по определению. Но иногда бывают "возьмём X следующего вида и через него хитровыебаным способом по лемме Y получим результат Z, из которого следует, что можно взять W и от противного прийти к требуемуму". При этом я примат. Люблю, конечно, задачки на доказательства порешать, которые преподы дают, но знать столько всего на экзамен это издевательство какое-то.
Аноним 15/03/17 Срд 01:13:10 #115 №12992 
>>12775 (OP)
Какая-то неведомая хуйня, вроде бы не произошло ничего страшного, но есть проблема, масштабы которой не пойму как оценить. Решать уравнения, неравенства и прочее могу, с параметрами, хуяметрами - не вопрос. Геометрию люблю всей душой, физику туда же. Так вот теперь к сути дела, я не могу никак научится воспринимать текстовые задачи и составлять исходя из условия точную модель ситуации, что в алгебре, что геометрии подобная лабудень. Самостоятельно не удается просто, я хуй знает, просто мозг отказывается воспринимать условия. Постоянно нуждаюсь в помощи.
Толи мозг кипит от этих всех формулировок дебильных или я тупой. С логикой какие-то проблемы что ль.
Пробовал решать олимпиадные задачи 7-8 класса - сосу хуй.

Было ли у кого подобное? Как лечили?
Аноним 15/03/17 Срд 02:00:57 #116 №12993 
>>12990
> и почти во многих требуется доказательство
Не только лишь во всех, надеюсь?

На мой взгляд, готовиться к конкретным вопросам, или тем более оценивать сложность экзамена по их количеству - верх уебанства. Я всегда просто читал конспект и повторял ключевые выкладки, чтобы быть уверенным, что воспроизведу их на экзамене. А список вопросов - это просто дополнительная опора для твоей памяти, когда ты их просматриваешь, поднимаешь материал в голове, и осознаёшь, что вот это ты помнишь, а вот это нужно ещё повторить. Не нужно пугаться их количества. Наоборот, чем они детализированнее, тем меньше шансов, что ты упустишь что-то важное. И да, тебе вовсе не обязательно воспроизводить лекции слово в слово. Если у какого-нибудь скалярного произведения выписано 20 свойств, от тебя никто не будет требовать их все. Если ты что-то упустишь, на это просто забьют хуй, или, если это было что-то реально важное, попросят написать прямо на месте. А если ты сдаёшь не лично лектору, то экзаменатор, скорее всего, вообще в душе не ебёт, что и как конкретно у вас было. За исключением старых пердунов, которые из 80 лет жизни 55 лет ходят принимать этот экзамен к этому лектору. Но это патологический случай. Вообще, чем моложе преподаватель, тем меньше он доёбывается до незначительной хуйни. Хотя если ты зафейлишь определение предела, то тебя это не спасёт.
Аноним 15/03/17 Срд 13:46:04 #117 №13014 
>>12993
>Не только лишь во всех, надеюсь?
Блджад, не проверил пост.

По матану у меня как раз "патологический случай", мда.
А есть смысл билеты писать по ходу семестра? И перечитывать их постоянно.
Аноним 15/03/17 Срд 18:14:51 #118 №13022 
>>13014
Есть.
Аноним 15/03/17 Срд 19:08:51 #119 №13025 
>>12990
Если ты понимаешь материал и можешь прочесть 3-4 раза за день до экзамена все конспекты, а после утром - заранее выписанные вопросы просмотреть за часик, то экзамен пролетает без проблем. Приходишь, пишешь, уходишь.
Ну и это, надеюсь, не надо объяснять, что подготовка к экзамену идёт с первого по последнее аудиторное занятие, а дальше - только повторение пройденного?
нечеткая логика и квантовые компьютеры Marcus 15/03/17 Срд 22:07:14 #120 №13029 
Сведущие, прошу ваших мнений, только начинаю учиться и сейчас выбрал для себя тему "Нечеткая логика", но боюсь что пока я начну въезжать в тему, появятся квантовые компьютеры и решат эту проблему. Как вы думаете, так ли это?
Аноним 15/03/17 Срд 22:53:20 #121 №13033 
>>12775 (OP)
помогите пожалуйста, знаю надо было учиться, ходить на пары, но долбаеб

Вычислить меру Лебега множества:
V = { (x; y) : 0<y<1/(1+x^2), x<0} на плоскости

нихуя не знаю, заебался перечитывать определение меры в википедии, в статьях из гугла какая-то непонятная херь

Как это сделать? интеграл лебега и мера лебега это одно и то же? где найти прочитать, как просто решить это дерьмо?
Аноним 15/03/17 Срд 23:10:52 #122 №13034 
>>13033
Высиляешь интеграл dxdy по заданному множеству.
Аноним 15/03/17 Срд 23:19:25 #123 №13035 
>>13034

типа просто двойной интеграл и все?
это и будем мера лебега?
Аноним 15/03/17 Срд 23:31:02 #124 №13036 
братушки, поясните пожалуйста про прямой и обратный пределы(про обратный поконкретнее), что они из себя представляют простыми словами, и если сможете то расскажите также про то как п-адические числа построить с помощью обратного предела. спасибо
Аноним 15/03/17 Срд 23:31:16 #125 №13037 
>>13034
пи пополам получил, а точно так, чот эт слишком просто, если я не накосячил. мне просто эту херню надо в курсач написать, серьезно просто обычный интеграл с первого курса?
Аноним 16/03/17 Чтв 00:05:35 #126 №13038 
>>13037
Что есть мера?
Аноним 16/03/17 Чтв 00:10:21 #127 №13039 
>>13038
величина, размер мерить, я хз. но просто она же называется Лебега. И я смог найти чтобы где-то говорилось что достаточно просто взять интеграл. я просто еще и к своему стыду понятия не имею что-такое интеграл, знаю только Ньютона-Лейбница
Аноним 16/03/17 Чтв 00:14:54 #128 №13040 
>>13039
Ну, голубчик, вы бы взяли томик Зорича по матану, и так сказать провели бы ликбез. А после Зорича можно уже вещи по-серьезнее взять.
Аноним 16/03/17 Чтв 00:16:01 #129 №13041 
>>13039
И да, не всегда можно взять интеграл, это зависит от множества, от того насколько оно замудренное.
Аноним 16/03/17 Чтв 00:36:37 #130 №13043 
>>13041
Спасибо, ты мне помог, возможно, когда я этот сдам курсач и еще 6 долгов меня даже не отчислят
Аноним 16/03/17 Чтв 00:45:50 #131 №13044 
Помогите плз решить:
Перейти к полярным координатам или к обобщенным полярным координатам и вычислить двойной интеграл ∫∫dxdy
за заданной областью D
D: x^2+y^2<=x, x^2+y^2<=y
Аноним 16/03/17 Чтв 00:46:29 #132 №13045 
>>13044
В чем проблема?
[mailto:[email protected]Аноним 16/03/17 Чтв 01:19:27 #133 №13046 
20170316000946.jpg
>>13045
Со всем)))Дохожу до этого момента, а с построением графика и дальнейшим решением проблема. Зато есть ответ(в ответнике): p/8-1/8
Аноним 16/03/17 Чтв 01:27:32 #134 №13047 
>>13046
https://www.wolframalpha.com/input/?i=polar+plot+sin(theta),+cos(theta)
Аноним 16/03/17 Чтв 01:29:14 #135 №13048 
>>13046
ну можно посмотреть на уравнения области, даже нарисовать их, и сделать нормальную замену.
Аноним 16/03/17 Чтв 01:44:26 #136 №13049 
IMG20170316004147HDR.jpg
Clip2net170316004021.png
Clip2net170316003953.png
>>13046
> p/8-1/8
Хм, что-то не то. Причем у вольфрам альфы тоже ответ вышел как у меня (я даже два интеграла отдельно взял для точности). Или я упоролся с областью интегрирования?
Аноним 16/03/17 Чтв 03:03:39 #137 №13051 
>>12978
Тебе и этого хватит. Тот же Сивухин охватывает вообще всё, что тебе в принципе может когда-нибудь понадобиться с практической точки зрения. Хотя это всё ещё не инженерный курс, со всеми вытекающими.
>Почему матфизиком стал
Сложно сказать. Поступил на физфак, поскольку всегда любил физику. А математику полюбил уже в универе. Я восхищался её стройностью и цельностью, тем, как на моих глазах выстраивают и связывают воедино всё, что я ещё со школы знал какими-то обрывками заклинаний и рецептов. Выбор кафедры матфизики был для меня естественным в такой ситуации.
>какими проблемами занимаешься, почему и т.д.
Мой научник занимается спектральной теорией дифференциальных операторов - в частности, спектром оператора Шрёдингера с периодическим потенциалом. Это имеет самое прямое отношение к ФТТ, зонной теории проводимости и всему такому - периодические потенциалы характерны как раз для кристаллических решёток, а спектр оператора Шредингера - это, по сути, множество допустимых энергий для пробного электрона. В частности, изучаются всякие пороговые эффекты, переходы из связанного состояния в свободное, гомогенизации (предельные спектры при стремлении периода решётки к нулю) с получением довольно хороших - квадратичных - оценок погрешностей, и так далее. Пока что я изучаю всю эту науку.

Хотя вообще-то есть тема, которая меня занимает с того самого момента, как я узнал, что такое квантмех и как он работает. Это динамические теории коллапса. Мне никогда не нравилось, что понятия наблюдения и наблюдателя вносятся искусственно, на уровне постулатов, что в квантмехе два фактически несовместимых друг с другом динамических принципа. Я ещё на третьем курсе, не читая никаких статей, самостоятельно пришёл к выводу, что линейное детерминированное уравнение Шрёдингера должно быть всего лишь приближением реального закона, который должен приводить в известных предельных случаях к унитарной эволюции или коллапсу. Я пытался придумать альтернативу, но ничего разумного у меня не получилось, и я со временем оставил попытки. И только сравнительно недавно я с удивлением обнаружил, что люди в мире вообще-то занимаются этой проблемой, и есть ощутимые результаты, хотя у них хватает принципиальных проблем. Вот хочу найти время и разобраться в этой теме как следует.
>>13029
Какую ещё проблему?

Квантовые компьютеры к нечёткой логике отношение не имеют никакого.
>>13036
У тебя есть некое семейство объектов, как-то связанных между собой стрелками. Объекты занумерованы элементами частично упорядоченного множества, а стрелки идут или от меньшего к большему (прямой предел), или наоборот (обратный). Если в твоём частично упорядоченном множестве есть максимальный элемент, то предел будет равен ему. Если нет, то он как бы добавит его искусственно. Т.е. в первом случае у нас пределом будет объект, в который есть стрелки из любого другого объекта, согласованные со всеми остальными. Во втором, соответственно, наоборот. Скажем, если мы возьмём цепочку расширяющихся множеств
A1 -> A2 -> A3 -> ...
то её прямым пределом будет их объединение.
И наоборот, обратным пределом сужающейся цепочки множеств
B1 <- B2 <- B3 <- ...
будет их пересечение.
Все стрелки - естественные вложения множеств, само собой.
>и если сможете то расскажите также про то как п-адические числа построить с помощью обратного предела
Ну зададимся простым числом p, и будем рассматривать кольца вида Z/pk. По смыслу элемент этого кольца - это младшие k разрядов некоторого целого числа в p-ичной системе счисления. Между ними существуют естественные морфизмы проекции из больших k в меньшие, которые работают как отрезание лишних разрядов. Таким образом, получаем диаграмму
Z/p <- Z/p2 <- Z/p3 <- ...
Здесь у нас в роли частично упорядоченного множества выступает N, в котором максимального элемента нет. Пределом будет "искусственный максимальный элемент", т.е. некое кольцо Zp, которое можно было бы поставить в конец диаграммы
Z/p <- Z/p2 <- Z/p3 <- ... <- Z/pk <- ... <- Zp
Это значит, что от элемента x ∈ Zp можно при каждом k в каком-то смысле "отрезать" левую часть, чтобы получить k последних разрядов. Нетрудно понять, что это числа, бесконечные влево, т.е. целые p-адические числа.
Аноним 16/03/17 Чтв 03:11:18 #138 №13052 
>>13051
Интересно. А можешь посоветовать каких-то книг по математике, которые тебе понравились? Буду благодарен.
Аноним 16/03/17 Чтв 06:32:51 #139 №13055 
>>12833
Однажды Ковалевская язвительно пошутила про скотов, с тех пор скоты не не любят Ковалевскую
Аноним 16/03/17 Чтв 09:07:31 #140 №13057 
>>13051
>понятия наблюдения и наблюдателя вносятся искусственно, на уровне постулатов, что в квантмехе два фактически несовместимых друг с другом динамических принципа.
Не только это.
Самое дубовое что сталкивает квантмех с теорией эйнштейна это квантовая телепортация. Эйнштейн лично запретил чему бы то нибыло передаваться со скоростью света, а квантмех запутанные частицы передает чуть ли не черту на рога.
Хотя с другой стороны, эйнштейн сказал что скорость света это максимум, но не уточнил какая именно скорость света. Можно конечно с помощью максвела её вывести, но все же.
Аноним 16/03/17 Чтв 09:10:17 #141 №13058 
>>13051
>>13057
Забыл добавить, ты же в каком то институте/нии работаешь, так!?

Можешь там поспрашивать про книжки для прикладного применения физики!?
Аноним 16/03/17 Чтв 10:52:48 #142 №13059 
>>13051
Я думал, раз "нечеткая логика" применяется в ситуациях когда ответ не очевиден, то наличие "квантового компьютера" даёт возможность "просчитать" вероятности для каждого из случаев. Буду благодарен если поправите.
Аноним 16/03/17 Чтв 10:56:22 #143 №13060 
>>13051
просто вектор развития, который я для себя определил, "нечеткая логика-->нейронные сети.
Аноним 16/03/17 Чтв 15:18:31 #144 №13064 
>>13051
>Мой научник занимается спектральной теорией дифференциальных операторов - в частности, спектром оператора Шрёдингера с периодическим потенциалом.
Как Сан Саныч поживает?
Аноним 16/03/17 Чтв 20:23:57 #145 №13068 
>>13052
Рудин меня в своё время впечатлил. Если ты ньюфаг, можешь его почитать. Ещё сходу могу вспомнить Неструева, и книжку Хелемского по функциональному анализу.
>>13059
Нечёткая логика применяется к нечётким высказываниям - "на улице холодно" или "2 - маленькое число". А если утверждение на самом деле либо истинно, либо ложно, но мы просто не знаем этого наверняка - это не нечёткая логика, это байесовские вероятности. Квантовая суперпозиция имеет одинаково мало общего и с тем, и с другим.
И да, если квантовый компьютер имеет вероятностный фактор в своей работе, это ещё не значит, что он хорошо вычисляет вероятности.
>>13060
Лучше многомерным анализом, матрицами и графами займись. Для нейросетей это всяко полезнее нечёткой логики будет.
>>13064
Мимо. Тот Сан Саныч, которого я знаю, чем-то совсем другим занимается.
Аноним 16/03/17 Чтв 22:15:00 #146 №13070 
Inteegral.png
Суп, матемач. Нид хелп. Как решать пикрел? Как я понял, линия пересечения поверхностей - это окружность радиуса -/2.
Записал параметрическое уравнение, проинтегрировал, но ответ не сходится. x=-/2cos(t), y=-/2sin(t), z=-/2?
Аноним 16/03/17 Чтв 22:22:21 #147 №13072 
>>13070
При подстановке твоей параметризации в уравнения они оба должны превратиться в тождества. Проверяй.
Аноним 16/03/17 Чтв 22:23:59 #148 №13073 
>>13072
Алсо, в задаче прямо-таки напрашивается формула Стокса.
Аноним 16/03/17 Чтв 22:32:15 #149 №13075 
>>13068
>«Рудин» — первый роман Ивана Тургенева, написанный в 1855 году.
Аноним 17/03/17 Птн 00:02:44 #150 №13079 
>>13075
Walter Rudin (May 2, 1921 – May 20, 2010)[2] was an Austrian-American mathematician and professor of Mathematics at the University of Wisconsin–Madison.[3]
In addition to his contributions to complex and harmonic analysis, Rudin was known for his mathematical analysis textbooks: Principles of Mathematical Analysis,[4] Real and Complex Analysis,[5] and Functional Analysis[6] (informally referred to by students as "Baby Rudin", "Papa Rudin", and "Grandpa Rudin", respectively).
Аноним 17/03/17 Птн 07:35:38 #151 №13083 
>>13079
> as "Baby Rudin", "Papa Rudin", and "Grandpa Rudin", re
А я думал это просто идиотский форчевский мемчик очередной.
Аноним 17/03/17 Птн 09:23:43 #152 №13084 
Посоны, как вы закончив школы/вузы продолжаете заниматься математикой? У многих после школы/вуза ненависть к этой науке появляется же. Только нормальные увлечения типо рисования/написания музыки и спасают. Вы или поехавшие тут, либо поехавшие, ну или вы просто тут поехавшие.
Аноним 17/03/17 Птн 11:03:22 #153 №13085 
>>13084
У многих и к книгам ненависть появляется, из-за того, что марьванна заставляла Тургенева читать. Так что ж теперь? Их можно только пожалеть.
Аноним 17/03/17 Птн 11:04:23 #154 №13086 
>>13084
ХЗ, у меня в школе было полное безразличие к точным наукам вообще, а вот после вуза как понеслооо
Аноним 17/03/17 Птн 12:07:07 #155 №13087 
Как лучше всего работать с Началами Эвклида?
Аноним 17/03/17 Птн 12:44:56 #156 №13088 
>>13087
Решил начать с нуля?
Аноним 17/03/17 Птн 12:48:21 #157 №13089 
>>13085
Это ты сейчас так тонко намекаешь на >>13075 или что?
Аноним 17/03/17 Птн 12:49:59 #158 №13090 
>>13087
Двачую на счет Евклида. Повторяю все с низов, учитывая, что в детстве ненавидел Геометрию из-за кучи теорем по сравнению с той же Алгеброй в школе. Как все это переваривать? Не беря в счет очевидные вещи, конечно же.
Аноним 17/03/17 Птн 13:20:29 #159 №13091 
>>13088
Да. Хочу развить еще и геометрическую интуицию и все такое подобное. Не подскажешь что-то по этому поводу?
>>13090
Такое, да.
Аноним 17/03/17 Птн 15:33:13 #160 №13095 
>>13087
Лучше всего вот так http://aleph0.clarku.edu/~djoyce/java/elements/elements.html
Ещё лучше взять что-нибудь посовременнее
Аноним 17/03/17 Птн 17:57:23 #161 №13099 
>>12775 (OP)
>Почему простые числа считаются такими важными?
Вопрос к профессорам из /math от завсегдатаев тупых вопрос треда из /b.
Аноним 17/03/17 Птн 18:08:43 #162 №13100 
>>13099
Простые числа образуют мультипликативный базис целых чисел. Свойства любого целого числа сводятся к свойствам простых сомножителей этого числа. Поэтому изучение целых чисел = изучение простых чисел.

Это как молекулярно-кинетическая теория в физике. Целые числа состоят из простых чисел так же, как физические тела состоят из атомов и молекул.
Аноним 17/03/17 Птн 18:09:27 #163 №13101 
>>13100
Охуеть. Мерси и дай Перельман тебе здоровья.
Аноним 17/03/17 Птн 20:43:56 #164 №13107 
>>13090
Вы ебанутые? Не лезь, она тебя сожрёт блядь! Загугли что такое движение, их классификацию. Всё! Дальше берешь линейную алгебру.
Аноним 17/03/17 Птн 21:17:34 #165 №13109 
>>13107
Стыдно как-то браться за что-то серьезное при этом плавая в школьной программе.
Аноним 17/03/17 Птн 21:19:06 #166 №13110 
>>13107
А после полного погружения в алшебру можно анализом занятся?
Аноним 17/03/17 Птн 21:43:50 #167 №13111 
Я тут почитал, что мозг всячески пытается избегать когнитивного диссонанса. А можно построить обучение математики так, чтобы возникали когнитивные диссонансы, ты искал бы решение и исправлял когнитивный диссонанс? Я практиковал это в нескольких вещах, было удивительно хорошо и просто. А как организовать это в математике? Как сделать, дабы возникали когнитивные диссонансы и потом ты мастерски их решал получая удовольствие? Возможно ли так вообще? Как вообще перестроить мышление для математики, если брать глобально?
Аноним 17/03/17 Птн 22:17:01 #168 №13113 
>>12973
это я знаю. Дальше как расписать?
Аноним 17/03/17 Птн 23:43:00 #169 №13116 
>>13111
>А можно построить обучение математики так, чтобы возникали когнитивные диссонансы
Дык оно всегда такое, собственно. Интереснее построить обучение математике так, чтобы диссонансов не было.
Аноним 18/03/17 Суб 01:57:13 #170 №13120 
>>13113
А что дальше расписывать то? Это вторая производная, по-видимому от температуры, по х-координате. Определяет выпуклость/вогнутость температурного профиля. Давай конкретный пример, в общем, тогда и подскажем тебе чего-то поконкретнее.
Аноним 18/03/17 Суб 04:03:04 #171 №13122 
Почему пустое множество является подмножеством любого множества?
Аноним 18/03/17 Суб 04:20:44 #172 №13123 
>>13122
Подмножество - это когда мы из множества берём часть элементов. Из любого множества можно взять ничего.
Аноним 18/03/17 Суб 04:48:44 #173 №13124 
>>13123

Непонятно все равно. По определению: (А⊂В)=∀х (х∈А⇒х∈В), но нам нечего брать.
Аноним 18/03/17 Суб 05:02:45 #174 №13125 
>>13124
Это не мешает утверждению быть истинным.
Аноним 18/03/17 Суб 05:11:04 #175 №13126 
>>13125
Каким образом? Если нам нечего брать, то мы не можем ничего доказать по определению, т.к. определение завязано именно на этом. Допустим, А – пустое множество, а так же А⊂В. Тогда ∀х (х∈А⇒х∈В). Но А=∅, ∄х∈А
В чем я неправ? Не понимаю.
Аноним 18/03/17 Суб 05:15:07 #176 №13127 
>>13126
Квантор всеобщности сам по себе не привязан ни к какому множеству. Для любого x - значит вообще для любого x. Всякий раз, когда какой-нибудь x принадлежит A, он принадлежит и B. Это выполнено для А=∅.
Аноним 18/03/17 Суб 05:19:03 #177 №13128 
>>13127
> Квантор всеобщности сам по себе не привязан ни к какому множеству.

Я понимаю, но он привязан к определению подмножества ведь. Через это подмножество определяется.

> Это выполнено для А=∅.

Можно понятнее как-то объяснить? Вообще не понимаю. Это ломает мне сознание.
Аноним 18/03/17 Суб 05:26:53 #178 №13129 
>>13127

> Всякий раз, когда какой-нибудь x принадлежит A, он принадлежит и B.

Но в нашем случае ∄х (х∈А⇒х∈В), следовательно, условие ведь не может выполняться.
Аноним 18/03/17 Суб 05:31:28 #179 №13130 
>>13128
>но он привязан к определению подмножества ведь
Нет. У тебя есть выражение, ∀х P(x). Ты не знаешь и не обязан знать, какая семантика у предиката P(x) и как он вообще устроен. Ты вообще можешь понятия не иметь, что у тебя есть какие-то A и B. У тебя есть чёрный ящик, который умеет принимать на вход какой-то объект и говорить в ответ "да" или "нет". И написанное выше означает просто, что чёрный ящик всегда скажет "да", что бы ты ему ни подсунул. Объект может быть абсолютно любым, который допускается твоей теорией. x = ∅, x = {∅}, x = π/1488!, x = Александр Гротендик. Всё что угодно. И всё это сработает, потому что если в импликации предпосылка ложна, то это автоматически делает всю импликацию истинной.
>>13129
>∄х (х∈А⇒х∈В)
Wrong.
∄х х∈А
но
∀х (х∈А⇒х∈В)
Второе, между прочим, следует из первого.
Аноним 18/03/17 Суб 05:38:11 #180 №13131 
>>13130

Все равно непанятна. Кстати, я так же не понимаю, почему импликация с ложной посылкой истинна и как этим можно обосновать истинное следствие из этой посылки, ведь если следствие ложно, то, опять же, импликация истинна.

> Wrong

Но каким образом? Почему?

> ∄х х∈А
> но
> ∀х (х∈А⇒х∈В)

Но такого не может быть, если не существует никакого х. Нечем просто. Ну вообще. Любые попытки представить себе это просто невозможны.
Аноним 18/03/17 Суб 05:52:57 #181 №13132 
>>13131
>Кстати, я так же не понимаю, почему импликация с ложной посылкой истинна
Потому что импликация всегда работает в связке с квантором всеобщности.
∀х (P(x)⇒Q(x))
Если бы импликация при ложной предпосылке была ложной, то все выражения такого вида были бы ложными, если только оба предиката не являются совсем уж тавтологиями. Кроме того, если ты попробуешь написать какую-то другую таблицу истинности для импликации, то очень быстро поймёшь, что так быть не может.
>не существует никакого х
Существует, вот он: x = ∅. Это как минимум. В твоей теории присутствует пустое множество, раз ты о нём рассуждаешь. А если у тебя есть ещё какие-то правила конструирования множеств из уже существующих, то иксов у тебя вообще целая куча. Правда, ни один из них не является элементом A, но это ничему не мешает.
Аноним 18/03/17 Суб 06:04:45 #182 №13133 
>>13132

> Если бы импликация при ложной предпосылке была ложной, то все выражения такого вида были бы ложными

И как этим можно обосновать использование импликации с ложной посылкой в качестве получения истинных утверждений?

> Кроме того, если ты попробуешь написать какую-то другую таблицу истинности для импликации, то очень быстро поймёшь, что так быть не может.

А нельзя просто избавиться от этих дурацких парадоксов, дав какое-то более качественное определение?

> Существует, вот он: x = ∅. Это как минимум.

Тогда х∈А⇒∅∈∅⇒∅={∅}, что противоречит аксиоме регулярности.

> то иксов у тебя вообще целая куча

Но мы же допускаем, что А=∅, значит их просто нет. Мы ничем не оперируем. Вообще.
Аноним 18/03/17 Суб 06:12:53 #183 №13134 
>>13133
>И как этим можно обосновать использование импликации с ложной посылкой в качестве получения истинных утверждений?
Понятия не имею, о чём ты.
>А нельзя просто избавиться от этих дурацких парадоксов, дав какое-то более качественное определение?
А ты попробуй, говорю же. Возьми и попробуй определить импликацию как-то по-другому. Вариантов-то всего 16, их легко перебрать и убедиться, что больше ни один из них импликацией быть не может.
> Тогда х∈А
Нет.

У тебя не написано ∀х (х∈А). Это неверно, причём не только для A = ∅. Множества A с таким свойством вообще не существует.

У тебя написано ∀х (х∈А => х∈В). Это совершенно разные вещи.
Аноним 18/03/17 Суб 06:18:56 #184 №13135 
>>13134

Но нету ведь такого х, чтобы было х∈А => х∈В. По сути х вообще не принадлежит А. Ибо в А вообще нет элементов. Ну не существует такого х, чтобы принадлежал А. Значит и не существует такого х, принадлежащего А, который принадлежал бы и В. Значит условие не выполняется для пустого множества.
Аноним 18/03/17 Суб 06:22:07 #185 №13136 
>>13135
>Но нету ведь такого х, чтобы было х∈А => х∈В
Абсолютно любой x, который ты в состоянии придумать. Например, x = ∅.
>По сути х вообще не принадлежит А
Кажется, до тебя начинает доходить. Осталось только осознать, что в выражении (х∈А => х∈В) ничего не говорится о том, что x должен принадлежать A.
Аноним 18/03/17 Суб 06:27:11 #186 №13137 
>>13134

> больше ни один из них импликацией быть не может

Мне кажется, что тебе промыли мозги, и любую самую интуитивно очевидную логику ты отрицаешь. Либо я в ∅ тупой. Т.е. импликация, которая содержит в себе суждения, которые связаны между собой, импликацией быть не может? А то некоторые доказательства таки даются в стиле "у меня большой пенис"→"я еб@л твою жену".
Аноним 18/03/17 Суб 06:34:40 #187 №13138 
>>13136

> Например, x = ∅.

В таком случае (х∈А => х∈В) – противоречит аксиоме регулярности, та БЛДЖАД.

> Осталось только осознать, что в выражении (х∈А => х∈В) ничего не говорится о том, что x должен принадлежать A.

Допусти, что он НЕ ПРИНАДЛЕЖИТ А, тогда бессмысленно говорить о подмножестве, т.к. его нет нихера.
Аноним 18/03/17 Суб 06:40:20 #188 №13139 
>>13138

Его нечем задать
Аноним 18/03/17 Суб 06:42:13 #189 №13140 
>>13137
А мне ничего не кажется. Я ясно вижу, что ты не понимаешь очевидных вещей. Это, однако, ещё не значит, что ты тупой, хоть и повышает вероятность подобного. И нет, когда я говорю "возьми и сделай", это не фигура речи. Я действительно предлагаю тебе убедиться самому. Разве так сложно нарисовать 16 вариантов таблицы 2x2, внимательно на них посмотреть и закрыть для себя этот вопрос раз и навсегда?
>тебе промыли мозги
Воу, палехчи. Сегодня ты пишешь, что мне промыли мозги кто и зачем, интересно?, а завтра разоблачаешь еврофашистский заговор оффициальной науки и рисуешь православные таблицы электроатомов русов. Аккуратнее с этим дерьмом.
>>13138
>противоречит аксиоме регулярности
Не противоречит. Нигде не утверждается, что ∅ ∈ ∅. Из (х∈А => х∈В) это никоим образом не вытекает.
>Допусти, что он НЕ ПРИНАДЛЕЖИТ А, тогда бессмысленно говорить о подмножестве
Для любого множества найдутся элементы, которые ему не принадлежат. Что ж теперь?
Аноним 18/03/17 Суб 06:52:46 #190 №13141 
>>13140

> Я ясно вижу, что ты не понимаешь очевидных вещей.

Мне кажется, что это совсем не очевидные вещи. Тут сознание уже крошится.

> И нет, когда я говорю "возьми и сделай", это не фигура речи. Я действительно предлагаю тебе убедиться самому.

То же самое с этой хуйнюшкой (х∈А => х∈В)? Да, я действительно не понимаю, как может образоваться подмножество из ничего. По твоей логике ∀х (х∈А⇒х∈В) – совсем не противоречит тому, что ∄х∈А. Но если ∄х∈А, то х∈А⇒х∈В неверно, только потому, что х∈А не выполняется никак. Не из чего составить множество. Значит не существует никакого х∈А. Не существует его. Это точно, 100%. Значит уже такая херня под определение не подходит, поскольку нарушается его целостность.
Аноним 18/03/17 Суб 06:56:44 #191 №13142 
>>13141
> Но если ∄х∈А, то х∈А⇒х∈В неверно, только потому, что х∈А не выполняется никак.
Ему и не обязательно выполняться, чтобы импликация была верной.
>Не из чего составить множество
Так в нём и нет элементов, всё правильно. На то оно и пустое.
Аноним 18/03/17 Суб 06:58:55 #192 №13143 
>>13142

Смысл в том, что, ЫЫЫААААА!!!!(эх), получается, что х(не принадлежит)А⇒х∈В. Это уже не определение подмножества, а что-то другое совершенно.
Аноним 18/03/17 Суб 07:07:37 #193 №13144 
>>13143
Это действительно совсем другое утверждение, и оно не имеет никакого отношения к нашей ситуации. Я тебе объясняю, что именно ИСХОДНАЯ импликация (х∈А⇒х∈В) истинна ровно потому, что x ∉ A, независимо от того, верно ли х∈В или нет. Ложность условия автоматически влечёт истинность импликации, всегда.
Аноним 18/03/17 Суб 07:16:55 #194 №13145 
>>13144

И да, я понимаю, что если ∄х∈А, то это еще не значит, что ∄х∈В. А так же я понимаю, что ∀х (х∈А⇒х∈В) при ∄х∈А не становится неверным, ибо В может включать в себя разные х(но может и не все), однако что-то мне интуитивно подсказывает, что в этом есть некоторые неточности, а может и противоречия. Выглядит как-то слишком странно, но никаким образом не могу объяснить внятно, что именно меня смущает. Я привык к точным определениям, а это какое то не такое. И даже не могу объяснить в достаточной степени, что меня смущает.
Аноним 18/03/17 Суб 07:34:27 #195 №13146 
Mullerlayer0.gif
>>13145
Это обычное когнитивное искажение, вроде пикрелейтед. Человек, который до этого с таким не сталкивался, и не имеет математической культуры, будет убеждать окружающих, что красный отрезок длиннее. И сколько бы ты ни прикладывал линейку, его это не убедит. Пустое множество действительно устроено несколько шиворот-навыворот с точки зрения житейского здравого смысла, но с точки зрения логики здесь всё в порядке, никаких противоречий и неточностей нет. Похожие предрассудки многие века мешали людям придумать позиционные системы счисления - для этого нужна цифра ноль. Но ведь ноль - это "ничего", а как "ничего" может быть цифрой, т.е. "чем-то"? Ты про ноль знаешь с детства, поэтому у тебя он не вызывает такого диссонанса, как у людей в античности. А вот базовую теорию множеств в начальной школе не проходят, увы. Поэтому у многих проблемы с осознанием некоторых тривиальных вещей. Как это, чётных чисел столько же, сколько и всех натуральных чисел? Так не бывает, мне здравый смысл подсказывает. Ну и что, что биекция? Не убедительно! И только привычка позволяет перестать воспринимать подобные вещи в штыки, и выработать здравый смысл, адекватный математической реальности.
Аноним 18/03/17 Суб 07:37:45 #196 №13147 
>>13145

(А⊂В) = ∀х (х∈А⇒х∈В)

Больше всего проблема именно в том, что ∄х∈А.

> Как это, чётных чисел столько же, сколько и всех натуральных чисел? Так не бывает, мне здравый смысл подсказывает. Ну и что, что биекция? Не убедительно!

Это я сразу понял, без всяких проблем. Понял и запомнил. А тут ни с чем интуитивным связать не могу. Обычно связываю с чем-то таким, что может нести похожий смысл, а тут вообще беда.
Аноним 18/03/17 Суб 07:45:02 #197 №13148 
>>13147

Мне нужно какое-то более подробное доказательство, чтобы разжевывало на пальцах. Есть какая-то тавтология к ∀х (х∈А⇒х∈В)?
Аноним 18/03/17 Суб 08:17:29 #198 №13149 
>>13146

> Но ведь ноль - это "ничего", а как "ничего" может быть цифрой, т.е. "чем-то"?

Я могу согласиться с тем, что А/А=∅, но не совсем могу согласиться с тем, что пустое множество является подмножеством любого множества.
Аноним 18/03/17 Суб 08:21:36 #199 №13150 
>>13109
1) Ты всё все равно не запомнишь
2) Школьная программа - говно
Прекрати делить что-то на "серьезное" и "не серьезное". Всегда хочется с вертухи за "вышмат", "серьезная"... прописывать в еблет.
>>13110
Ещё топология нужна
Аноним 18/03/17 Суб 08:27:13 #200 №13151 
>>13122
Пусть А не подмножество множества В, тогда в А есть элемент x, который не принадлежит B.

В пустом множестве нет элемента, который не принадлежал бы любому множеству. Тогда пустое множестве является подмножеством любого множества.
Аноним 18/03/17 Суб 08:27:33 #201 №13152 
>>13150

Анон, может ты пояснишь за пустое множество?
Аноним 18/03/17 Суб 08:27:52 #202 №13153 
>>13152
выше пояснил
>>13151
Аноним 18/03/17 Суб 08:34:25 #203 №13154 
>>13151

Пусть каждый элемент x, являющийся элементом множества A, является элементом множества B. Но если A пусто, то не существует такого x, что принадлежит A, и высказанное только что утверждение ложно. Что на это можно сказать?
Аноним 18/03/17 Суб 08:42:46 #204 №13155 
>>13154
Общее утверждение ложно тогда и только тогда, когда можно привести контрпример.
Аноним 18/03/17 Суб 08:46:11 #205 №13156 
>>13155

Тут вся суть в том, что по определению подмножество какого-то множества является совокупностью объектов, которые находятся в этом каком-то множестве. Но если в подмножестве нет объектов? Вот этого я и не могу понять.
Аноним 18/03/17 Суб 08:47:26 #206 №13157 
>>13156

А вводить такой элемент, как "ничего", мне кажется слишком стремным.
Аноним 18/03/17 Суб 08:47:56 #207 №13158 
>>13154
Ты заебал.
>Пусть каждый элемент x, являющийся элементом множества A, является элементом множества B.
Да, тогда А - подмножество В.
>Но если A пусто, то не существует такого x, что принадлежит A, и высказанное только что утверждение ложно.
Если в А нет такого х, тогда в В нет такого же х. Всё! Это не значит, что утверждение ложное.
Аноним 18/03/17 Суб 08:50:32 #208 №13159 
>>13155
>привести
Вернее, доказать, что контрпример существует.
А то попросят тебя контрпримеры к получающимся выбором утверждениям наприводить.
Аноним 18/03/17 Суб 08:52:04 #209 №13160 
>>13159
Ну это я и подразумевал под "можно".
Аноним 18/03/17 Суб 08:55:31 #210 №13161 
>>13156
Каждое утверждение об элементах пустого множества истинно.
Для любого предиката P каждый элемент пустого множества обладает свойством P. Множество всех автобусов на Марсе пусто. Поэтому каждый автобус на Марсе красный. Кроме того, каждый автобус на Марсе не красный. Каждый элемент пустого множества равен сам себе, а также не равен сам себе. Это называется vacuous truth.

Каждый элемент пустого множества принадлежит A. Кроме того, каждый элемент пустого множества не принадлежит A.
Аноним 18/03/17 Суб 08:55:44 #211 №13162 
>>13158

> Если в А нет такого х, тогда в В нет такого же х.
> каждый элемент x, являющийся элементом множества A, является элементом множества B.

Все равно что-то не дает мне покоя... Каждый элемент х, не являющийся элементом А, может являться элементом В тамщето, ты неправ.
Аноним 18/03/17 Суб 08:57:58 #212 №13163 
>>13161
> Каждое утверждение об элементах пустого множества истинно.
Каждое ОБЩЕЕ утверждение.
Аноним 18/03/17 Суб 09:00:41 #213 №13164 
>>13161

> Для любого предиката P каждый элемент пустого множества обладает свойством P.

Это утверждение ложно, т.к. в пустом множестве нет элементов, это значит, что нет элементов, обладающих свойством Р. Дальше там ахинея пошла.
Аноним 18/03/17 Суб 09:08:47 #214 №13165 
>>13164
Утверждение ∀x(P) ложно тогда и только тогда, когда ∃x(¬P).
Согласен?
Аноним 18/03/17 Суб 09:09:36 #215 №13166 
>>13162
Да, ошибся.
Если в В нет такого х, тогда в А нет такого же х.
Аноним 18/03/17 Суб 09:11:23 #216 №13167 
>>13165

Да, согласен.
Аноним 18/03/17 Суб 09:12:45 #217 №13168 
>>13166

Ага, а в А нет никакого х. Понимаешь?
Аноним 18/03/17 Суб 09:17:15 #218 №13169 
>>13168
И хули?
>Если в В нет такого х, тогда в А нет такого же х.
Если А - пусто, то никакого противоречия тут нет, т.к. в А нет никакого элемента, который не принадлежал бы В.
Аноним 18/03/17 Суб 09:19:17 #219 №13170 
>>13167
1. Утверждение A→B равносильно утверждению ¬A ∨ B.
2. Отрицание утверждения A→B равносильно утверждению ¬(¬A ∨ B), что, в свою очередь, равносильно утверждению A ∧ ¬B.
3. Утверждение P∧Q→P - аксиома.

Рассмотрим утверждение ∀x(x∈M → P(x)).
Оно равносильно утверждению ∀x(¬x∈M ∨ P(x)).
Это утверждение ложно тогда и только тогда, когда ∃x(x∈M ∧ ¬P(x)).
Из этого утверждения следует, что ∃x(x∈M).

Пусть M = ∅. Это значит, что ¬∃x(x∈M).
Теорема. ∀x(x∈M → P(x)) для любого P.
Доказательство.
Предположим, что ¬∀x(x∈M → P(x)).
Значит, ∃x(x∈M ∧ ¬P(x)).
Значит, ∃x(x∈M).
Пришли к противоречию. Предположение ложно.
Значит, ∀x(x∈M → P(x)).
Аноним 18/03/17 Суб 09:21:18 #220 №13171 
>>13169
Воспользуюсь тем же
А пм В, если ∀x ∈ A -> x ∈ B - верно
Т.к. 0 -> 1 - верно, то пустое множество явл. подмножеством любого множества.
Аноним 18/03/17 Суб 09:24:45 #221 №13172 
>>13169

ТА СУКА, ТЫ СЛЕПОЙ. О подмножестве А можно сделать такие утверждения:

В А нет никакого элемента, который принадлежал бы В.
В А нет никакого элемента, который не принадлежал бы В.
Как ни крути, нет в них таких элементов.

Далее.

Если А включено в В, то каждый элемент А принадлежит В. Но если А не содержит в себе элементов, то В не принадлежат никакие элементы А, т.к. их там НЕТ. Можно сделать только один вывод: А не является подмножеством В.
Аноним 18/03/17 Суб 09:28:52 #222 №13173 
>>13172

Сука, вот не оправдывайтесь. Тут все завязано на элементах
Аноним 18/03/17 Суб 09:31:25 #223 №13174 
>>13172
Слышь блядь, ебало на ноль, пидор.

А - подмножество В, если каждый элем. А принадлежит В => А не подмножество В, если в А есть элемент, который не принадлежит В. Т.к. в пустом множестве нет элемента, который не принадлежит В, тогда пустое не является НЕ ПОДМНОЖЕСТВОМ. 3-тьего варианта есть, остается только ПОДМНОЖЕСТВО.
Аноним 18/03/17 Суб 09:31:51 #224 №13175 
>>13174
>3-тьего варианта есть
нет
fix
Аноним 18/03/17 Суб 09:34:59 #225 №13176 
>>13174

Т.е. то, что элементов нет – тебя это никак не волнует? Само понятие пустого множества противоречиво. Как и понятие множества, включающего все множества, которые не включают сами себя.
Аноним 18/03/17 Суб 09:35:00 #226 №13177 
>>13175
Если в А нет элементов, то утверждение: если х лежит в А, то х лежит в В, не будет ЛОЖНЫМ, оно будет ложным, только если х лежит в А, но х не лежит в В(1 → 0 | гугли импликацию)
Аноним 18/03/17 Суб 09:35:49 #227 №13178 
>>13176

*включают т.е. принадлежат
Аноним 18/03/17 Суб 09:37:21 #228 №13179 
>>13176
>понятие пустого множества противоречиво
ПАШОЛ В ЖОПУ МУДАК!!1
ТЫВ ГГОВНО ГОВНО ОБДРИСЛЫЙ КАЛ ВОТ ТЫ КТО!!!1
Аноним 18/03/17 Суб 09:37:45 #229 №13180 
>>13177

Этот со своими абсрактными импликациями всякими с ума сходит. Это нужно доказывать с содержательной точки зрения, раз формально, как ты говоришь, шиворот навыворот определено
Аноним 18/03/17 Суб 09:39:38 #230 №13181 
>>13176
Как ты заебал, просто доебали твои ахуительные истории.

Множество - фигура на плоскости. Если фигура_1 лежит внутри фигуры_2, то множество_1 явл. подмножеством множества_2. Пустое множество это фигура из 0 точек. Она может находится внутри любой фигуры.
Аноним 18/03/17 Суб 09:45:14 #231 №13182 
>>13180
Я нихуя не говорю.
Определение быть подмножеством можно дать через не_быть подмножеством и через это определение не_быть подмножеством легко показать, что пустое множество - подмножество любого множества. Что я в общем-то выше и писал. Если хочешь строгости, то юзай импликацию.
Аноним 18/03/17 Суб 09:46:37 #232 №13183 
>>13181

Что за шизофрения)) и лучше было бы дать определение такое: если отрезок А лежит на прямой В, то множество точек А включено в В. Но как ноль точек может быть включено в множество? Ты что несешь? Чтобы множество было включено в другое, то оно должно из чего-то состоять, блядь. Это из определения следует.
Аноним 18/03/17 Суб 09:50:19 #233 №13184 
>>13182

Так вот в том и суть, что в двух определениях ничего не выходит. Я понимаю вашу точку зрения, но что делать с теми вот элементами в определении??
Аноним 18/03/17 Суб 09:52:45 #234 №13185 
>>13184
А - подмножество В, если каждый элем. А принадлежит В => А не подмножество В, если в А есть элемент, который не принадлежит В. Т.к. в пустом множестве нет элемента, который не принадлежит В, тогда пустое не является НЕ ПОДМНОЖЕСТВОМ. 3-тьего варианта нет, остается только ПОДМНОЖЕСТВО.
Где? Всё подходит.
>>13183
>Но как ноль точек может быть включено в множество? Ты что несешь?
Без задней мысли. Причем может включено даже в самого себя.
Аноним 18/03/17 Суб 09:54:37 #235 №13186 
>>13185

Я тебе теперь по начальному определению толкую, дура4ок.

Если А включено в В, то каждый элемент А принадлежит В. Но если А не содержит в себе элементов, то В не принадлежат никакие элементы А, т.к. их там НЕТ. Можно сделать только один вывод: А не является подмножеством В. И дальше ни о каких элементах речи идти не может.
Аноним 18/03/17 Суб 10:07:52 #236 №13187 
>>13186
Ладно, заебал. Смотри:
Пусть А - не_подмножество В. Добавим туда элемент из B. A+{b} будет подмножеством В? нет
Теперь представим, что 0(пустое) все же не подмножество А, добавим туда элемент из A: 0+{a}, тогда 0+{a} не должно быть подмножеством А, но оно подмножество А, тогда предположение, что 0 - не подмножество НЕ ВЕРНО.
Аноним 18/03/17 Суб 10:09:20 #237 №13188 
>>13186

Действительно, если ни один элемент А не принадлежит В, то А не является подмножеством В.
Аноним 18/03/17 Суб 10:20:08 #238 №13189 
>>13187

Не, нихуя, я не про это толкую. Мы рассматриваем ситуацию не во времени, тут или/или, третьего/четвертого не дано. Аналогия с брадобреем. Брадобрей бреет всех, кто не бреет себя сам. Бреет ли брадобрей сам себя? Вы просто стараетесь упростить все формальным методом. Получается, что он в любом случае себя не побреет, потому что это нарушит его нерушимое правило. Но тут вне времени, он себя или бреет, или нет. Точно так же и тут. Порочный круг начинается на определении, на понятии элемента.
Аноним 18/03/17 Суб 10:24:32 #239 №13190 
>>13189
Здесь нет никаких проблем.
> А не подмножество В, если в А есть элемент, который не принадлежит В. Т.к. в пустом множестве нет элемента, который не принадлежит В, тогда пустое множество подмножество любого множества.

Ты смотри там, дальше, от равномощности квадрата и отрезка не охуей.
Аноним 18/03/17 Суб 10:51:50 #240 №13191 
>>13190

Ты как будто нихуя не понимаешь о чем я толкую. Попробуй вникнуть, а не кукарекать
Аноним 18/03/17 Суб 11:47:40 #241 №13192 
Screenshot2017-03-18-11-44-44-434com.miui.gallery.png
Господа, как научиться упрощать выражения такого вида? Что обозначает "/" в этих функциях? Есть какой-то подробный гайд, ну ли хотя бы разобранный пример. С меня два ароматных зеленых чая с манго
Аноним 18/03/17 Суб 11:50:13 #242 №13193 
>>13192
https://ru.wikipedia.org/wiki/%D0%9C%D0%BD%D0%BE%D0%B6%D0%B5%D1%81%D1%82%D0%B2%D0%BE
Аноним 18/03/17 Суб 11:50:59 #243 №13194 
>>13191
А или неА = 1
Иди на хуй.
Аноним 18/03/17 Суб 11:51:58 #244 №13195 
>>13192
Кстати, а откуда лист?
Аноним 18/03/17 Суб 11:54:02 #245 №13196 
>>13195
Какая-то методичка преподавателя по дискретке. Там задали прорешать несколько номеров, а вот как их делать (по диаграммам Вена / просто упрощать) не объяснили.
18/03/17 Суб 11:56:29 #246 №13197 
>>13196
Меня тут смутило только умножение в 17 примере. Скорее всего это пересечение, но точно я не уверен. А ещё какие-то условия даны?
Аноним 18/03/17 Суб 12:00:03 #247 №13198 
>>13197
Нет, не даны. Задание гласит: "Упростите выражения.", не слова больше. Меня вот интересует одно, к пересечениям и объединениям можно применять те же упрощения, что и к логич. выражениями с дизъюнкцией, конъюнкцией?
Аноним 18/03/17 Суб 12:57:02 #248 №13199 
>>13194

Сам пошел нахуй. Я тебе все обосновал.
Аноним 18/03/17 Суб 14:40:57 #249 №13200 
>>13164
Ой, всё. Если ты просишь объяснений, получаешь их, а потом игнорируешь и продолжаешь нести ту же самую хуйню по десятому кругу, то тебе здесь не место. Пиздуй в тред к предельному дебилу, вы с ним явно найдёте общий язык.
Аноним 18/03/17 Суб 14:59:57 #250 №13201 
>>13199
>Порочный круг начинается на определении, на понятии элемента.
А теперь приведи мне определение элемента. Я тебе несколько написал простым языком, почему пустое множество подмножество любого множества, но ты продолжаешь копротивлятся, не хотя поменять в своём манямирке определение быть подмножеством на эквивалентное не быть подмножеством, из которого легко получить обоснование твоего вопроса.
Аноним 18/03/17 Суб 15:22:21 #251 №13203 
Magicnotfornoobs.png
>>13170
Аноним 18/03/17 Суб 15:38:21 #252 №13204 
Привет, матаны. Занимаетесь ли вы какой-нибудь умственной гимнастикой для разминки перед работой/учебой? Посоветуйте упражнений для разгона, а то в последнее время начал безбожно тупить, концентрация и внимание тоже ни к черту.
Аноним 18/03/17 Суб 15:51:35 #253 №13205 
>>13146
>чётных чисел столько же, сколько и всех натуральных чисел
Какой пруф?
Аноним 18/03/17 Суб 15:59:13 #254 №13206 
>>13205
Кардинал множества натуральных равен кардиналу множества четных. Элементарно, 2=21, 4=22, 2n=2*n, ... Далее по индукции заключаем, что каждому четному числу можно сопоставить натуральное. С отрицательными - по аналогии.
Аноним 18/03/17 Суб 15:59:31 #255 №13207 
>>13206
Я мимо проходил, если что
Аноним 18/03/17 Суб 16:20:59 #256 №13208 
>>13205
f(n) = 2n - биекция.
Аноним 18/03/17 Суб 16:42:48 #257 №13209 
>>13204
В треде про доказательства теорем анон советовал neuronation.
Аноним 18/03/17 Суб 17:13:59 #258 №13210 
>>12862
> конкрит матматикс
Не слушай, не читай это говно. Книга очень сложная, но не своими концепциями (любой школьник эти концепции способен осилить, я не преувеличиваю), а тем, что:
1) написана как художественный текст, т е там нет нормального математического изложения в стиле "определение-теорема-доказательство", читать очень тяжело и неприятно;
2) это по сути сборник школьных (именно школьных) олимпиадных задач на определенную тематику. Если хочешь именно получить базу для изучения computer science, гугли mit mathematics for computer science. Задачи там реально для даунов погромистов, вся нужная теория есть и излагается намного лучше.

> сикп
Эх, щас бы на scheme писать в 2017...

Никогда не слушай тех, кто советует типа "классические" книги: Демидовича, Кнута, Рудина и т. д. Это люди с мышлением "деды говно жрали и мы будем". В 2017 году надо думать не о том как разорвать себе жопу олимпиадными задачами, а о том, как побыстрее изучить нужные и интересные разделы, потому что доступной информации слишком много и естественным состоянием для тебя должно быть "бля, я не успеваю изучить такие-то и такие-то штуки", а не "бля, я не могу решить пример номер 1488 из Демидовича".
Аноним 18/03/17 Суб 17:25:03 #259 №13211 
>>13210
>Никогда не слушай тех, кто советует типа "классические" книги: Демидовича, Кнута, Рудина и т. д.
А шо по анализу чтать??77
Аноним 18/03/17 Суб 17:33:27 #260 №13214 
>>13211
Ну зависит от целей. Если тебе нужен учебник для прикладников, то хз, не могу ничего подсказать. Если для математиков, то Pugh: Real Mathematical Analysis и Львовский: Лекции по математическому анализу. Львовский сложнее намного и упражения там очень хардкорные, советую их просматривать и пытаться решать, но отступаться от задачи, если не получилось решить за n минут.
Аноним 18/03/17 Суб 17:38:53 #261 №13215 
>>13214
Хотя, если ты ньюфаг и не привык по несколько дней думать над задачей , а потом все равно не решить, то можешь пытаться все подряд решать, чтобы жопу натренировать.
Аноним 18/03/17 Суб 18:02:04 #262 №13217 
>>13214
>>13210
>В 2017 году надо думать не о том как разорвать себе жопу олимпиадными задачами
Мань, олимпиады хорошо смекалочку развивают, в матеше без этого никак.
>а о том, как побыстрее изучить нужные и интересные разделы
Это какие? Гомологии? Топологии? Или что там популярно у школьников? Ах да, вспомнил, сейчас популярны модули на кольцами.
Может, перечислишь те разделы в которые надо вкатиться быстро, решительно?

Аноним 18/03/17 Суб 18:33:45 #263 №13218 
>>13217
> Гомологии? Топологии?
Как вариант.
> Может, перечислишь те разделы в которые надо вкатиться быстро, решительно?
"Надо" - это очень субъективное понятие, не хочу никому ничего навязывать. Если тебе надо круто решать олимпиады - дрочи Конкретную Математику, в этом плане книга хорошая, тут не поспоришь.
Аноним 18/03/17 Суб 18:40:54 #264 №13219 
Щас мы тут выясним что математики не считают нихуя, а просто доказывают теоремы.
А счет для быдла. Математики выше этого. Интегралы это математический спид, топология это жопоебля, все в теорию множеств.
Аноним 18/03/17 Суб 18:45:30 #265 №13220 
>>13219
> математики не считают нихуя, а просто доказывают теоремы. А счет для быдла. Математики выше этого.
То есть ты это до сих пор не осознал?
Аноним 18/03/17 Суб 19:01:25 #266 №13221 
>>13220
Математики считают, ты просто лицемерный лжец.
Аноним 18/03/17 Суб 20:11:52 #267 №13222 
Capture.PNG
Анон, что значит такая запись дифура?
Аноним 18/03/17 Суб 20:18:07 #268 №13223 
>>13222
Сигма_р и u - функции, которые надо найти.
h - известная функция.
А вот что значит d(sigma_r h)/dr не пойму. Это производная от двух функций или что? Никогда такого не видел.
Аноним 18/03/17 Суб 20:36:04 #269 №13224 
>>13217
>Мань, олимпиады хорошо смекалочку развивают, в матеше без этого никак.
Как же раньше без олимпиад смекалочку та развивали, и не понять!
18/03/17 Суб 20:50:25 #270 №13226 
>>13224
Ты хочешь вернуться к образу жизни безолемпиадных дидов, ирод?!
Аноним 18/03/17 Суб 22:09:19 #271 №13228 
>>13200
>>13201

Дауны не могут понять, что элементиков нету.
Аноним 18/03/17 Суб 23:49:06 #272 №13229 
>>13228
Как скажешь. Теперь уёбывай.
Аноним 19/03/17 Вск 00:44:35 #273 №13230 
>>13224
Раньше и без модулей как-то обходились. Тяжёлые были времена, что и говорить.
Аноним 19/03/17 Вск 01:09:14 #274 №13231 
>>13209
спасибо
Аноним 19/03/17 Вск 05:13:55 #275 №13232 
Сорт оф оффтоп, может на другой доске спросить стоит, но не знаю где. Короче, где/как вы пишете? Писать на бумаге имеет свои недостатки — неудобно каталогизировать/править/пересылать свои записи, к тому же, наверное тут кто как, но лично я печататю куда быстрее и легче, чем пишу, поэтому если ваш конспект состоит не из сплошных формул (а такое бывает относительно редко) — печатать легче. Опять же, значительная часть работы так и так в компьютере происходит — большинство вычислений, различные визуализации и т.д. Даже матрицы вручную перемножать — какая-то пустая трата времени. Печатать, в свою очередь, неудобно формулы — набирать текст сразу в LaTeX это пиздец, абсолютно бесперспективное занятие, к тому же иногда бывает удобно накорябать какой-нибудь рисунок от руки и т.п.

Так какова же она, идеальная система конспектирования в 2017 году? Какой workflow и сочетание инструментов обеспечит максимальное удобство?
Аноним 19/03/17 Вск 07:31:28 #276 №13235 
>>13232

Доска с мелом тебе в помощь
Аноним 19/03/17 Вск 07:32:20 #277 №13236 
>>13235
Недостатки уже названы.
Аноним 19/03/17 Вск 08:11:39 #278 №13237 
Безымянный.png
Имеется непрерывная дробь. Поясните, появившаяся единица в числителе как-то связана с переворачиванием дроби? Ничего в интернетах не нахожу.
Аноним 19/03/17 Вск 08:30:02 #279 №13238 
111.gif
>>13237
Аноним 19/03/17 Вск 10:21:57 #280 №13239 
>>13232
Я веду не конспекты, а дневник, в котором пишу все что меня удивило/показалось интересным, что вызвало сложности и т д. Каждое определение/теорему не вижу смысла записывать. Дневник в техе пишу.

> набирать текст сразу в LaTeX это пиздец,
Нет, почему? Придрочиться просто надо.

> к тому же иногда бывает удобно накорябать какой-нибудь рисунок от руки и т.п.
Рисую от руки -> фоткаю -> посылаю фотку сам себе в вк -> includegraphics.
Аноним 19/03/17 Вск 10:23:41 #281 №13240 
>>13239
Алсо, для скорости я стараюсь использовать слова вместо формул тогда, когда это возможно.
Аноним 19/03/17 Вск 10:27:08 #282 №13241 
>>13232
Еще я сейчас прочитал твой пост и подумал, что для удобства поиска в будущем можно ставить тэги в духе [гамалогии], [тапалогии] и т д. Пока что не ставлю, потому что не так давно начал и у меня пдфка страниц 50.
Аноним 19/03/17 Вск 10:32:18 #283 №13242 
>>13232
Пробовал писать конспекты - пришел к выводу, что это неоправданная трата времени. Обычно читаешь разные книги + википедию + math.stackexchange, обозначения везде разные, порядок изложения разный и получается так, что приводить все к своим обозначениям и своему порядку изложения - это слишком большая трата времени, которая точно не будет оправдана, если нет цели написать свой учебник. Запоминание должно происходить за счет обдумывания и решения задач, а не за счет того, что ты пишешь.
Аноним 19/03/17 Вск 10:39:48 #284 №13243 
>>13230
Да, ты прав! Спасибо деду за ^победу^ модули.
Аноним 19/03/17 Вск 10:40:39 #285 №13244 
>>13243
Спасибо деду за победу модули.
быстрофикс
Аноним 19/03/17 Вск 10:44:44 #286 №13245 
>>13230
Без них и сейчас можно обойтись.
Аноним 19/03/17 Вск 10:53:17 #287 №13246 
Так полином это равно многочлен, но просто красивее название? Полиномиальная функция равно многочленной функции типа, да?
Аноним 19/03/17 Вск 10:53:56 #288 №13247 
Верно ли, что функция, которая переводит точку на S^1 в угол из [0; 2pi), который ей соответствует, переводит открытые множества в открытые, но не является непрерывной?
Аноним 19/03/17 Вск 11:01:05 #289 №13248 
>>13247
Да. Что открытые в открытые - очевидно, а пример открытого множества, прообраз которого не открыт, [0;pi). Открытого в индуцированной с R топологии, конечно.
Аноним 19/03/17 Вск 11:06:36 #290 №13249 
>>13248
> Что открытые в открытые - очевидно
Для меня это только геометрически очевидно. Но в принципе, формально я это доказывать не хочу, поэтому и попросил проверить. Спасибо.
Аноним 19/03/17 Вск 11:29:54 #291 №13250 
>>13239
А можешь скинуть примеры, пожалуйста?
Аноним 19/03/17 Вск 11:55:56 #292 №13251 
Selection015.png
Selection016.png
Selection017.png
>>13250
Пикрелейтед 1 сейчас писал, 2 и 3 - старое с картинками. Утверждения на 2 и 3, конечно, не заслуживают, чтобы я их записывал, просто я хотел сохранить картинки, которые придумал для иллюстрации.
Аноним 19/03/17 Вск 12:04:39 #293 №13252 
>>13251
Кстати, я сейчас не могу понять, почему верно то, что я писал на пикрелейтед 2 и 3, вроде там хуйня какая-то.
Аноним 19/03/17 Вск 12:05:06 #294 №13253 
Какие же вы крутые, математики, я в шоке
Аноним 19/03/17 Вск 12:05:44 #295 №13254 
>>13252
Хотя не, все правильно.
Аноним 19/03/17 Вск 13:05:47 #296 №13255 
>>13253
А ящик все равно через вольфрам с маткадом решают.
Аноним 19/03/17 Вск 13:13:05 #297 №13256 
А сколько лет вы уже учите математику?
Аноним 19/03/17 Вск 13:26:38 #298 №13257 
>>13246
>Так полином это равно многочлен
Да.
Аноним 19/03/17 Вск 14:29:28 #299 №13259 
Selection018.png
Предлагаю решить задачу для 1 курса.
Аноним 19/03/17 Вск 15:34:46 #300 №13261 
>>13210
> сикп
> scheme
Кто-то не в теме.
Аноним 19/03/17 Вск 15:36:32 #301 №13262 
>>13259
Ну что ж, попытаюсь.
Компакт, конкретно здесь, в R - это замкнутое и ограниченное множество.
Пересечение произвольного набора замкнутых множест - замкнутое. (В силу правиладе моргана и топологического свойства, что произвольное объединение открытых - открыто.)
Значит прообраз _любого_ замкнутого множества - замкнут.
Значит прообраз _любого_ открытого множества - открыт.
Что является определением непрерывности f. Д-но.

Аноним 19/03/17 Вск 15:46:40 #302 №13264 
>>13262
> Значит прообраз _любого_ замкнутого множества - замкнут.
Здесь ошибка. Не значит.
Аноним 19/03/17 Вск 16:10:40 #303 №13265 
>>13261
Мы про это говорим?
https://en.wikipedia.org/wiki/Structure_and_Interpretation_of_Computer_Programs
Аноним 19/03/17 Вск 16:16:56 #304 №13266 
>>13265
Да. Книжка же не про схему.
Аноним 19/03/17 Вск 16:20:14 #305 №13267 
gif.latex.gif
>>13259
Как-то так примерно.
Аноним 19/03/17 Вск 16:20:30 #306 №13268 
>>13266
Ок, я просто так хуйню сказал, чтобы выебнуться, я был не прав.
Аноним 19/03/17 Вск 16:21:10 #307 №13269 
>>13267
Можно обычную картинку? У меня не видно ничего.
Аноним 19/03/17 Вск 16:23:33 #308 №13270 
s18.png
>>13269
Мог просто в отдельной вкладке пикчу открыть. Ну да ладно.
Аноним 19/03/17 Вск 16:23:42 #309 №13271 
>>13269
Открой в новой вкладке.
Аноним 19/03/17 Вск 16:31:32 #310 №13272 
>>13267
В стрелке на 3 строчке ошибка. Откуда следует, что предел существует? По твоему рассуждению получается, что для любой сходящейся последовательности {x_n} сходится {f(x_n)}.
Аноним 19/03/17 Вск 16:40:31 #311 №13273 
>>13272
Из того, что f(Kn) - это последовательность вложенных компактов, имеющая своим пересечением одну точку.
Аноним 19/03/17 Вск 17:27:03 #312 №13274 
1.png
Как доказать эту замечательную вещь, дорогой друг? Что-то я совсем в теоретические доказательства не умею, а сдаватьпоказывать это уже завтра!
Аноним 19/03/17 Вск 18:48:28 #313 №13278 
Я бьюсь с этими ебаными интегралами второй день и в итоге все равно нихуя не получается. Помогите вот хоть с чем-то из этого:
Найти площадь, ограниченную этими линиями

• y=7x-2x^2; x+y=7/2

Или то же задание, но функции такие:

• y=-x^2; y=2e^x; x=0; x=1 (тут у меня даже на графике ебучая фигура не получается, потому что -х^2 и 2е^х не пересекаются бляяяяять)
Аноним 19/03/17 Вск 18:59:31 #314 №13280 
>>13273
> f(Kn) - это последовательность вложенных компактов,
Откуда это следует?
Аноним 19/03/17 Вск 19:10:01 #315 №13281 
>>13273
Не, все, я пожалел, что тебе ответил. Одно дело, если бы ты по невнимательности ошибку допустил, но ты же вообще хуйню пишешь.
Аноним 19/03/17 Вск 19:30:12 #316 №13282 
>>13127
>Квантор
Прочитал как Кантор. Кантор всеобщности, ну вы понели.Рубрика: Математики шутят.
Аноним 19/03/17 Вск 19:33:21 #317 №13283 
>>13192
Разница, по нормально вродеп дополнение называется.
A={1,2,3,4,5}
B={3,4,5,6,7}
A/B={1,2} Типа как откусили кусок общий от A.
Аноним 19/03/17 Вск 19:48:56 #318 №13284 
>>13280
>for each compact set K, f(K) is compact
>>13281
Ну и катись со своей домашкой, мне-то что.
Аноним 19/03/17 Вск 20:00:53 #319 №13285 
>>13278
http://www.mathcentre.ac.uk/resources/uploaded/mc-ty-areas-2009-1.pdf
http://tutorial.math.lamar.edu/Classes/CalcI/AreaBetweenCurves.aspx
Аноним 19/03/17 Вск 20:13:16 #320 №13286 
>>13239
> Нет, почему? Придрочиться просто надо.
А пишешь ты прямо в живом латехе, каким-нибудь емаксом? Даже без всяких щадящих обёрток типа маркдаун + MathJax?
Аноним 19/03/17 Вск 20:13:25 #321 №13287 
1489943331.jpg
А вот такое знает кто? соре за качество, это не я, это камера такая
Аноним 19/03/17 Вск 20:15:26 #322 №13288 
>>13287
Аллах знает.
В показательную форму переведи
Аноним 19/03/17 Вск 20:17:37 #323 №13289 
>>13288
Какую форму?
Аноним 19/03/17 Вск 20:22:23 #324 №13290 
>>13286
В обычном текстовом редакторе пишу (sublime text), потом компилирую через консоль.
Аноним 19/03/17 Вск 20:22:40 #325 №13291 
>>13285
Спасибо большое
Аноним 19/03/17 Вск 20:23:27 #326 №13292 
20126443.jpg
>>13289
Да в показательную же.
Аноним 19/03/17 Вск 20:25:39 #327 №13293 
>>13284
> Ну и катись со своей домашкой, мне-то что.
Ну я-то уже решил, захотел поделиться интересной задачей. Если никто не решит, через 3 дня запощу решение.
Аноним 19/03/17 Вск 20:25:51 #328 №13294 
>>13292
Да это которая? Я мб и знаю, но я не на русском термины учил, не ебу чё за форма
Аноним 19/03/17 Вск 20:30:12 #329 №13295 
>>13293
Чего там решать-то, она элементарная.
Аноним 19/03/17 Вск 20:30:52 #330 №13296 
>>13294
Гугл ты тоже не на русском учил?
Аноним 19/03/17 Вск 20:37:40 #331 №13297 
>>13296
Все, нашел, спасибо. Ебаный пиздец, вместо комплексного с тригонометрией ебаться :(
Аноним 19/03/17 Вск 21:26:49 #332 №13299 
>>13238
А что это за правило? Оно тоже не находится нигде.
Аноним 19/03/17 Вск 21:30:21 #333 №13300 
>>13299
Тебе бы, прежде чем в непрерывные дроби лезть, с элементарной алгеброй разобраться. С тем, что такое деление, например.
Аноним 19/03/17 Вск 21:36:32 #334 №13301 
>>13300
Ну это из лекции "Математика для гуманитариев", там, вроде, всё просто должно быть.
Аноним 19/03/17 Вск 21:38:48 #335 №13302 
>>13301
Так и есть, это свойство в пятом классе проходят. Разделить на a/b - то же самое, что умножить на b/a.
Аноним 19/03/17 Вск 23:27:44 #336 №13304 
>>12775 (OP)
Двачематики/таполаги/гамолаги/модульщики над кольцами, каждый век в истории человечества был веком какого то математика.
Например ньютон довил в 17 веке, эйлер в 18, гаусс в 19, в 20 сложно кого то выбрать, так как оттуда некоторые до сих пор шевелятся.
Так вот, стоит ли читать высеры этих величайших математиков? Сейчас они имеют какую то познавательную ценность? Или васян из международного института по строению и изучению ящиков для песка может в сто раз лучше написать?
Аноним 20/03/17 Пнд 00:09:06 #337 №13306 
>>13304
>Сейчас они имеют какую то познавательную ценность?
Да
>Васютко из МИСИЯДП может в 100 раз лучше написать?
Да

Противопоставлять эти вопросы некорректно. 'Познавательная ценность' скорее историческая, ты можешь наблюдать за развитием науки и мотивировками, что подчас полезно для более глубокого понимания предмета. Использовать как учебники конечно же не стоит.
Аноним 20/03/17 Пнд 00:11:59 #338 №13307 
>>13306
Это я про архаическое говно мамонта, а вот 20 век сюда приплетать не стоит
Аноним 20/03/17 Пнд 04:19:14 #339 №13319 
p0027.png
p0028.png
>>13306
>Использовать как учебники конечно же не стоит.
На самом деле стоит. Тот же Фихтенгольц - пересказ Коши, только хуже.
http://rgho.st/8GXhPgjNz
Аноним 20/03/17 Пнд 11:37:33 #340 №13321 
>>13319
Притом у Коши часть теорем неверна. На свалку истории. Хотя, можно Харди навернуть, он по чистой матеше угорал.
Аноним 20/03/17 Пнд 11:46:01 #341 №13322 
>>13321
Примеры неверных теорем быстро, решительно.
Аноним 20/03/17 Пнд 13:00:34 #342 №13323 
>>13287
Переведи в синусы косинусы pi/3, может поможет.
Аноним 20/03/17 Пнд 13:01:31 #343 №13324 
>>13304
Риман после Гаусса.
Аноним 20/03/17 Пнд 13:36:34 #344 №13325 
Помогите, как делать дальше. Есть мн-во (a-b, -a+2b, 2a-b), нужно проверить полноту, как это сделать, если условие замкнутости выполняется?
Аноним 20/03/17 Пнд 15:55:02 #345 №13328 
>>13324
Чего так?
Аноним 20/03/17 Пнд 17:36:12 #346 №13332 
>>13304
Жан Реле! Потому что пучки придумал! ВСЁ ПУЧКОМ! ПУЧК ПУЧК
Аноним 20/03/17 Пнд 18:10:16 #347 №13333 
Добрый день.

Я не математик, я студент механик. Поэтому у меня есть некоторые сложности.

Я решаю задачи устойчивости систем. Не суть важно, что я там делаю, в механику посвящать не вижу смысла.


Суть решения сводится к получению некоторой системы уравнений. Эта система имеет бесконечно много решений. Решения я ищу, приравняв определитель матрицы уравнений к нулю.
Но мне нужен первый корень, то есть первая форма потери устройствости.

Функция root в маткаде и матлабе ищет корень на некотором промежутке. Но не ищет первый.
Короче, как всегда находить первый корень?
Желательно с примером.
20/03/17 Пнд 18:30:56 #348 №13334 
>>13333
Квадрипл, лол.
Внезапно, твой пост указывает на то, что все ЧЕСТЫИ МОТИРМОТИКИ угоряющие по АЛГУБРИОЧИСКАЙ ГОЕМЕТРИИ И ТЕПЛОГОИИ неправы, когда говорят, что математики не считают, и называя считающий людей инжинерами. Поскольку, инжинеры постоянно приходят к математикам для решение уравнений.
Математики считают, запомни это юнный ВРОТЕНДИК.
Аноним 20/03/17 Пнд 18:35:53 #349 №13335 
>>13334
Ничего не понял из твоего поста.

>>13333
Ах да. Корень то есть. Его легко найти с помощью графика. Но мне нужно автоматически их искать, чтоб график соорудить.
Аноним 20/03/17 Пнд 18:46:11 #350 №13336 
>>13333
Что значит "первый"? Наименьший?
20/03/17 Пнд 18:51:04 #351 №13337 
>>13335
>Ничего не понял из твоего поста.
Это просто наши идеологические разборки, не бери в голову.
Аноним 20/03/17 Пнд 20:00:09 #352 №13338 
>>13336
Первый попавшийся на глаза.
Аноним 20/03/17 Пнд 20:26:50 #353 №13339 
>>13336
Первый в смысле первый. Да, наименьший.
Аноним 20/03/17 Пнд 20:36:33 #354 №13341 
>>13333
Я не шарю в матлабе (дрочу на Mathematica), но, например, можно поискать нужную тебе функцию, которая найдет все корни. Например такая https://www.mathworks.com/help/symbolic/vpasolve.html
А потом отсортируешь корни
Аноним 20/03/17 Пнд 22:21:53 #355 №13343 
Как понять квадратные уравнения?
Аноним 20/03/17 Пнд 22:42:14 #356 №13345 
.png
>>13336
>>13333
Вот, короче. 2 график на картинке. Видите, что корней (D=0) несколько. Как ловить всегда самый маленький?

>>13343
А чё там понимать?


Аноним 20/03/17 Пнд 22:52:37 #357 №13346 
>>13345
Ну, я вот не понимаю откуда взялись все эти дискриминанты, 4ac и почему они вообще существуют и появились. И зачем нужны квадратные уравнения, собственно.
sage[mailto:sage] Аноним 20/03/17 Пнд 22:56:40 #358 №13348 
>>13346
почитай Гашков "квадратный трехчлен для умственно отсталых"
Аноним 20/03/17 Пнд 23:03:37 #359 №13350 
>>13345
Берешь первую производную.
В точках в которых эта производная превращается в ноль это точки минимума/максимума.
Берешь еще одну производную а точках в которой первая производная стала нулем, чтобы узнать это локальный или глобальный минимум/максимум и что это именно, минимум или максимум.

Задача уровня того же ящика.
Аноним 20/03/17 Пнд 23:17:45 #360 №13351 
Помогите декартово умножить две окружности. Не могу осилить. И как найти его геометрическую диагональ?
Аноним 20/03/17 Пнд 23:25:12 #361 №13352 
>>13346
http://lmgtfy.com/?q=%D0%B2%D1%8B%D0%B2%D0%BE%D0%B4+%D1%84%D0%BE%D1%80%D0%BC%D1%83%D0%BB%D1%8B+%D0%BA%D0%B2%D0%B0%D0%B4%D1%80%D0%B0%D1%82%D0%BD%D0%BE%D0%B3%D0%BE+%D1%83%D1%80%D0%B0%D0%B2%D0%BD%D0%B5%D0%BD%D0%B8%D1%8F
Аноним 20/03/17 Пнд 23:37:12 #362 №13353 
>>13345
Возьми значение, которое заведомо меньше, и начинай искать от него, постепенно сокращая отрезок.
Может, можно под твою задачу придумать что-то типа метода монте-карло, который выдаст самое низкоэнергетическое решение, но в общем случае - только так.
>>13351
Окружность с отрезком сначала перемножь, а потом подумай.
Аноним 20/03/17 Пнд 23:37:53 #363 №13354 
>>13350
Это всё понятно. Мне как численно ловить.
Аноним 21/03/17 Втр 00:16:53 #364 №13357 
>>13353
А численно как это делается?
Аноним 21/03/17 Втр 00:31:13 #365 №13358 
>>13353
>Окружность с отрезком сначала перемножь, а потом подумай.
Представил себе окружность в центре кординатной плоскости. Потом добавил ещё одну ось, которая представляет множество точек прямой. Умножил -- получил цилиндр. Подумал. Не получилось представить.
Аноним 21/03/17 Втр 00:31:49 #366 №13359 
>>13358
>кординатной
координатной*
Аноним 21/03/17 Втр 00:38:27 #367 №13360 
>>13357
Бери отрезок и запускай решатель. Нашёлся корень - уменьшаешь отрезок. Не нашёлся - сдвигаешь вперёд. И так до победного.
Аноним 21/03/17 Втр 00:38:51 #368 №13361 
>>13358
Как из отрезка получить окружность?
Аноним 21/03/17 Втр 00:41:40 #369 №13362 
>>13361
Предлагаешь скрутить цилиндр, который получился в результате умножения окружности на прямую, так, что получится тор?
Аноним 21/03/17 Втр 01:23:08 #370 №13363 
>>13360
Короче, для моей задачи я тупо построил график по точкам. Посчитал пару десятков значений критической силы, и построил график. Не знаю, примет ли препод, лол. Зависимость всё равно явно видна.

Аноним 21/03/17 Втр 13:25:40 #371 №13378 
КАК НАЙТИ СИНУСЫ/КОСИНУСЫ РАНДОМНЫХ УГЛОВ?
Вот мне нужно найти синус угла 17 градусов, как мне его расссчитать без рисования треугольника/единичной окружности и без таблиц с калькулятором? Есть какой то финт ушами позволяющий хотя бы первые правильные пять знаков после запятой получить?
Аноним 21/03/17 Втр 14:02:34 #372 №13379 
>>13378
Ряд Тейлора есть, но оно того не стоит. Гугл всяко лучше.
Аноним 21/03/17 Втр 15:14:00 #373 №13380 
>>13378
На руках проще всего через треугольник.
Аноним 21/03/17 Втр 15:24:39 #374 №13381 
>>12775 (OP)
Подскажите книги по терверу/матстату. Тервер читаю Ширяева сейчас, в теоретическом плане все устраивает, но слишком сухо что ли. Хотелось бы параллельно читать что-нибудь более наглядное, с интересными задачами и тп. По матстату вообще хз что читать. Можно на ангельском. Бэкграунд 3 курса мехмата.
Аноним 21/03/17 Втр 16:57:18 #375 №13383 
А можно математику надротить как собака Павлова? Вообще нет желания в этом разбираться учитывая на каком я уровне и какой нужен. А ведь нужно!
21/03/17 Втр 17:15:26 #376 №13384 
>>13383
Ты можешь надрочить только решение однотипных задач по однотимным алгоритмам. Так что нет.
А можно подробнее про уровни.
21/03/17 Втр 17:15:47 #377 №13385 
>>13384
>.
?
Аноним 21/03/17 Втр 22:20:09 #378 №13405 
x=1/(t-t^2)
y=1/(t-t^3)
Нужно построить график этой параметрической функции. В условии не сказано, чем принадлежит t.
Аноним 21/03/17 Втр 22:22:36 #379 №13406 
>>13405
Берешь маткад
@
Строишь
[mailto:1] Аноним 21/03/17 Втр 22:39:32 #380 №13407 
>>13383
В школе так и делают
Аноним 22/03/17 Срд 00:07:05 #381 №13419 
CodeCogsEqn.png
>>12775 (OP)
помогите пожалуйста, возможно ли исключить t из двух уравнений, задающих движение точки. Или во втором уравнении слишком много косинусов, синусов и корней и выразить из этой системы траекторию в явном виде невозможно?
Аноним 22/03/17 Срд 00:13:46 #382 №13420 
>>13419
Возможно. У тебя сорт оф треугольная система.
Аноним 22/03/17 Срд 00:32:15 #383 №13421 
>>13420
что значит треугольная система? да, там ошибка, перед корнем там должна быть дробь 7/8
Аноним 22/03/17 Срд 00:35:28 #384 №13422 
>>13421
Это значит, что тебе не нужно выражать t из второго уравнения, чтобы его исключить из системы.
Вырази косинус и синус через x и y.
Аноним 22/03/17 Срд 00:40:31 #385 №13423 
>>13422
я понимаю, но у меня так не получается со вторым уравнением
первое можно переписать в виде:
cos(3\pi t)=40x-1;
и если удастся из второго уравнения выразить синус, то можно оба уравнения возвести в квадрат и сложить,
но второе уравнение в таком виде переписать у меня не выходит, и я не знаю, возможно ли его в принципе так переписать.
Аноним 22/03/17 Срд 00:41:45 #386 №13424 
>>13423
Так подставь выражение для косинуса во второе уравнение, а потом спокойно вырази из него синус.
Аноним 22/03/17 Срд 03:34:56 #387 №13425 
14863305391260.png
>>12775 (OP)
Хочу познать азы геометрии за 7-9 класс, посоветуйте авторов или хороших книжек, и задачник к нему если можно. Обидно что нет в интернете таких кошерных учебников по геометрии как алгебра по Мордковичу.
Аноним 22/03/17 Срд 09:49:30 #388 №13427 
>>12775 (OP)
Дайте задачник какой то, в котором бы пояснялся ход решения и почему правильный ответ такой как указанно.
22/03/17 Срд 09:56:21 #389 №13428 
>>13427
Демидович + Полный решебник на китайском.
Аноним 22/03/17 Срд 10:10:41 #390 №13431 
>>13428
А что то другое есть?
Аноним 22/03/17 Срд 11:05:04 #391 №13433 
>>13431
Решебников мало, а вот задачников много. Просто китайцы прорешали ВСЕ здачи из Демидовича. Поэтому такой набор будет полным.
Аноним 22/03/17 Срд 13:42:01 #392 №13435 
>>13433
Но там же ответы на пиздоглазом. Как читать решебник то?
Аноним 22/03/17 Срд 14:01:36 #393 №13436 
>>13435
Там из контекста задачи будет понятна большая часть решений.
Аноним 22/03/17 Срд 15:12:39 #394 №13437 
>>12775 (OP)
Бля, почему в 1 томе Кострикина издания 2000 и 2004 года столько опечаток? В остальных томах такого нет надеюсь?
Аноним 22/03/17 Срд 17:28:37 #395 №13443 
>>13436
А что за антидемидович? В поиске рядом с китайским решебником встречается.
Аноним 22/03/17 Срд 20:24:35 #396 №13444 
>>13259
Никто не смог решить задачу для 1 курса, пощу свое решение. B(x, eps) означает открытый шар радиуса eps с центром в x (метрика стандартная).
Аноним 22/03/17 Срд 20:25:07 #397 №13445 
Selection019.png
>>13444
Аноним 22/03/17 Срд 20:46:49 #398 №13446 
>>13445
Хуйня. Гораздо интереснее, будет ли оно работать без первой аксиомы счётности.
Аноним 22/03/17 Срд 20:59:11 #399 №13447 
>>13446
Я думаю, тебе рано изучать пространства, в которых не выполняется первая аксиома счетности. Ты программу 1 курса не осилил.
Аноним 22/03/17 Срд 21:11:21 #400 №13448 
>>13447
А может, просто ты не осилил моё доказательство. Подумай об этом, сынок.
Аноним 22/03/17 Срд 22:04:51 #401 №13452 
1490209437.jpg
Нужно указать наклонную А1О на плоскость АВС. Помогите пожалуйста вообще не вдупляю как можно провести наклонную
Аноним 22/03/17 Срд 22:13:21 #402 №13454 
s19.png
>>13446
А хотя, в принципе, тоже не очень интересно. Но уже что-то.

Пусть A - множество всех функций из R в R.
Топологическое пространство X - это множество A с топологией, порождённой базой {Uh(f)} со всевозможными f∈A и h: R -> R>, принимающих строго положительные значения во всех точках, кроме может быть счётного их числа.
Топологическое пространство Y - это тоже A, но с дискретной топологией.
Отображение φ: X -> Y работает как тождественное на A.

Ясно, что непрерывным оно не будет. Проверим условия задачи. Второй пункт очевидно выполнен, осталось проверить, что компакты переходят в компакты. В Y компактами являются конечные множества и только они. Докажем, что то же самое справедливо и для X. Пусть S ⊆ X - какое-то бесконечное множество. Предъявим его покрытие открытыми множествами, из которого нельзя выбрать конечное подпокрытие.
1) Пусть найдётся точка x ∈ R, такая что πx(S) - бесконечное множество. Тогда возьмём функцию h равной нулю в x и единице во всех остальных точках R, и Uh(f), f∈S, будет искомым покрытием.
2) Пусть такой точки не найдётся. Тогда возьмём h(x) равным половине минимального расстояния между двумя различными точками πx(S).

Контрпример построен.
Аноним 22/03/17 Срд 22:34:21 #403 №13455 
14902094916430.jpg
>>13452
Аноним 22/03/17 Срд 22:36:54 #404 №13456 
>>13455
Спасибо, друг
Аноним 22/03/17 Срд 23:23:46 #405 №13457 
>>13456
Обращайся.
Аноним 22/03/17 Срд 23:54:43 #406 №13461 
CodeCogsEqn (1).png
>>13424
помогите пожалуйста доделать этот пример
после подстановки косинуса во второе уравнение и складывание у меня получается какое-то жуткое уравнение, которое равно отрицательному числу. Я правильно решаю или это уравнение траектории вообще, в принципе невозможно привести к нормальному виду? Или у меня ошибка? Проверьте пожалуйста.
Аноним 22/03/17 Срд 23:59:55 #407 №13462 
>>13461
Арифметику я проверять не буду, вольфрам проверит. Но в принципе что-то вроде этого и должно было получиться.
Аноним 23/03/17 Чтв 00:18:50 #408 №13465 
>>13462
ничего, что вот это уравнение равно отрицательному числу? Напиши пожалуйста, как в вольфраме проверить, что написать?
Аноним 23/03/17 Чтв 00:23:37 #409 №13466 
>>13465
Понятия не имею, зачем ты перенёс это число направо, но нет, ничего. То, что слева, может принимать и отрицательные значения.
Просто набираешь (выражение_для_косинуса)^2+(выражение_для_синуса)^2-1, и всё.
Аноним 23/03/17 Чтв 00:35:43 #410 №13467 
>>13466
я просто знаю, что точка движется по эллипсу, и поэтому спрашиваю, ну тип это же радиус в правой части, и он не может быть отрицательным. Я не понимаю, как записать в вольфраме эти синусы и косинусы, объясни подробней пожалуйста и ещё, как построить в вольфраме график этого выражения, которое у меня получилось?
Аноним 23/03/17 Чтв 00:41:20 #411 №13468 
>>13467
Это было бы эллипсом, если бы не было корня. А так это чёрт знает что.
Аноним 23/03/17 Чтв 00:46:11 #412 №13469 
>>13468
ну ладно, что там с вольфрамом, можно это уравнение нарисовать в вольфраме?
Аноним 23/03/17 Чтв 00:49:07 #413 №13470 
>>13469
https://www.wolframalpha.com/examples/
Аноним 23/03/17 Чтв 01:14:59 #414 №13471 
Snap 2017-03-23 at 01.14.24.png
>>13470
ну вот что получилось, что это значит?
Аноним 23/03/17 Чтв 01:36:53 #415 №13475 
>>13471
К нулю приравняй.
Аноним 23/03/17 Чтв 02:07:55 #416 №13476 
Snap 2017-03-23 at 02.05.21.png
>>13475
спасибо. Вот, получился эллипс. А можно в вольфраме как-нибудь это выражение упростить, чтобы по виду уравнения сразу было видно, что это эллипс, или же привести к виду y=f(x)? Или к более простому виду это выражение не привести?
Аноним 23/03/17 Чтв 03:35:15 #417 №13478 
>>13476
Если оно похоже на эллипс - это ещё не значит, что это эллипс. Избавься от корня, после чего перенеси всё влево и попробуй заставить вольфрам это разложить на множители.
Аноним 23/03/17 Чтв 14:48:16 #418 №13486 
>>13476
>или же привести к виду y=f(x)?
Эта запись подразумевает, что для любого x, существует единственный y, которые связаны функцией f. Уже на примере окружности видно, что нет такой f, ибо каждому x удовлетворяют два значения.
Аноним 23/03/17 Чтв 18:47:54 #419 №13493 
Ребят, на лекциях по матану рассказывали про всякие формулы тейлора, формы(формулы(???)) лагранжа и доп. члены. Че за хуйня? Я моргнул и потерялся, нахуй это нужно, как этим пользоваться? перед заходом в гугл решил написать тут, может кто-то чего объяснит, хотя бы идейно, че да как.
Аноним 23/03/17 Чтв 20:32:07 #420 №13494 
>>13493
Любая функция хоть немножечко локально полином. Разность функции и ее немножечко полинома -остаточный член. Его можно выразить по-разному: через интеграл, через производную и т.д. Вот одна из форм (остаточного члена) и называется формой лагранжа
Аноним 23/03/17 Чтв 20:49:44 #421 №13495 
Snap 2017-03-23 at 20.45.54.png
Snap 2017-03-23 at 20.46.38.png
>>13478
зачем раскладывать на множители?
Ранее неправильно была набрана формула, вот траектория. И там внизу wolfram написал solutions, это и есть упрощённое выражение , выражающее y через х?
Аноним 23/03/17 Чтв 20:56:03 #422 №13496 
Selection022.png
>>13493
Пусть f дифференцируема в точке x. Тогда
f(x + dx) = f(x) + f'(x)dx + o(dx).
Естественно задаться таким вопросом: можно ли с помощью производных получить более хорошее приближение? Например, найти a2 такое, что
f(x + dx) = f(x) + f'(x)dx + a2 (dx)2 + o(dx^2 ).
Может, можно бесконечно раскладывать? Продолжение мотивировки на пикрелейтед.
Аноним 23/03/17 Чтв 21:00:12 #423 №13497 
>>13496
Что касается доказательств формулы Тейлора, то они все уродливые, тут ничего не поделаешь.
Аноним 23/03/17 Чтв 21:20:20 #424 №13498 
>>13497
Нонсенс.
Аноним 23/03/17 Чтв 22:03:22 #425 №13499 
>>13498
Ну покажи мне неуродливое. Вот самое нормальное, которое я знаю:
http://math.stackexchange.com/questions/481661/simplest-proof-of-taylors-theorem
Ну кароч возьмём простое число например 1488 23/03/17 Чтв 23:22:15 #426 №13502 
>>12775 (OP)
>Список от анона с dxdy. Довольно внушителен, тоже рекомендуется к прочтению. Является дополнением к списку старго ОП-а.
http://pastebin.com/YP1uaUyd
Что из этого осваивать если нужны знания на уровне где-то между олимпиадой физтех-ОММО и вышка-всеросс? Может что-то, что зайдёт вместе с курсами фоксфорда (ну там на бесплатные курсы ещё заданий нет, поэтому я хуй знает что решать, кроме олимп прошлых лет и Шабунина, но этот задачник для подготовки чисто к физкеку, а это уже не актуально)? Там просто не рационально сразу все книги одновременно читать, а летом-то можно будет освоить и оставшееся да.
Аноним 23/03/17 Чтв 23:37:59 #427 №13503 
>>13502
не знаю что там в списке и вообще долбоеб-неолимпиадник, но думаю будет полезно поучить что-то связанное с линалом, мат. логикой. А ещё что-нибудь типа "школьной математики с точки зрения высшей"
вообще, после семестра в среднем вузе на примате (читали курсы по дискретной математике (комбинаторика, мат. логика и немного графов было), мат. анализу, лин. ал), смог решить 4 или 5 заданий из олимпиады вышки по математике. понятно, что в вузе далеко выходящих за понимание школьника знаний мне не дали.
Аноним 24/03/17 Птн 01:06:31 #428 №13505 
>>13502
Список тот к олимпиадам имеет косвенное отношение. И вообще он недоделан, рандомен и преисполнен картофаном, лучше его нахуй убрать.

>знания на уровне где-то между олимпиадой физтех-ОММО и вышка-всеросс
Скилл тогда уж, а не знания, знаний для олимпиадок достаточно в объеме чуть превышающем школьную программу.
Если хочешь готовить к олимпиаде, то самое актуальное - это задачники Прасолова, почти все олимпиады (особенно мск) - это рандомный сет из этих задачников. Как правило, хотябы одна треть будет взята оттуда.
Другое дело, что и задач там овердохуя, все ты не прорешаешь.
>фоксфорд
и прочая поебота - это бесполезное наебало для выкручивания денег из доверчивых школяров

Олимпиадный фольклор со временем сильно меняется, если хочешь не просто тренировать ум, а чтоб задачки были наиболее близки к тем, что могут попасться, можешь еще порешать
>Элементы математики в задача: Через олимпиады и кружки к профессии
>Математика в задачах. Сборник материалов выездных школ

Бери что-нибудь из этого и дрочи все лето по 2 Grot в день.
Аноним 24/03/17 Птн 09:16:33 #429 №13507 
>>12775 (OP)
сап матемач.
чем отличается математическая статистика от статистики финансов ?
статистику финансов знаю, там дисперсия, корреляция, регрессия, мода, анализы и тд.
в мат.статистики в принципе тоже самое ведь да ?
Аноним 24/03/17 Птн 09:47:44 #430 №13508 
Поясните, пожалуйста, за такую вещь, как HoTT - гомотипическая теория типов. В частности, закроет ли она вопросы об основания математики?
Аноним 24/03/17 Птн 11:17:14 #431 №13509 
изображение.png
>>13502
>Теория чисел (для школьников):
> Курс арифметики - Жан Пьер Серр
Это тролинг такой?
[mailto:1] Аноним 24/03/17 Птн 12:08:27 #432 №13510 
>>13507
Очевидно, что статистика финансов - приложение мат.статистики к финансам.
Аноним 24/03/17 Птн 16:15:23 #433 №13511 
Кто здесь получил 37 на олимпиадке Эйлера?
Аноним 24/03/17 Птн 19:33:37 #434 №13514 
>>12775 (OP)
Дайте простеньких книжек для вкатывания в матан, линал, теорвер, функан, комплан
Аноним 24/03/17 Птн 21:20:23 #435 №13517 
>>13514
Глянь списки литературы в шапке, там всё есть.
Аноним 24/03/17 Птн 22:03:59 #436 №13518 
>>13508
Тебе в основания математики тред.
Аноним 24/03/17 Птн 22:23:09 #437 №13519 
>>13517
Есть, но там как только открываешь книжку на тебя гамалогеи с тапалогеямы вываливаются.
Мне для первого ознакомления, с прицелом на том что я нихуя не знаю в данных областях.
Аноним 24/03/17 Птн 22:26:43 #438 №13520 
>>13519
Ты всё сразу захотел? Без них никак. Учишь множества, категории, алгебру, топологию. Потом читаешь остальное.
24/03/17 Птн 22:37:45 #439 №13521 
>>13520
Смотри, для начала можешь просмотреть Куранта Дифференциально и интегрально исчисление том 1. Параллельно учишь множества и читаешь Констрикина введение в алгебру. Всего Куранта читать не обязательно, прочти 300 страниц, их довольно просто читать, там много воды, чтобы получить интуицию и идёшь читать Зорича.
Аноним 24/03/17 Птн 22:50:49 #440 №13522 
2017-03-2422h4332.png
Аноны, гляньте оглавление книги по планиметрии, хочу её прочесть вместо того, что дано в списке (Понарин: “Элементарная геометрия” 1 том). В Понарине просто не все понятно, какие-то термины забыты даже. Можно в общем заменить её или лучше взять из списка, и гуглить термины по ходу дела?
Аноним 24/03/17 Птн 22:54:28 #441 №13523 
>>13522
А что за книга? Название скажи, полистаю.
Аноним 24/03/17 Птн 23:03:57 #442 №13524 
>>13523
Бутузов Кадомцев 2005г. Планиметрия
Аноним 24/03/17 Птн 23:10:13 #443 №13525 
>>13520
Не слушай этого поехавшего, можно прекрасно и без этого поначалу обходиться.
Аноним 24/03/17 Птн 23:10:24 #444 №13526 
>>13520
А как диды учили без всего этого? или то не математика была?
Аноним 24/03/17 Птн 23:47:19 #445 №13528 
>>13526
У местных пучкистов то что не самое модное не математика.
Аноним 25/03/17 Суб 00:17:15 #446 №13529 
>>13525
Ну вот я и прошу поначалу. Я же писал, когда открываю книжки из оппоста меня тапалогеями гомеоморфно выворачивают на изнанку.
Аноним 25/03/17 Суб 09:25:22 #447 №13535 
>>13525
Анализ... без топологии... топология... без множеств и категорий... линейная алгебра... без полей, колец, групп...

>>13529
Ну так выучи, что такое тапалогии, и всё.
Аноним 25/03/17 Суб 10:36:00 #448 №13538 
>>13535
Вот это ты жирдяй...
Аноним 25/03/17 Суб 11:39:21 #449 №13539 
>>13425
bump
Аноним 25/03/17 Суб 11:53:29 #450 №13540 
изображение.png
изображение.png
>>13538
Я серьезно.
>>13526
Сравни пик1 и пик2.
Аноним 25/03/17 Суб 12:12:27 #451 №13541 
>>13540
>Я серьезно.
>Анализ... без топологии... топология... без множеств и категорий... линейная алгебра... без полей, колец, групп...
Аноним 25/03/17 Суб 12:14:26 #452 №13542 
>>13540
Но он только начинает учить. Ему ни то, ни другое сейчас не надо. Что ему надо, так это интуитивное понимание для начала. Ты же понимаешь, что подавляющему большинству людей от того, что у тебя на скринах становится скучно и они просто дропают обучение.
Аноним 25/03/17 Суб 12:34:21 #453 №13543 
>>13542
Если ему от этого скучно, то зачем он пытаются учить? Это бесполезно.
>>13541
Как ты выучишь топологию без знания множеств и линал без знаний по алгебре? НИКАК! Там много не надо, конкретно по множествам: множества, действия над ними(объединение, пересечение, разность, декартово произведение), отображения множеств, отношение эквивалентности, фактор-множества, упорядоченные множества(частино-, вполне-...).
Аноним 25/03/17 Суб 13:01:41 #454 №13544 
>>13543
>Если ему от этого скучно, то зачем он пытаются учить? Это бесполезно.
Есть много простых, интересных и полезных книг по математике. Нахера вы всем подряд своей ссаной теорией множеств тычете? Начинающий может отлично и без нее обходиться. А когда поймет, что дальше без нее нельзя продвигаться, то вот тогда и будет о ней читать.
Аноним 25/03/17 Суб 13:15:18 #455 №13546 
Безымянный2.png
Может не по тематике, не знаю. Делаю преобразование лапласа в маткаде, вместо 8Y(s) он вычисляет как 8*laplase(говно), как пофиксить и сделать как на 2 примере?
Аноним 25/03/17 Суб 13:40:39 #456 №13548 
>>13544
>Начинающий может отлично и без нее обходиться.
>Дайте простеньких книжек для вкатывания в матан, линал, теорвер, функан, комплан
Чтобы вкатиться туда, куда он хочет, не может. Да и что в ней сложного, блядь? За день-два можно освоить необходимый минимум.
Аноним 25/03/17 Суб 13:55:47 #457 №13549 
>>13514
Ильин-Позняк есть по матану и, по-моему, по линалу. Для начала норм.
Для теорвера нужен матанализ, для функана матанализ и линал, для комлана нужен матанализ.
Вернешься когда выучишь матан и линал
Аноним 25/03/17 Суб 14:05:00 #458 №13550 
>>13549
>Ильин-Позняк
Убей себя.

>>13514
Зорич, Кострикин-Манин, Мостеллер-Рурке-Томас, Колмогоров-Фомин, Шабат
Аноним 25/03/17 Суб 14:13:11 #459 №13551 
>>13549
>>13550
Бля, я тупо скопировал не посмотрев.
Матан я знаю уже, по двум книжкам лузина.
Аноним 25/03/17 Суб 14:38:05 #460 №13552 
dS1iRlJhWw.jpg
7YK3DKpRXY.jpg
>>12775 (OP)
Блять, помогите дауну, пожалуйста.
Не могу решить эту парашу.
Разложил, а дальше как действовать?
Аноним 25/03/17 Суб 14:46:09 #461 №13554 
>>13552
Два яблока минус одно яблоко.
Аноним 25/03/17 Суб 14:47:09 #462 №13556 
>>13554
Ответ один?
Аноним 25/03/17 Суб 14:47:57 #463 №13558 
>>13556
да
Аноним 25/03/17 Суб 14:50:03 #464 №13559 
>>13558
Боже, спасибо.
Я сидел минут тридцать над ним.
Аноним 25/03/17 Суб 14:52:37 #465 №13560 
>>13550
>Зорич
Убей себя
Аноним 25/03/17 Суб 14:53:21 #466 №13561 
>>13552
1
Аноним 25/03/17 Суб 14:55:19 #467 №13562 
>>13560
Проблемы? Лучший учебник матана из существующих на русском языке.
Аноним 25/03/17 Суб 14:56:29 #468 №13563 
>>13562
Нет. Лоран Шварц, Львовский будут в 100 раз лучше.
Аноним 25/03/17 Суб 14:59:43 #469 №13564 
asdasd.png
А тут как?
Аноним 25/03/17 Суб 15:02:38 #470 №13565 
>>13564
никак, избавься от иррациональности в знаменателе
Аноним 25/03/17 Суб 16:08:52 #471 №13568 
Пацаны, я просто так спрошу, на всякий случай
Определения типа: A - это B. Они же работают в две стороны, т.е. A <=> B, так?.
Аноним 25/03/17 Суб 16:24:00 #472 №13570 
>>13568
Определения - да.
Аноним 25/03/17 Суб 16:36:48 #473 №13571 
>>13570
спасибо
Ёбаный стыд Аноним 25/03/17 Суб 18:03:08 #474 №13578 
ellips.png
Запрос в Гугле "длина эллипса", первый результат.

http://formylu.ru/geometriya/formuly-perimetra/dlina-ellipsa
Аноним 25/03/17 Суб 18:07:54 #475 №13579 
Сижу и думаю, откуда эта формула в голове у автора взялась.
Понятно, что проскользнула в мыслях какая-то аналогия с формулой длины окружности, но какая именно аналогия? Как это вообще можно написать в здравом уме?
Аноним 25/03/17 Суб 18:10:50 #476 №13580 
Тут где-то кидали две книги: одна вроде бы учила думать, а другая решать. Как-то там how to prove it или как там.
Аноним 25/03/17 Суб 18:16:08 #477 №13581 
>>13578
Ну, как простенькая аппроксимация, она сойдет.
Аноним 25/03/17 Суб 18:25:39 #478 №13582 
Clip2net170325172518.png
>>13581
Во, смотри.
Аноним 25/03/17 Суб 18:29:45 #479 №13583 
>>13580
Эти?
https://en.wikipedia.org/wiki/How_to_Solve_It
https://www.amazon.com/How-Prove-Structured-Approach-2nd/dp/0521675995
Аноним 25/03/17 Суб 18:29:55 #480 №13584 
>>13559
Проиграл. Вспомнил что-то мем из тесача, где там пепельные упыри стилизованные под "мемчики)0))" из 2013, и типо написано "искал двемерскую головоломку месяц)))00)))"
Аноним 25/03/17 Суб 18:31:08 #481 №13585 
>>13584
классика, сам такого персонажа знаю, который месяц наиграл в морку не найдя головоломку
Аноним 25/03/17 Суб 18:36:41 #482 №13586 
Clip2net170325173601.png
>>13582
Только там по оси абсцисс не b, а b/2, если что. Я упоролся. Вот тут я пофиксил вроде бы. Давно с таблицами не работал, сорри.
Аноним 25/03/17 Суб 19:13:31 #483 №13587 
Это сильно хуёво писать для относительного дополнения множеств X - Y вместо X \ Y ?
Аноним 25/03/17 Суб 19:41:23 #484 №13588 
И все таки, есть книжки которые поясняют за математику без теории множеств?
Аноним 25/03/17 Суб 19:46:30 #485 №13589 
>>13588
совсем без? или с минимумом? если второе, то любые книжки для первачей
Аноним 25/03/17 Суб 20:12:12 #486 №13590 
>>13587
Да пиши как хочешь, единственное, что важно — понимают тебя или нет.
Аноним 25/03/17 Суб 20:34:12 #487 №13592 
blob
Вот откуда должна появится геометрическая интуиция?
Аноним 25/03/17 Суб 20:39:31 #488 №13593 
>>13592
Картинка? При чем тут картинка? Хотел спросить о математической индукции, но в результате стало совсем стыдно и... ДА Я НИ ЧЕРТА НЕ ПОНИМАЮ, у меня явная истерика. Какие-то аналогии непонятные, принцип домино понятный. Но доказательства математической индукции... домино мы взяли сколько надо изначально, установили и уже не манипулируем, не представляем домино на боку или еще что-то такое. А в математике, похоже это везде, какие-то символы не пойми откуда и по каким законам мы можем их взять, почему их вообще можно взять ни откуда и ПРОСТО ОЧЕВИДНО связать. У меня реакция первобытной обезьяны. В особенности из-за того, что это задачка для 8 класса. Все понимают, а я какой-то ущербный, пожалейте меня. Слезы просто.
Аноним 25/03/17 Суб 20:54:46 #489 №13594 
>>13589
Давай эти самые книжки/авторов пищущих такие книжки.

Ну почти любую математическую конструкцию можно натянуть на теорию множеств, даже определение числа 1, даже аллаха как элемент множества выдуманных персонажей, так что совсем без множеств даже на пальцах посчитать не получится. Дети спокойно считают в свои считалочки и без множеств, я о том что количество пальцев на рухак можно вполне легально втиснуть в множества из за их "резиновости", как например множество гвоздей в коробке
Без языка теории множеств....как бы это сказать так чтобы понятно было...вот чтобы описывали конструкции, например бесконечно малое/большое не многоходовочкой по множествам и подмножествам с расслоениями на пучках сечений модульных, афинно зависимых гиперпространственных проекций на гиперплоскость с неэвклидовой геометриией, а как последовательность чисел которая меньше наперед заданного числа, находящаяся между нулем и заданным числом/последовательности чисел которые больше заданного числа, и чтобы потом было подробное описание логической конструкции описывающей почему именно они бесконечные.

Не понятно наверное нихуя. Ну вот в множествах считают что ты уже знаешь определения и теоремы которые тебе потом будут подавать в виде множеств. А я их не знаю. И мне их нужно выучить не участвуя в множествах.
Аноним 25/03/17 Суб 20:59:01 #490 №13595 
>>13593
По мат.индукции анон в /b/ пояснял что если условие выполняется для двух случаев (например n и n+1), то смотрим выполняется ли данное условие к разности случаев, и если да, то всё нормально
>Слезы просто
ты тян? (ну просто тянки так обычно пишут) асечку писечку плз
Аноним 25/03/17 Суб 21:09:22 #491 №13596 
>>13594
любые книжки по матанализу и такому для нематематиков-первокурсников
Аноним 26/03/17 Вск 01:22:49 #492 №13602 
>>13592
>>13593
Но так принцип математической индукции позволяет только проверить, что гипотеза верна.
Из одной индукции ты не выведешь сумму
1^2 + 2^2 + ... n^2. Приходится использовать ту или иную интуицию хотя бы для того, чтобы гипотезу придумать.
Аноним 26/03/17 Вск 11:58:20 #493 №13608 
>>13593
Есть множество Х.
Если x_n обладает каким-то свойством и из этого следует, что x_(n+1) обладает таким же свойством, то все элементы в Х начиная с x_n обладают таким свойством.
Про домино.
Выстроили в ряд бесконечное кол-во доминошек. Если N-ая доминошка упадет, то(из этого следует что) упадет и следующая за ней N+1-ая доминошка. Т.к. первая доминошка упадет, то упадут и все за ней.
Аноним 26/03/17 Вск 12:52:54 #494 №13610 
>>12775 (OP)
>гендерного равенства тред
>на пиках одни девушки
А вообще я хотел задать вопрос.
Что из математики нужно изучить после школьного курса чтобы понимать более 3/4 части всех документов arxiv.org?
Аноним 26/03/17 Вск 12:53:45 #495 №13611 
>>13610
http://imperium.lenin.ru/~verbit/MATH/programma.html
3 курса
Аноним 26/03/17 Вск 15:30:43 #496 №13616 
>>13611
Просто иди отсюда в свой загон для шизиков.
Аноним 26/03/17 Вск 15:40:26 #497 №13617 
>>13596
Мне ходить по всем шарагам и спрашивать что там на первом курсе советуют прочитать?
Аноним 26/03/17 Вск 15:41:14 #498 №13618 
>>13610
>понимать более 3/4 части всех документов arxiv.org
Хуясе у тебя запросы. Может тебе еще царский путь в тапалогею нужен?
Аноним 26/03/17 Вск 16:01:11 #499 №13619 
>>13618
top-book же
Аноним 26/03/17 Вск 16:55:32 #500 №13620 
уравнение

4*tg^2(x) + 3/cos^2(x) + 3 = 0
получается
cos(2x) = 4

где я мог ошибиться?
Аноним 26/03/17 Вск 17:57:10 #501 №13622 
>>13619
Для неё нужно немного алгебру знать. В первых же задачах используется билинейная симметричная форма.
Аноним 26/03/17 Вск 18:26:25 #502 №13623 
Снимок.JPG
Здарова, парни такая проблема чему равен этот предел я получил 3, но в ответах он равен бесконечности
Аноним 26/03/17 Вск 18:27:10 #503 №13624 
>>13623
Нет, он равен 3.
Аноним 26/03/17 Вск 18:34:09 #504 №13626 
>>13624
ошибся там у x который рядом с тройкой 3 степень
Аноним 26/03/17 Вск 18:34:25 #505 №13627 
>>13623
ошибся там у x который рядом с тройкой 3 степень
Аноним 26/03/17 Вск 19:56:47 #506 №13628 
>>13626
>>13627
Так в 2-ой степени или 3-ей? При 3x^3 да, равен бесконечности.
Аноним 26/03/17 Вск 20:26:13 #507 №13629 
heeeeeeeelo.png
Помогите пожалуйста, нужны точные ответы на эти 3 вопроса
только в них вообще не уверен, никак не могу понять
вроде решается все устно
1)2 вообще хз
2)2
3)4?
Аноним 26/03/17 Вск 21:10:30 #508 №13630 
>>13616
Куда именно? Не понял.
 Аноним 26/03/17 Вск 22:18:52 #509 №13631 
>>13629
Определения читай, василий.
Аноним 26/03/17 Вск 22:32:06 #510 №13632 
Добрый день. Вкатился в Зорича. Такой вопрос.Отношение R - подмножество декартового квадрата. Если оно антисимметрично, то следует ли из этого, что оно -- подмножество диагонали этого декартового квадрата?
Аноним 26/03/17 Вск 22:39:23 #511 №13633 
>>13623
Если третья степень, то бесконечность (делим числитель и знаменатель на x3) это случаем не домашка с фоксфорда?
Аноним 26/03/17 Вск 22:44:04 #512 №13634 
>>13632
Нет.
Аноним 26/03/17 Вск 22:52:32 #513 №13635 
>>13634
Ладно. Пересечение этого R с транспонированным R будет подмножеством диагонали того самого декартового квадрата?
Аноним 26/03/17 Вск 23:03:20 #514 №13636 
>>12775 (OP)
Аноним 26/03/17 Вск 23:05:32 #515 №13637 
67849867564.png
>>12775 (OP)
выручайте, аноны
Аноним 26/03/17 Вск 23:15:15 #516 №13638 
>>13635
Да.
Аноним 26/03/17 Вск 23:18:01 #517 №13639 
>>13638
Спасибо.
Аноним 26/03/17 Вск 23:24:47 #518 №13640 
>>13638
Ну и ещё вопрос. Антисимметричное отношение содержит диагональ множества, на котором оно определено?
Аноним 26/03/17 Вск 23:26:18 #519 №13641 
>>13640
Не обязательно. Хотя самый распространённый пример - это отношение порядка, а оно содержит.
Аноним 27/03/17 Пнд 00:47:51 #520 №13644 
>>13637
Не вижу тапалогий! Ничем помочь не могу! Удачи!

С уважением, звук с 1:34 по 1:36 в песне Chicane – Come Tomorrow
Аноним 27/03/17 Пнд 01:15:09 #521 №13645 
>>13637
В последней задаче некорректная постановка. Вторая тривиальна. В первой тангенс половинного угла.
Аноним 27/03/17 Пнд 06:07:36 #522 №13646 
>>13631
Да определения то я знаю
Аноним 27/03/17 Пнд 09:31:52 #523 №13647 
>>13588
Юамп
Аноним 27/03/17 Пнд 10:04:29 #524 №13648 
>>13588
Разве что те, которые вышли в 19-м веке, или на крайняк в начале 20-го. Множества давно и прочно стали частью естественного математического языка. Требовать математику без теории множеств - это примерно как требовать художественных текстов без буквы "а", на том основании, что тебе сложно её писать. Нелепица полнейшая. Наоборот, матшкольников нынче стараются научить множествам как можно раньше, чтобы они у них не вызывали суеверного ужаса и не мешали интуиции, когда дело дойдёт до действительно сложных конструкций.
Аноним 27/03/17 Пнд 10:39:07 #525 №13649 
>>13648
>без буквы "а"
можно по японски писать.
Аноним 27/03/17 Пнд 11:41:12 #526 №13650 
>>13648
>Начиная со второй половины XX века представление о значении теории и её влияние на развитие математики заметно снизились за счёт осознания возможности получения достаточно общих результатов во многих областях математики и без явного использования её аппарата, в частности, с использованием теоретико-категорного инструментария (средствами которого в теории топосов обобщены практически все варианты теории множеств).
УРЕТЕ, ВАШИ ТОПАСЫ НЕ ТОПАСЫ, ПРУФЫ НЕ ПРУФЫ, А ТО ШТО КАЖДЫЕ ПОЛГОДА НОВУЮ ТЕОРИЮ МНОЖЕСТВ ПРИДЖУМЫВАЮТ ТАК ЭТО ФИЧА А НЕ БАГ
Аноним 27/03/17 Пнд 12:05:29 #527 №13651 
>>13650
Речь не об аксиоматической теории множеств, ебло. Когда вы уже научитесь разделять формализм и содержание? Я говорю о множестве как понятии. Современная математика без него немыслима. Категории - это средство описания, не более того. Все осязаемые примеры категорий строятся из множеств. Собственно, без понятия класса ты категорию даже не определишь толком.
Аноним 27/03/17 Пнд 12:17:13 #528 №13652 
>>13588
https://www.youtube.com/watch?v=PI5ipmVL-Kw
Аноним 27/03/17 Пнд 12:17:53 #529 №13653 
>>13652
Разве что что-то подобное, да.
Аноним 27/03/17 Пнд 12:55:55 #530 №13654 
>>13617
ильин-позняк
Аноним 27/03/17 Пнд 13:12:31 #531 №13655 
>>13651
А числа это вообще абстракция и их не существует. Дальше что?

>>13652
А еще толще можешь? Скажи аксиомы пеано не определяют N
Аноним 27/03/17 Пнд 13:15:18 #532 №13656 
>>13654
А векторный анализ?
Аноним 27/03/17 Пнд 13:17:33 #533 №13657 
>>13655
Вполне себе существуют.
>аксиомы пеано не определяют N
А это уже смотря как их понимать.
Аноним 27/03/17 Пнд 13:20:14 #534 №13658 
>>13656
Векторный анализ - это всякие внешние дифференциалы роторы и дивергенции. Оно тебе пока не надо.
Аноним 27/03/17 Пнд 13:30:13 #535 №13659 
>>13658
Надо.
Аноним 27/03/17 Пнд 13:49:18 #536 №13660 
>>13659
https://www.amazon.com/Div-Grad-Curl-All-That/dp/0393925161
Аноним 27/03/17 Пнд 13:49:22 #537 №13661 
>>13659
Тогда второй том Зорича к твоим услугам.
Аноним 27/03/17 Пнд 14:41:44 #538 №13662 
>>13651
Осязаемость зависит от привычки. Я вот в последнее время стал сомневаться в существовании некоторых эзотерических конструкций вроде large cardinals, потому что не могу придумать для них категорного обоснуя.

Ну и категория прекрасно определяется чисто категорным языком без всяких классов, конечно.
Аноним 27/03/17 Пнд 14:50:33 #539 №13663 
>>13662
>не могу придумать для них категорного обоснуя
А зачем их придумывать?
Аноним 27/03/17 Пнд 14:56:24 #540 №13664 
>>13663
Для меня реально то, что имеет категорный обоснуй.
Аноним 27/03/17 Пнд 15:00:49 #541 №13665 
>>13662
Как бы ты ни определял категорию, это всё равно сущность, которая содержит другие сущности, в некотором смысле. От того, что ты не произносишь слова "класс", он отсюда никуда не девается. В теории категорий, самой по себе, ещё ниоткуда не следует, что хоть одна категория вообще есть на свете, и не существует способов конструирования новых категорий из существующих. И даже если какая-то категория дана тебе свыше самим Гротендиком, у тебя нет других способов выяснить, есть ли в ней какой-нибудь классификатор подобъектов или там декартовы квадраты, кроме как ковырять её теоретико-множественную реализацию. Этот язык слишком общий, чтобы говорить о конкретике каким-то замкнутым способом.
Аноним 27/03/17 Пнд 15:08:10 #542 №13666 
Поясните за умножение по Рыбникову.
Аноним 27/03/17 Пнд 15:09:32 #543 №13667 
>>13666
Тупое говно тупого говна. Говно. А на сегодня всё. До новых встреч.
Аноним 27/03/17 Пнд 15:14:30 #544 №13668 
>>13666
>умножение по Рыбникову
пиздец какой
Аноним 27/03/17 Пнд 15:40:53 #545 №13669 
>>13667
Почему тогда звучит так убедительно?
Аноним 27/03/17 Пнд 15:44:46 #546 №13670 
>>13669
Потому что для людей, не знающих, что такое умножение, даже собака будет звучать убедительно.
Аноним 27/03/17 Пнд 16:12:28 #547 №13671 
>>13665
А откуда следует, что множества есть? К ним все привыкли и они кажутся настолько убедительными, что вопрос об их существовании не ставится. Но ведь их существование, по сути, дело личной веры математика. Для меня ETCC и ETCS не менее убедительны, чем ZFC.
Аноним 27/03/17 Пнд 16:35:31 #548 №13672 
>>13671
А коран не менее убедителен комиксов про карателя?
Аноним 27/03/17 Пнд 16:36:33 #549 №13673 
>>12775 (OP)
Это уже не смешно. Я биты три треда спрашиваю один и тот же вопрос и ни один таполаг с гамолагом не ответил.
Как вывести формулы интегралов без привлечения фокуса с обратными производными?
Аноним 27/03/17 Пнд 16:38:29 #550 №13674 
>>13672
Давай серьёзно.
Аноним 27/03/17 Пнд 16:59:41 #551 №13675 
>>13671
Есть пустое множество, и есть небольшой набор естественных операций. ZF их просто конкретизирует. Из этого строится всё что нужно. Я не сомневаюсь, что всё то же самое можно закодировать и на чисто категорном языке, но говорить, что здесь нет множеств и они не нужны - это лукавство, поиск лазеек в формулировках. Множества - естественный инструмент для конструирования объектов и изучения их внутренней структуры, категории - для изучения взаимосвязей с другими объектами.
>>13673
Никак. Тебе вообще никто не гарантирует существование каких бы то ни было "формул". Даже чтобы написать ряд тейлора для первообразной, тебе сначала надо понять, как интегрировать мономы.
Аноним 27/03/17 Пнд 17:03:18 #552 №13676 
Поясните за современные основания математики и геометрии (что сейчас принято в обоих).
Аноним 27/03/17 Пнд 17:09:36 #553 №13677 
>>13675
Естественные операции? Кому вы рассказываете, в самом деле. Например, аксиома подстановки - это не естественная операция, а чуть ли не оккультизм.
Аноним 27/03/17 Пнд 17:10:59 #554 №13678 
>>13633
спасибо уже решил, это одно из заданий которое будет на контрольной
Аноним 27/03/17 Пнд 17:20:45 #555 №13679 
>>13660
Нихуя ты загнул.

>>13654
А про остальные разделы математики? Статистика, комплексный анализ, вероятности.
Аноним 27/03/17 Пнд 17:23:05 #556 №13680 
>>13679
>Нихуя ты загнул.
Что не так? Очень простая и доходчивая книга. Скачать то же 4 издание на либгене .
Аноним 27/03/17 Пнд 17:25:55 #557 №13681 
>>13675
Интегралы основываются обычно на том что они обратные по значению от производных.
А юез обоснования этого, просто как площадь под графиком/предел суммы можно или нет?
Аноним 27/03/17 Пнд 17:27:12 #558 №13682 
>>13679
http://matematika.phys.msu.ru/stud_gen
Аноним 27/03/17 Пнд 17:27:29 #559 №13683 
>>13681
Ну те же суммы Римана тебя чем не устраивают? Ко-ко-ко Лебег ко-ко-ко.
Аноним 27/03/17 Пнд 17:36:11 #560 №13684 
>>13682
>кафедра математики
>любые книжки по матанализу и такому для нематематиков-первокурсников
Аноним 27/03/17 Пнд 17:37:16 #561 №13685 
>>13684
это для физиков
Аноним 27/03/17 Пнд 17:38:43 #562 №13686 
>>13684
Хотя там не везде книги есть, но вообще просто ищи X для физиков
Аноним 27/03/17 Пнд 17:49:11 #563 №13687 
Почему в школах не учат теорию чисел?
Аноним 27/03/17 Пнд 17:50:02 #564 №13688 
>>13687
По той же причине почему в школах не учат общению и взаимодействию с другими людьми.
Аноним 27/03/17 Пнд 17:50:20 #565 №13689 
>>13687
По определению.
Аноним 27/03/17 Пнд 17:50:58 #566 №13690 
>>13677
Да нет, что же в ней оккультного. Множество, по своей сути - это чёрный ящик, умеющий говорить "да" и "нет". От того, что мы к его выходу добавим ещё одну дополнительную проверку, это не перестанет быть чёрным ящиком, умеющим говорить "да" и "нет".
Аноним 27/03/17 Пнд 17:54:18 #567 №13691 
>>13687
учат
Аноним 27/03/17 Пнд 18:05:43 #568 №13692 
Лучше сначала учить аналитическую геометрию, а потом линейную алгебру?
Аноним 27/03/17 Пнд 18:15:07 #569 №13693 
>>13692
у нас на физфаке книжки были по двум предметам сразу, в таком порядке, как в книжке и учишь
Аноним 27/03/17 Пнд 18:41:27 #570 №13695 
>>13693
сложно на физфак попасть? Собираюсь туда идти, но олимпиады не затащил (но в принципе некоторые знания и умение работать имеются- спохватился слишком поздно что надо чуть-чуть выходить за школьный курс).
Так-то я бы на физтех пошёл, мне тупо 15 минут на элке ехать до него, а без олимпиад лапу сосать- самое обидное что в /un/ аноны ноют что "луче бы на физфак поступал патаму что там тёлочки гыгыг, а на физкеке тока суецид да суецид", а я, наученный ошибками социального взаимодействия, сейчас стремлюсь стать maximum dodik
Аноним 27/03/17 Пнд 18:58:59 #571 №13697 
>>13695
Я по олимпиадке поступал, в белорусский БГУ.
В мой год недобор был.
Аноним 27/03/17 Пнд 19:08:41 #572 №13698 
>>13697
>БГУ
это который BSU или BRGU???
У вас там есть что-то наподобие СУНЦа?
Мне просто кажется ещё что бОльшая часть СУНЦов после 11го у себя на родине останутся, в своём вузе тут есть вообще кто с физфака МГУ?
Аноним 27/03/17 Пнд 19:12:47 #573 №13699 
>>13698
BSU
Есть Лицей БГУ
Аноним 27/03/17 Пнд 19:19:38 #574 №13700 
>>13690
Аксиома говорит вот что. Пусть ф - правильно построенная формула языка теории множеств первого порядка, в которую буква B не входит свободно. Пусть p - её строчка параметров. Тогда для любого множества A и любого набора параметров p если ф функциональна относительно A, то существует множество B, состоящее в точности из таких элементов, которые являются ф-образами элементов A.

Эта аксиома является эзотерической потому, что позволяет легко доказывать существование множеств, о пустоте или непустоте которых в принципе ничего сказать нельзя.

Например, правильно построенной формулой является формула "мощность x строго больше алеф-0, но строго меньше континуума". Применив эту формулу к какому-нибудь кардиналу большему, чем континуум, мы получим множество кардиналов M, пустота которого равносильна континуум-гипотезе. Множество M существует, но его элементы никакими средствами не могут быть изучены. Нелепость какая-то.

А ведь ф - любая правильно построенная формула. Которая к тому же может зависеть от любого числа параметров. Это, если вдуматься, позволяет буквально доказать существование Ктулху.

В самом деле, доказательство сводится к точному теоретико-множественному описанию того, что мы понимаем под Ктулху. Если C(x) - утверждение, что множество x является Ктулху, то может быть построена формула f(x, y), которая функциональна по x и каждому иксу сопоставляет Ктулху. Применив эту формулу к какому-нибудь одноэлементному множеству, мы докажем, что существует множество, единственным элементом которого является Ктулху.

Более того, если считать, что мы имеем право перенумеровать Великих Древних ординалами, то мы можем доказать существование любого наперед заданного количества экземпляров Ктулху. Даже алефа с трансфинитным нумером различных ктулх.
Аноним 27/03/17 Пнд 19:47:28 #575 №13703 
>>13700
Фтагн, фтагн.
Не хотите ли вы стать пропагандистом ктулху? Оплата по собеседованию, ставка плюс количество новообращенных.
Аноним 27/03/17 Пнд 20:13:17 #576 №13706 
В Википедии витиевато написано о числах Мерсена. Если задано 2^n - 1 , где n это простое число, на выходе получиться простое число?
Аноним 27/03/17 Пнд 20:17:27 #577 №13707 
>>13706
2^8-1=255
Аноним 27/03/17 Пнд 20:18:08 #578 №13708 
>>13707
8 не простое число. Простое число это такое число, что делиться только на себя и на единицу.
Аноним 27/03/17 Пнд 20:25:56 #579 №13709 
>>13706
не обязательно
в вики написан контрпример
Аноним 27/03/17 Пнд 20:27:32 #580 №13710 
>>13708
Бля, я прочитал жопой.
Аноним 27/03/17 Пнд 20:30:32 #581 №13711 
>>13709
Тогда зачем это нужно? Я ведь тоже могу придумать свой одночлен, но почему его назовут моим именем? А есть такое, чтобы получались?
Аноним 27/03/17 Пнд 20:31:24 #582 №13712 
>>13711
Насколько я знаю (я не специалист), нету такого чтобы получалось, но люди постоянно пытаются придумать.
Аноним 27/03/17 Пнд 20:39:31 #583 №13713 
>>13712
Нашел еще какие-то числа Софи Жермен, но там приписка, что и для них не доказано бесконечность такого ряда. Что есть лучшим решением?
Аноним 27/03/17 Пнд 21:06:37 #584 №13714 
>>13706
>Если задано 2^n - 1 , где n это простое число, на выходе получиться простое число?
Нет. Обратное утверждение верно.
Аноним 27/03/17 Пнд 21:54:20 #585 №13716 
Как можно переучиться?Перестать быть шаблонным дауном которого ставит в тупик что-то новое и начинать думать.
Аноним 27/03/17 Пнд 22:50:20 #586 №13717 
>>12775 (OP)
Производная! Все учите производные! С помощью производной я доказал небесконечность вселенной!!!
Аноним 27/03/17 Пнд 22:54:38 #587 №13718 
Да, да, говорю вам: бесконечности нет. Если мир бесконечен — то средняя плотность материи в нем должна быть равна нулю. А так как она не нуль — это мы знаем — то, следовательно, вселенная — конечна, она — сферической формы и квадрат вселенского радиуса, y2 = средней плотности, умноженной на... Вот мне только и надо подсчитать числовой коэффициент, и тогда... Вы понимаете: все — конечно, все — просто, все — вычислимо; и тогда мы победим философски, — понимаете?
Аноним 27/03/17 Пнд 23:11:27 #588 №13719 
Снимок экрана от 2017-03-27 22-54-10.png
>>13718
>>13717
двач.ую
кароч сижу такой учю усердно производные и тут ты такой пишешь про производные ну и я, так сказать, подумал "вот ето совпадение!" и решил ответить тебе пиздец долго со скриншотом ебался
Аноним 27/03/17 Пнд 23:17:01 #589 №13720 
>>13700
>но его элементы никакими средствами не могут быть изучены.
Кажется, ты опять путаешь формальные теории и содержательные. ZF - это не священное писание, не 10 заповедей теории множеств. Это просто некий конкретный набор операций, которые над множествами можно делать. Они сформулированы в виде аксиом просто для того, чтобы не было разночтений в том, каких свойств мы ожидаем от множеств. Мир множеств на самом деле намного богаче, чем то, что из них выводимо. Вместо эзотерики вроде трансфинитных ординалов можно взять самые обычные натуральные числа и корни диофантовых уравнений. Уж это-то объект совершенно земной, и корни либо существуют, либо нет. А вот поди ж ты. Это говорит лишь о том, что наш инструментарий недостаточен для таких вещей.
Аноним 27/03/17 Пнд 23:21:20 #590 №13721 
>>13716
>>13652
Аноним 28/03/17 Втр 01:16:02 #591 №13724 
Насколько быстро растет количество делителей числа? Сколько максимум делителей может быть у чисел порядка 10^15?
Аноним 28/03/17 Втр 02:07:02 #592 №13726 
Здравствуйте

посоветуйте что интересного можно почить про операционное исчисление, оригиналы?

в универе данную тему не дали..а самому въезжать сложна
Аноним 28/03/17 Втр 02:43:42 #593 №13727 
>>13724
Ну, к примеру, число 13492656777600 имеет 11520 делителей. Это довольно близко к максимуму.
>>13726
Википедия -> преобразование Лапласа. Там и теория, и свойства, и даже таблица изображений. Не хватит - смотришь раздел "библиография" и читаешь что приглянётся. Но вряд ли в этом будет нужда, на твоём уровне тема совершенно элементарная.
Аноним 28/03/17 Втр 04:59:00 #594 №13730 
>>13720
С такой точки зрения должна существовать некая единая вселенная множеств, а все формальные теории суть просто попытки описать эту гипотетическую единую вселенную. Единая вселенная существовала бы, даже если бы ни одной формальной теории не было.
Аноним 28/03/17 Втр 09:56:09 #595 №13733 
>>13727
> на твоём уровне тема совершенно элементарная
ага, когда препод так говорит, оказыввается всё наоборот
Аноним 28/03/17 Втр 12:55:10 #596 №13736 
Как перевести систему счисления на иррациональное основание, например, число Эйлера?
Аноним 28/03/17 Втр 12:58:40 #597 №13737 
>>13736
Нах?
Аноним 28/03/17 Втр 12:59:47 #598 №13738 
>>13737
Для удовольствия.
Аноним 28/03/17 Втр 13:14:27 #599 №13739 
Это правда, что 3/3 равно 0.9999999999999 ?
Аноним 28/03/17 Втр 13:27:29 #600 №13740 
>>13739
как же вы заебали
0.(9) = 1
Аноним 28/03/17 Втр 13:37:55 #601 №13741 
>>13739
Да, раздели 1 на 3, затем умножь на 3.
Аноним 28/03/17 Втр 13:42:20 #602 №13742 
>>13736
Системы счисления бывают только с целым основанием. Можно, конечно, придумать какой-нибудь изврат, но он будет абсолютно неюзабелен.
Аноним 28/03/17 Втр 13:53:51 #603 №13744 
>>13742
Но математики постоянно придумывают извраты. Не уж-то нет готового уже?
Аноним 28/03/17 Втр 14:38:34 #604 №13749 
>>13736
https://en.wikipedia.org/wiki/Non-integer_representation
Аноним 28/03/17 Втр 15:04:03 #605 №13750 
>>13742
https://habrahabr.ru/post/302178/
Аноним 28/03/17 Втр 15:06:55 #606 №13751 
Бомбит от математиков. Хули вы такие умные? Вот, блядь, одним дано, а другим не дано, и всё, хоть выпиливайся. Можешь зазубрить, но интуитивно понять могут только избранные. И в жизни вы на порядок умнее, сука. Настолько умнее, что до быдлопроблем типа политики, денег, статусности, айфонов,подворотов, еот, прочего говна и быдла вроде нас вам никакого дела нет. Конечно, мне бы тоже не было, если бы я видел эту красоту и умел летать. Но я слепой червь, блляядь. Сгорел.
https://www.youtube.com/watch?v=494uupfgWSQ
Аноним 28/03/17 Втр 16:02:24 #607 №13752 
>>13751
Хуйню какую-то несешь, нужно просто понимать, а не "интуитивно". Ни разу еще не встречал человека, который захотел что-то конкретное понять, старался для этого, но все равно нихуя не понял. Остальные же только сидят и ноют, мол склад ума не тот, математику/физику только избранные понимают и прочие кукареки.
Аноним 28/03/17 Втр 16:08:52 #608 №13753 
>>13752
А я встречал. Старательный парень, не быдло, на все пары ходит, всё записывает, пытается разобраться, но вот просто мозгами природа обделила. И это не только в математике - у него во всём так, вплоть до ориентирования на местности. Поступил по квоте из Крыма.
Аноним 28/03/17 Втр 16:11:55 #609 №13754 
>>13752
> но все равно нихуя не понял
Я не говорил про нихуя, я говорил про то, что смысла не ловишь. Есть формула какая-то, я вроде понимаю как она получилась и как она работает и когда применять и прочее. Но если спросить меня почему так, я обосрусь, потому что НЕ ПОНИМАЮ почему так. Вот, например, биномиальные коэффициенты, разложение (1+x)^n, число e. Я только на днях понял, что это всё вытекает из треугольника Паскаля, а он из фундаментальных свойств чисел, из правил, по которым из единиц получаются все числа. И вот это только сейчас я понял, до этого это были разрозненные знания из нескольких областей математики, а сколько ещё мимо меня прошло? Да хуй знает. Провал, понимаешь? Это провал, я ебланище тугое, могу повторить, заучить грубо говоря, но смысла не увижу в большинстве случаев.
У меня вот слух абсолютный, могу мелодию из матрицы на слух наиграть, а играть не умею, и ебать мне слух режет, когда люди без слуха петь пытаются. Слуху нельзя обучиться, так и математическому слуху ты и за двадцать лет не научишься. А хочется почувствовать ту красоту и глубину связей,ту мистику, которую я на дгнях от треугольника Паскаля словил
Аноним 28/03/17 Втр 16:18:37 #610 №13755 
>>13754
> математическому слуху ты и за двадцать лет не научишься
У меня был знакомый из консерватории, он без нот играть не мог, вот я про что. Кнопки нажимает и получается музыка. Тут что-то вроде того, математические кнопки ты нажимать то можешь научиться, но слышать математику некоторые могут вообще не умея кнопки нажимать, а некоторые будут хуй сосать по жизни.
Аноним 28/03/17 Втр 16:20:38 #611 №13756 
>>13755
Или дальтоники те же, они ж не переходят на красный свет, понимают, что это красный, но не видят его таким, какой он на самом деле.
Аноним 28/03/17 Втр 17:35:59 #612 №13760 
>>13754
>Слуху нельзя обучиться
пиздёж
Аноним 28/03/17 Втр 17:49:32 #613 №13761 
Какой ICQ нужно иметь дабы понимать хоть математику, которую вроде бы должен знать каждый образованный человек?
Аноним 28/03/17 Втр 17:52:29 #614 №13762 
>>13760
>Относительный слух, или интервальный слух, — способность помнить разницу в высоте звуков. Эта важнейшая составляющая музыкального слуха даёт музыканту возможность определять интервалы, — то есть частотные отношения между нотами в мелодии, гармонических интервалах и аккордах.
>Развитие относительного слуха входит в обязательную программу профессиональной подготовки музыканта, этим занимается дисциплина сольфеджио.


>Абсолютный слух — способность достаточно точно помнить высоту звука, позволяющая носителю определять и называть любые услышанные ноты без предварительного прослушивания заранее известных звуков
>Эта способность может формироваться в раннем детстве, а во взрослом возрасте её приобретение крайне затруднительно.
Аноним 28/03/17 Втр 17:55:31 #615 №13763 
>>13762
крайне затруднительно != невозможно
Аноним 28/03/17 Втр 17:57:35 #616 №13764 
>>13762
http://lmgtfy.com/?q=%D1%83%D0%BF%D1%80%D0%B0%D0%B6%D0%BD%D0%B5%D0%BD%D0%B8%D1%8F+%D0%BD%D0%B0+%D1%80%D0%B0%D0%B7%D0%B2%D0%B8%D1%82%D0%B8%D0%B5+%D0%B0%D0%B1%D1%81%D0%BE%D0%BB%D1%8E%D1%82%D0%BD%D0%BE%D0%B3%D0%BE+%D1%81%D0%BB%D1%83%D1%85%D0%B0
Аноним 28/03/17 Втр 17:57:56 #617 №13765 
>>13763
Это значит, что если и возможно, то крайне затруднительно, единичные случаи, да и то не факт, что эти люди не обладали им до этого, в то время как пыталось учиться этому и учить очень много людей.
Аноним 28/03/17 Втр 17:58:04 #618 №13766 
>>13761
100
Аноним 28/03/17 Втр 17:58:43 #619 №13767 
>>13764
И что это доказывает?
Аноним 28/03/17 Втр 18:00:32 #620 №13768 
>>13764
В. Б. Брайнин
О НЕКОТОРЫХ ВОЗМОЖНОСТЯХ РАЗВИТИЯ АБСОЛЮТНОГО
СЛУХА1
Если читатель рассчитывает на сенсацию, его ждёт разочарование.
Развитие абсолютного слуха в тех случаях, когда предварительное
тестирование предрасположенности к нему не обнаружило, станет
возможным (или невозможным) лишь тогда, когда природа абсолютного
слуха прояснится. Из обилия публикаций на эту тему однозначной картины
пока что не складывается. Указывается на то, что целые народы могут
обладать абсолютным слухом, если с рождения говорят на так называемых
тональных языках (китайцы, вьетнамцы – мой опыт преподавания группе
вьетнамцев-немузыкантов это подтвердил). Есть сообщения о том, что все
младенцы имеют соответствующий потенциал, который закрепляется лишь
будучи востребованным. В нашей школе мы проводим эксперимент по
обучению малышей с 12-ти месяцев, но, во-первых, это уже не младенцы, а,
во-вторых, о результатах говорить пока преждевременно – эксперимент
сравнительно свежий.
http://www.brainin.org/Method/absolute_pitch_RU.pdf
Аноним 28/03/17 Втр 18:02:20 #621 №13770 
>>13768
>если с рождения говорят на так называемых
>тональных языках
кек, китайский, например, использует относительный слух
Аноним 28/03/17 Втр 18:04:34 #622 №13771 
>>13767
Короче, мне реально влом объяснять, но допустим, чтобы освоить X нужно затратить 10000 часов. Если у тебя талант, дели это на 2-10, если ты гений, то еще на 2-10. То, что у тебя таланта нету, говорит только о том, что придется больше времени убить. Поэтому рационально расходовать время на то, что тебе дается. Но люди иррациональны.
Аноним 28/03/17 Втр 18:05:24 #623 №13772 
>>13770
Да не о том же речь, я говорю о том, что в математике тоже существует похожая штука, интуитивное понимание, которое неясно как развивать и вообще возможно ли это во взрослом возрасте. Вспомни, тот же Гаусс в 15 лет себя проявил, да и у других, Римана, например, похожие истории были.
Аноним 28/03/17 Втр 18:12:53 #624 №13774 
>>13771
Понимаешь, некоторые вещи ты не осилишь сколько ты не потрать. Ты просто неверно будешь видеть, ну дело же не в упорстве и усердии, вот выше я пример писал про связь e,(1+x)^n и треугольник паскаля, я вообще на это случайно вышел, когда в б фракталы постили и я стал их гуглить. Я учился неплохо, вуз технический закончил, но только тогда разные факты из комбинаторики, матанализа, алгебры, теорвера получили настоящее объяснение и понимание почему так в голове, а не видимое.
Попал в ноту. Когда в следующий раз такое произойдёт и произойдёт ли вообще неясно, никак ты это не разовьёшь, а если вспомнить какой объём часов у нас был посвящён математике я 95% просто-напросто упустил. И ведь проблема в том, что упорство, 1000 часов это всё круто, но тут ты думаешь, что понимаешь, решаешь задачи, получаешь одобрение препода, сдаёшь экзамены с теорией и практикой, у тебя даже мысли не возникнет, что ты вообще нихуя не знаешь на самом деле.
Мне кажется вообще, что детям, рассказывая теоремы и доказательства, начиная с теоремы Пифагора, нужно ещё рассказывать то, как автор к этому пришёл, тогда мб не так всё хуёво будет. Потому что во взрослом возрасте это походу почти нереально развить.
Аноним 28/03/17 Втр 18:12:58 #625 №13775 
Как понять модульную арифметику? Можно ли с ней вообще работать вручную?
Аноним 28/03/17 Втр 18:13:37 #626 №13776 
>>13774
> когда в б фракталы постили
https://ru.wikipedia.org/wiki/%D0%A2%D1%80%D0%B5%D1%83%D0%B3%D0%BE%D0%BB%D1%8C%D0%BD%D0%B8%D0%BA_%D0%A1%D0%B5%D1%80%D0%BF%D0%B8%D0%BD%D1%81%D0%BA%D0%BE%D0%B3%D0%BE
>Если в треугольнике Паскаля все нечётные числа окрасить в чёрный цвет, а чётные — в белый, то образуется треугольник Серпинского.
Забыл.
Аноним 28/03/17 Втр 18:14:13 #627 №13777 
>>13776
То есть если бы в б кто-то не запостил фракталы я бы никогда так и жил дальше, думая,что что-то знаю.
Аноним 28/03/17 Втр 18:16:36 #628 №13779 
>>13777
и это норм
Аноним 28/03/17 Втр 18:17:56 #629 №13780 
>>13779
Именно, блядь! Потому и сгорел!Жил бы и жил себе, а так я типа приоткрыл дверцу, но понимаю, что никогда, блядь, никогда всей красоты не увижу. Я в жизни никогда никому не завидовал. А теперь зависть какая-то мерзкая гложет, злюсь на себя за это и горииит очаааг.
Аноним 28/03/17 Втр 18:26:11 #630 №13781 
>>13772
>Гаусс
> Уже в двухлетнем возрасте мальчик показал себя вундеркиндом. В три года он умел читать и писать, даже исправлял счётные ошибки отца. Согласно легенде, школьный учитель математики, чтобы занять детей на долгое время, предложил им сосчитать сумму чисел от 1 до 100. Юный Гаусс заметил, что попарные суммы с противоположных концов одинаковы: 1+100=101, 2+99=101 и т. д., и мгновенно получил результат: 5050
>Первый эффектный успех пришел к Гауссу, когда ему не было еще девятнадцати - доказательство того, что можно построить правильный 17 - угольник циркулем и линейкой.
>В 1796 году Карл Фридрих Гаусс, учащийся первого курса Геттингенского университета, решил задачу, перед которой математическая наука пасовала более двух с лишним тысяч лет. Несмотря на то, что еще древними греками были найдены способы построения с помощью только лишь циркуля и линейки правильных многоугольников с числом сторон 3, 4, 5, 15, а также с числом сторон, большим в 2 раза, в отношении прочих правильных многоугольников царила полная неизвестность. И вот именно в этот день будущий «король математиков» Гаусс догадался, как построить правильный 17-угольник, кстати, также, с помощью циркуля и линейки

2000 лет, анон. Чувак в школе изобрёл арифметическую прогрессию. Какие нахуй 10000 часов? Я, конечно, экстремальный пример привёл, но именно ту внутреннюю интуицию, только гораздо меньшей силы, я имею ввиду.
Аноним 28/03/17 Втр 18:38:53 #631 №13782 
>>13781
То есть как и со слухом отсутствие этой интуиции не помешает тебе стать математиком или просто хорошо усваивать материал
>Бытующим недоразумением является представление, что обладание абсолютным слухом — гарантия музыкальной гениальности[4]. Иногда родители, обнаружив это качество у ребёнка, заставляют его заниматься музыкой, даже если нет ни особых способностей, ни желания. Другая сторона этого же недоразумения — мнение, что без абсолютного слуха в музыке ничего серьёзного добиться нельзя (что опровергается хотя бы примерами П. И. Чайковского или Р. Вагнера).
>Абсолютный слух полезен для музыкальной деятельности, но, вопреки бытующему мнению, не приносит своему обладателю значительных преимуществ.
но без неё ты никогда полностью не увидишь всей красоты. Это нечестно.
Аноним 28/03/17 Втр 19:32:19 #632 №13783 
Как получить геометрическое просветление? Всегда геометрия давалась плохо. Может есть еще какой-то науч-поп по аналитической геометрии?
Аноним 28/03/17 Втр 19:41:51 #633 №13784 
logo.png
>>13783
Гугли пифагоровы числа.
Аноним 28/03/17 Втр 19:43:15 #634 №13785 
>>12775 (OP)
математика нужны чтобы меньше думать.
Аноним 28/03/17 Втр 19:46:28 #635 №13786 
>>13784
Погуглил, посмотрел. Забавно конечно, но непонятно как это поможет.
Аноним 28/03/17 Втр 19:47:50 #636 №13787 
metapost.85.gif
>>13786
А так?
Аноним 28/03/17 Втр 19:49:42 #637 №13788 
>>13787
Выглядит привлекательно, но я сомневаюсь, что могу как-то сосчитать, ввести какую-то формулу и прочая для этой картинки. Могу только смотреть, может даже любоваться.
Аноним 28/03/17 Втр 19:56:24 #638 №13789 
>>13781
Да он там такую хуйню творил в 12 лет написал какую то дисертацию что йоба ученые того времени не верили что аффтару этого талмуда 12 лет.
Аноним 28/03/17 Втр 20:56:17 #639 №13794 
>>13754
> Но если спросить меня почему так, я обосрусь, потому что НЕ ПОНИМАЮ почему так.
Потому что надо доказательства всех утверждений читать. Естественно, если быть быдлом и просто применять формулы, которые препод дал, не будешь ничего понимать. Это не только к математике относится, но и ко многим другим областям жизни.
Аноним 28/03/17 Втр 20:58:42 #640 №13795 
>>13761
Нужен icq как у хлебушка, чтобы верить в то, что существует некий набор знаний, которым должен обладать каждый "образованный" человек.
Аноним 28/03/17 Втр 21:06:00 #641 №13796 
>>13795
Такой набор знаний действительно существует. Уж по крайней мере математика по 6 класс включительно в него входит.
Аноним 28/03/17 Втр 21:08:16 #642 №13797 
>>13795
Согласен
Аноним 28/03/17 Втр 21:08:44 #643 №13798 
>>13795
Не с тобой, с этим>>13796

Аноним 28/03/17 Втр 21:09:41 #644 №13799 
>>13794
Доказательства надо помнить?
Аноним 28/03/17 Втр 21:12:33 #645 №13801 
>>13799
Ход мысли из доказательства скорее
Аноним 28/03/17 Втр 21:14:02 #646 №13802 
>>13801
Как вычленять ход мысли? Есть алгоритм?
Аноним 28/03/17 Втр 21:16:49 #647 №13803 
>>13774
Читал я историю одного математика, он свой предмет начал понимать только на 2-3 год преподавания.

>>13802
Зачем тебе алгоритм, чтобы вычленить из доказательства основные логические ходы?
Аноним 28/03/17 Втр 21:18:21 #648 №13804 
>>13803
То есть для доказательств надо использовать логику?
Аноним 28/03/17 Втр 21:35:39 #649 №13806 
>>13804
Скорее даже не логику, а идеи.
Например, интегралы связаны общий идеей разбиения на бесконечно малые величины, которые затем суммируются. Уже по этому ты сможешь сперва кратко, а затем подробно восстановить у себя в голове ход доказательства.
Аноним 28/03/17 Втр 21:37:19 #650 №13807 
>>13806
Может где-то описаны эти идеи?
Аноним 28/03/17 Втр 21:41:19 #651 №13808 
>>13796
> Такой набор знаний действительно существует. Уж по крайней мере математика по 6 класс включительно в него входит.
Очень интересно узнать, по каким критериям ты определяешь, что входит в этот набор, а что нет. Вот представь, что я тоже верю в существование такого набора. Допустим я считаю, что любой образованный человек должен знать определение хаусдорфова пространства. Попробуй оспорить.
Аноним 28/03/17 Втр 21:47:21 #652 №13809 
>>13807
Они именно в доказательствах, и их оттуда надо научиться вычленять. Сначала это будет непросто, и из-за этого надо будет возвращаться к ранее пройденному, чтобы вспоминать и закреплять эти идеи.
Выписывай леммы, теоремы и определения, записывай доказательства, следи за тем, как доказывают и из каких соображений вообще это нужно.

>>13808
Имхо, набор необходимых знаний не просто существует, а его ещё и расширять постоянно нужно.
Аноним 28/03/17 Втр 21:47:48 #653 №13810 
>>13808
Ой, оставь свою демагогию при себе. Ясно, что человек, не умеющий читать и писать, никак не может считаться образованным.
Аноним 28/03/17 Втр 21:48:42 #654 №13811 
>>13799
Не знаю. Вопрос был в том, как понять "почему так". Для меня "понять почему так" = "понять доказательство" = "разложить переходы между утверждениями, которые мне кажутся неочевидными, на последовательность маленьких очевидных переходов".
Что за фигня? Аноним 28/03/17 Втр 22:15:04 #655 №13814 
333.jpg
Как это может быть ОДНОВРЕМЕННО? На 110% ведь = на 10%?
Так всё-таки, НА СКОЛЬКО?
Аноним 28/03/17 Втр 22:21:07 #656 №13815 
>>13814
110% = 10% + 100%
либо
210% = 110% + 100%
Очевидно имеется в виду второе.
Аноним 28/03/17 Втр 22:49:22 #657 №13817 
>>13810
Я думаю, что слово "образованный" не имеет никакого смысла, пока не зафиксируешь предметную область. Умение читать и писать - это грамотность. Неграмотный человек не может стать образованным ни в какой области, потому что у него нет доступа к информации. Поэтому его можно назвать необразованным. Человек, который не знает математику за 6 класс, все еще имеет доступ к очень большому количеству информации.
Аноним 28/03/17 Втр 22:51:33 #658 №13818 
>>13802
>Человек, который не знает математику за 6 класс, все еще имеет доступ к очень большому количеству информации.
Например?
Аноним 28/03/17 Втр 22:51:50 #659 №13819 
>>13817
>>13818

Аноним 28/03/17 Втр 22:54:56 #660 №13820 
>>13817
Образованность - это предмет консенсуса. Ты образованный, если достаточное количество людей согласно тебя таковым признать, обладая точной информацией о твоих знаниях и навыках. Если ты убедишь большинство людей, что без хаусдорфовости жизни нет, то это немедленно станет обязательным условием, чтобы называться образованным человеком.
Аноним 28/03/17 Втр 23:34:08 #661 №13824 
>>13820
Я утверждаю, что в любой стране большинство (т е больше половины) людей не способны решить квадратное уравнение, потому что много лет этим не занимались. Следовательно, знание математики за 6 класс не является необходимым условием образованности.
Аноним 28/03/17 Втр 23:41:02 #662 №13825 
>>13824
В моё время квадратные уравнения были в седьмом классе. А шестой - это дроби.
Аноним 28/03/17 Втр 23:47:10 #663 №13826 
>>13796
Каждый сам для себя устанавливает уровень, на котором ему комфортно знать ту или иную область науки. Слишком много факторов имеется, в том числе и никак не зависящих от человека, которые человек учитывает, устанавливая этот уровень.
Аноним 28/03/17 Втр 23:48:39 #664 №13827 
>>13826
>Слишком много факторов имеется,
Чтобы бросаться такими словами как "каждый, должен, образованность" и прочими.
Аноним 29/03/17 Срд 01:05:21 #665 №13828 
MSP25771f07e2533gd556790000347c8a55hb19h132.gif
MSP316123g569g8g458f97200004bggc6cf738df64h.gif
Прикиньте, я тут видос смотрел по истории математики научпопный, ну и короче там про Египет рассказывали, типа они считали, что площадь круга диаметром 9 равна площади квадрата со стороной 8 64 для них было священное число и там про глаз Гора рассказывали, типа там его глаз расхуячило на 64 осколка, и все осколки боги ему помогли обратно восстановить, кроме последнего, а для меня 64 ещё более священное число по некоторым причинам, но я в отличие от ведущего знал, что 64 это 63 и 0, потому эта история меня заинтересовала и стал алхимичить короче
Ну и чё, в итоге походу наебались учёные и ведущий, лол. Я ещё думаю как так такие сооружения строили и на целых две сотых наёба.
А выходит вот: пикрелейтед равен 3.1416313. Второе число это 1-1/(золотое сечение), которое им точно было известно. Такие дела.
А 4/81 получается из того, что диаметр равен 9, радиус равен 9/2, в квадрате получается (9/2)^2= 81/4. Домножаем на pi, получаем 63.6172512352
63+2/(sqrt(5)+1) же равно 63.618034
Пишу потому что мало ли меня приглючило под ночь, лол и там ошибка какая-то бредовая.


>Египтяне предполагали, что площадь круга S диаметром d равна площади квадрата, сторона которого составляет 8/9 диаметра: Это правило соответствует приближению ≈ 3,1605 (погрешность менее 1 %)
https://ru.wikipedia.org/wiki/%D0%9C%D0%B0%D1%82%D0%B5%D0%BC%D0%B0%D1%82%D0%B8%D0%BA%D0%B0_%D0%B2_%D0%94%D1%80%D0%B5%D0%B2%D0%BD%D0%B5%D0%BC_%D0%95%D0%B3%D0%B8%D0%BF%D1%82%D0%B5#.D0.92.D1.8B.D1.87.D0.B8.D1.81.D0.BB.D0.B5.D0.BD.D0.B8.D0.B5_.D0.BF.D0.BB.D0.BE.D1.89.D0.B0.D0.B4.D0.B5.D0.B9
Выходит, у египтян точность повыше то была, чем 1 %, лол.
Аноним 29/03/17 Срд 01:19:11 #666 №13829 
MSP7771hc24ca4bi63d46900002e0bcd13d552g17h.gif
>>13828
>2/(sqrt(5)+1)
Охуеть, загуглил, и вот как это число представляется в виде цепной дроби. Нет, вы понимаете? АБСОЛЮТНАЯ ДРОБЬ, ОСКОЛОК ГЛАЗА ГОРА. Сакральненько.
Аноним 29/03/17 Срд 01:22:11 #667 №13830 
220px-Oudjat.SVG.png
oko-uadzhet2.jpg
>>13829
>ОСКОЛОК ГЛАЗА ГОРА
>Сумма шести знаков, входящих в Уаджет, и приведенных к общему знаменателю: 32/64 + 16/64 + 8/64 + 4/64 + ²/64 + 1/64 = 63/64
Аноним 29/03/17 Срд 11:48:10 #668 №13833 
>>13828
>>13829
>>13830
И что дальше? Может ты еще архимедовскую задачу о вычислении площади круга распишешь.
Аноним 29/03/17 Срд 13:46:29 #669 №13835 
>>13833
Да ничего, просто думаю, что египтяне использовали более точное значение пи, чем об этом пишут.
Аноним 29/03/17 Срд 13:56:48 #670 №13836 
>>13833
Ну и в Индии, например,долгое время юзали pi=3.1416, с его помощью рассчитывали длину экватора и только на 100 км ошиблись. Показалось интересным, что в таком приближении золотое сечение всплывает, хотя считается, что между пи и золотым сечением явной связи нет.
Аноним 29/03/17 Срд 16:09:38 #671 №13838 
MSP25151giaegb6de17c6if0000222hcde4672bigg8.gif
>>13833
Ну и ещё оттуда точное значение пи выводится, лол. Да хз, дело было ночью, смотрел передачу, услышал про то, что в Египте площадь квадрата со стороной 8 считали равной площади круга с диаметром 9 и чёт меня понесло. Сорян, что тред засрал.
Аноним 29/03/17 Срд 16:47:48 #672 №13840 
>>13761
90+
Аноним 29/03/17 Срд 17:30:13 #673 №13843 
Capture.PNG
Аноны помогите, пытаюсь решить, делаю вроде по логике все правильно, а ответ не сходится с указанным. (Правильный: 2a)
Нарисовал для себя, и по рисунку получилось так:
(a - MB) + (AC - PC) + MP
Если следовать правильному ответу, то можно AC заменить на a, ведь точки касания равноудалены от точки A? Но дальше я не знаю как сократить, чтобы вышло 2a.
Аноним 29/03/17 Срд 17:44:55 #674 №13844 
Логарифм f(x) по основанию g(x) равно константа. И g и f больше нуля и g в степени константа равно f. Всё так?
Аноним 29/03/17 Срд 18:11:43 #675 №13848 
>>13844
да
Аноним 29/03/17 Срд 18:20:34 #676 №13849 
>>13848
Аноны, подтвердите. Он не врёт?
Аноним 29/03/17 Срд 18:21:20 #677 №13850 
4863093453729f16714cb.jpg
>>13849
Подтверждаю.
Аноним 29/03/17 Срд 18:22:16 #678 №13851 
>>13848
>>13850
Спасибо
Аноним 29/03/17 Срд 18:27:38 #679 №13853 
>>13838
Аноны, я в замешательстве. Множество людей в интернете говорит, что пи не может быть выражено через фи, так как пи трансцендентное число, а фи алгебраическое. Но почему тогда ответ получается верным в любой программе?
Аноним 29/03/17 Срд 18:33:44 #680 №13854 
>>13853
To start off, it's necessary to establish the following:
e and π are transcendental numbers (there does not exist a non-zero polynomial equation with rational coefficients of which either is a root)
phi (by which, I am assuming you mean the golden ratio) is not a transcendental number
So lets say you have a transcendental number a. Exponentiation of a to a rational power, addition of a rational constant to a, and multiplication of a with a rational, nonzero number would all result in a transcendental answer.
Аноним 29/03/17 Срд 18:40:53 #681 №13855 
>>13853
Арксинус тебя не смутил? arcsin 1 = pi / 2, например.
Аноним 29/03/17 Срд 18:46:39 #682 №13856 
Снимок экрана от 2017-03-29 18-44-41.png
>>13855
Но почему тогда пишут, что нельзя выразить и никакой связи между ними быть не может? Я тоже думаю, а как же arcsin(sqrt(3)/2)=pi/3, например?
Аноним 29/03/17 Срд 18:49:53 #683 №13857 
Снимок экрана от 2017-03-29 18-49-32.png
>>13855
Аноним 29/03/17 Срд 18:54:04 #684 №13858 
>>13856
Нельзя выразить алгебраическую связь. А синус и арксинус не являются алгебраическими функциями.
Аноним 29/03/17 Срд 18:56:43 #685 №13859 
Снимок экрана от 2017-03-29 18-54-53.png
>>13858
То есть это такая массовая путаница получается? Спрашивают же не просто об алгебраической связи, а вообще о хоть какой-нибудь.
>Is there any connection between the golden ratio
> Is there an identity that links π and ϕ (the golden ratio)?
>Can the golden ratio accurately be expressed in terms of e or π
Аноним 29/03/17 Срд 18:59:46 #686 №13860 
>>13859
>о хоть какой-нибудь
Это довольно бессмысленный вопрос. Для любых чисел x и y найдется функция, "выражающая одно через другое. Например,
f (t) = t + y - x => f (x) = y

Понятно, что если не ограничивать себя с выбором функции, можно подобрать и какую-нибудь существующую.
Аноним 29/03/17 Срд 19:00:00 #687 №13861 
>>13858
>>13859
>Кстати, связь между «пи» и другой удивительной величиной — золотым сечением — на самом деле так и не доказана.
А на русском вообще искать бесполезно, одна шизотерика и типа вот этого только.
Аноним 29/03/17 Срд 19:01:46 #688 №13862 
>>13860
Ну ладно, спасибо. Да хотел поискать другие интересные соотношения между ними, загуглил
connection between pi and golden ratio а выдало кучу ссылок на форумы, ВиО и там больше половины ответов заплюсованных говорят о том, что нет связи и прочее.
Аноним 29/03/17 Срд 19:40:04 #689 №13865 
>>13857
что-то я проиграл в голос блять. Там на пи сокращается получается ведь кок раз алгебраическое значение ФИ-то
так ведь можно с чем угодно сделать, с e там и с другими
Аноним 29/03/17 Срд 19:43:50 #690 №13866 
Безымянный.png
Добрый вечер. Пишу калькулятор для компьютерной игры про гонки. Нужна помощь в построении формулы.

Мне нужно знать сколько заливать топлива чтобы хватило точно на определенное количество кругов и в какой максимально поздний момент включать гибридную установку.
Допустим мне нужно проехать 15 кругов с минимальным количеством топлива. Помимо двигателя в машине есть гибридная установка, которая за каждый круг подзаряжается на 4%. И при включении тратит 8% на круг, но сокращает расход топлива с 1.4 до 0.5.

Спасибо.

Аноним 29/03/17 Срд 19:46:37 #691 №13867 
>>13866
Нихуя не понял что тебе надо, но тебе скорее всего надо либо в /sci/ либо в /au/ в крайнем случае /vg/.
Аноним 29/03/17 Срд 21:00:27 #692 №13870 
>>13866
абсолютно тривиальная задача, решение которой является следствием теоремы Сосницкого
ну посмотри сколько там для конкретной машины уходит топлива/круг, потом умножаешь на кол-во кругов и заливаешь. Это если без гибридной установки. А гибридную установку в таком случае ты можешь включать в любой очень короткий момент до окончания гонки, т.е. dx. Но как я понял тебе надо так штобы допустим у тебя 15 кругов, и ты кароч допустим 13 проехал на топливе, а ещё 2 на установке. Тогда просто составь уравнение что кол-во топлива является m/(m+n), тогда убер-заряд это n(m+n). Ну и делай уравнение для n, когда за несколько кругов идёт заполнение, а потом в два раза быстрее тратишь. Может быть там ещё о мощности получается (1/m если на одном топливе, а 1/(m+n) если на топливе и убере)
Аноним 29/03/17 Срд 21:05:19 #693 №13871 
>>13843
Я слишком тупой вопрос задал?
Аноним 29/03/17 Срд 21:55:10 #694 №13872 
>>13814
>>13815

Если мы УЖЕ имеем ответ - 210%, что видно из графика, зачем из большего вычитать меньшее? Проценты так не вычисляют. Получается ахинея.
Аноним 29/03/17 Срд 22:13:47 #695 №13874 
>>13843
Ну смотри, в предельном положении получается одна сторона равна 0, а две другие равны a. Остаётся только доказать, что ответ не зависит от положения касательной.
Аноним 29/03/17 Срд 22:20:15 #696 №13875 
>>13872
> Проценты так не вычисляют.
вопрос конвенции
Аноним 30/03/17 Чтв 00:21:42 #697 №13880 
Привет.
Прошу помощи для мотивации.
Я уже всех кого только можно заебал своим нытьём, но, испытаю шанс ещё раз.
В школе не учился (проф.спорт, травма, пропущенная школа). Теперь мне 24 года, год назад взахлёб прочитал книгу Колмогорова "профессия математик", теперь влюблён в эту науку по уши. Однако, мотивации поступать в университет в 25 нет. Старый, кругом будет вчерашняя школота, насмешки, предвзятое отношение и т.д. и т.п. Друзья! Помогите найти мотивацию, чтобы поступить в ВУЗ и заняться наукой. Или добейте до конца.)
Аноним 30/03/17 Чтв 00:29:34 #698 №13882 
>>13880
>)
Добил.
Аноним 30/03/17 Чтв 00:31:47 #699 №13883 
>>13882
И на этом спасибо
Аноним 30/03/17 Чтв 00:36:06 #700 №13884 
30207600.jpg
>>13883
Да ладно, я любя. А нафиг тебе мотивация? В плане, понимаю там мотивация по карьере лезть или учёбу подтягивать. Это ж не тот случай.
Аноним 30/03/17 Чтв 00:40:30 #701 №13885 
>>13884
Ну, чтобы в 25 поступить в университет, где 99% группы - вчерашние школьники, нужна серьёзная мотивация. Чувствовать себя стариком-аутсайдером в конкуренции с более молодыми студентами в перспективе заняться наукой это не очень хорошо.
Аноним 30/03/17 Чтв 00:42:42 #702 №13886 
>>13885
А зачем тебе в университет?
Аноним 30/03/17 Чтв 00:44:32 #703 №13887 
>>13886
Доступ к вычислительным центрам, научным материалам и руководствам, ""корочка"", научный руководитель, работа в лаборатории и т.д. и т.п.
Сейчас быть ученым-одиночкой невозможно
Аноним 30/03/17 Чтв 00:46:01 #704 №13888 
>>13887
>Сейчас быть ученым-одиночкой невозможно
Неправда. Я два часа назад сегодня доказал, что не существует нечётных совершенных чисел, лол.
Аноним 30/03/17 Чтв 00:47:56 #705 №13889 
>>13888
Поздравляю
Аноним 30/03/17 Чтв 01:12:24 #706 №13890 
0.48384400201240206589-3[1].jpg
>>12775 (OP)
Поясните, что там Соловьёв преподаёт
Аноним 30/03/17 Чтв 01:17:50 #707 №13891 
>>13887
>Доступ к вычислительным центрам
Что ты там вычислять собирался?)))
Аноним 30/03/17 Чтв 01:18:31 #708 №13892 
>>13891
Анус пса))))))))00000000000
Аноним 30/03/17 Чтв 01:21:07 #709 №13893 
>>13880
Ты хоть школьной программой владеешь? Может лучше тян найдёшь.
Аноним 30/03/17 Чтв 01:28:17 #710 №13894 
>>13890
На доске элементарная логика для первокурсников. Если точнее - определение формальной теории первого порядка и сбоку, кажется, зайчатки моделей.
Аноним 30/03/17 Чтв 01:29:19 #711 №13895 
>>13894
Но причём тут Соловьёв?
Аноним 30/03/17 Чтв 01:37:23 #712 №13896 
>>13880
> насмешки, предвзятое отношение и т.д. и т.п.
На самом деле нет. В нормальном вузе всем будет плевать, особенно если ты не тупой. Мне сейчас 26, я второкурсник, нормально общаюсь с одногруппниками, никакого барьера из-за возраста нет. А в шарагу идти ради науки бессмысленно.
Аноним 30/03/17 Чтв 01:38:03 #713 №13897 
>>13895
No fucking idea.
Аноним 30/03/17 Чтв 01:38:45 #714 №13898 
>>13896
Расскажи как вкатился, чем занимался до этого, что сподвигло идти учиться?
Аноним 30/03/17 Чтв 01:45:58 #715 №13899 
>>13898
Поступал после школы, но как-то не пошло, забил на всё, отчислили. Предки устроили на завод в родной мухосрани. Я там поработал какое-то время, и понял, что провести вот так всю жизнь я не хочу. Подал заявку на егэ, как выпускник прошлых лет, набрал приличный балл, поступил, учусь. Собственно, вся кулстори.
Аноним 30/03/17 Чтв 01:47:00 #716 №13900 
>>13899
>Подал заявку на егэ, как выпускник прошлых лет
Там же только 4 года действует?
Аноним 30/03/17 Чтв 01:48:46 #717 №13901 
>>13900
Нет. Это само егэ действует то ли 2, то ли 3 года.
Аноним 30/03/17 Чтв 01:50:56 #718 №13902 
>>13901
>Это само егэ
Ну я про него и говорю же. Как-то 26 лет, второй курс, поступал в 24, в 21 писал егэ что ли?
Аноним 30/03/17 Чтв 01:52:22 #719 №13903 
>>13902
Школу окончил в 2007, егэ писал в 2013, поступил в 2015, скопив достаточно денег.
Аноним 30/03/17 Чтв 01:53:44 #720 №13904 
>>13903
Ага, тогда понятно.
Аноним 30/03/17 Чтв 01:57:38 #721 №13905 
>>13903
А зачем деньги?
Аноним 30/03/17 Чтв 01:58:41 #722 №13906 
>>13905
Платно же только можно второе получать. Да и лучше подкопить, чтобы на работу не отвлекаться, наверное.
Аноним 30/03/17 Чтв 02:01:38 #723 №13907 
>>13906
Это не второе высшее. Не имеет значения, сколько раз ты поступал, отчислялся и восстанавливался. Если у тебя нет законченного высшего - ты можешь претендовать на бюджет наравне со всеми. Приёмную комиссию интересуют только баллы егэ, а не твой возраст и предыстория.
>Платно же только можно второе получать
This. Хотя на всё время не хватит один хуй. Максимум до следующей зимы растяну. ДС всё-таки.
Аноним 30/03/17 Чтв 02:05:34 #724 №13908 
>>13906
Второе-то да, но у тебя разве первое есть? Если ни один вуз не выдавал тебе диплом бакалавриата государственного образца, то на бюджет в разные места можно поступать без всяких проблем. Отчислили из одного вуза - можешь по тем же ЕГЭ поступать в другой или даже в этот же самый на этот же факультет (в таком случае даже сданные экзамены автоматически перезачтутся).
Аноним 30/03/17 Чтв 02:07:15 #725 №13909 
>>13908
>но у тебя разве первое есть?
Ага, есть первое. Я чё и удивился, что егэ у тебя остался, у меня в прошлом вышел, 21 если что. А ты, оказывается, уже потом сдавал.
Аноним 30/03/17 Чтв 09:17:51 #726 №13910 
Screenshot20170330-101247~2.png
Не понял рассуждения после
> Чтобы в этом убедиться

Поясните, что за преобразования, да и причем тут скалярное умножение?
Аноним 30/03/17 Чтв 09:54:41 #727 №13911 
>>13910
Тау квадрат — это скалярный квадрат.
Аноним 30/03/17 Чтв 10:08:44 #728 №13912 
>>13911

Но тут ведь говорится про τ и dτ/ds. Потом какие-то не совсем понятные преобразования идут, вроде d(τ²)/ds= 2τ*dτ/ds. Последнее – это что такое и как? Почему не сразу 2τ?
Аноним 30/03/17 Чтв 10:24:22 #729 №13913 
>>13912

Понял, сложная функция. Спасибо за внимание.
Аноним 30/03/17 Чтв 12:01:30 #730 №13915 
Движения плоскости - это изометрии из C в С?
Аноним 30/03/17 Чтв 15:40:23 #731 №13919 
Есть какие-то советы как понять математику на уровне первокурсника и дальше?
Аноним 30/03/17 Чтв 15:44:29 #732 №13920 
>>13915
Да.
>>13919
Читать учебники.
Аноним 30/03/17 Чтв 15:48:00 #733 №13921 
blob
http://cyberleninka.ru/article/n/logika-s-isklyucheniem-na-algebre-furie-dualnyh-operatsiy-neyrosetevoy-mehanizm-redutsirovaniya-kognitivnogo-dissonansa Есть такая статья.

Очень нужно ее понять. На каком уровне математики надо быть дабы понять эту статью?
Аноним 30/03/17 Чтв 17:23:48 #734 №13922 
>>13921
Уровне /b
Аноним 30/03/17 Чтв 17:51:35 #735 №13923 
>>13921
2-3.
Аноним 31/03/17 Птн 08:00:01 #736 №13934 
Котаны, напомните, как называются поверхности, точки которых равноудалены друг от друга. Аля эквидистанта для плоской кривой концентрические окружности, а тут нужно вспомнить термин для поверхностей допустим для сфер с общим центром
Аноним 31/03/17 Птн 08:50:14 #737 №13938 
>>13934
Тоже эквидистанта, нет?
Аноним 31/03/17 Птн 10:50:40 #738 №13941 
>>12775 (OP)
Матаны, посоветуйте годный курс по обыкновенным диффурам, только не Хартман и Арнольд. Можно на английском.
Аноним 31/03/17 Птн 11:54:24 #739 №13942 
>>13941
Фарлоу
Аноним 31/03/17 Птн 14:01:10 #740 №13944 
>>13942
Там по урматам, мне обыкновенные нужны.
Аноним 31/03/17 Птн 15:10:28 #741 №13945 
>>13941
Понтрягин
Аноним 31/03/17 Птн 16:58:50 #742 №13947 
>>13941
Дифференциальные уравнения и вариационное исчисление. Автор: Эльсгольц Л.Э
Аноним 31/03/17 Птн 17:22:48 #743 №13949 
Что такое математика?
Аноним 31/03/17 Птн 17:26:14 #744 №13950 
>>13949
Математика - любая деятельность, которую считают математикой математики.
Аноним 31/03/17 Птн 17:32:53 #745 №13951 
>>13949
Изучение модулей над кольцами.
Аноним 31/03/17 Птн 17:48:53 #746 №13952 
>>13951
А вот и ПУЧКист подъехал.
Аноним 31/03/17 Птн 18:02:48 #747 №13954 
>>13952
Блядь, из-за вас мне теперь стыдно обсуждать пучки. Ироды.
Аноним 31/03/17 Птн 19:00:17 #748 №13955 
>>13954
ПУЧК ПУЧК ПУЧК
Аноним 31/03/17 Птн 20:21:26 #749 №13956 
>>13955
Типа того.
Аноним 31/03/17 Птн 20:26:23 #750 №13957 
>>13952
Сейчас морфизмик появится, давай на функторах. Пучкнул тебя!
Аноним 31/03/17 Птн 20:36:33 #751 №13958 
>>13949
Наука, занимающаяся поиском инвариантов.
Аноним 31/03/17 Птн 21:10:14 #752 №13960 
Что такое геометрия?
Аноним 31/03/17 Птн 21:21:47 #753 №13962 
>>13960
Планиметрия. Остальное - мутное гавно шизофреников.
Аноним 31/03/17 Птн 21:23:10 #754 №13963 
euclid.jpg
>>13960
Наука о землемерии.
Аноним 31/03/17 Птн 21:32:38 #755 №13964 
>>13960
Познание вечного бытия.
Аноним 31/03/17 Птн 21:34:33 #756 №13965 
Что такое алгебра?
Аноним 31/03/17 Птн 21:34:36 #757 №13966 
>>13960
Числа правят миром через свойства геометрических фигур.
Аноним 31/03/17 Птн 21:39:47 #758 №13967 
>>13965
Наука о восполнении и противопоставлении.
Аноним 31/03/17 Птн 22:23:16 #759 №13968 
Что такое любовь?
Аноним 31/03/17 Птн 22:23:38 #760 №13969 
>>13968
Детка не рань меня, больше не рань.
Аноним 31/03/17 Птн 22:32:11 #761 №13970 
>>13965
Ну это когда ты типа кольца изучаешь, группы там абелевы, категории, а потом заходишь в тред и громко кричишь ПУЧК ПУЧК ПУЧК.
Аноним 31/03/17 Птн 22:39:49 #762 №13971 
>>13949
>http://ilib.mccme.ru/pdf/kurant.pdf
Кстати, вот это стоящее чтиво для вкатывания, как по вашему?
Аноним 31/03/17 Птн 23:25:29 #763 №13973 
>>13782
>Чайковского
Закончил вуз на ебучую службу, работал пол-года что ли.
Дропнул все нахуй и ушел лбом в музыку с нуля, тренировался играть на пианино с помощью нарисованными от руки клавишами.

Великий русский композитор если что.
Аноним 31/03/17 Птн 23:36:25 #764 №13974 
Посните за "конструктивизм" и почему он так забавно выставлен в отрицательном свете в ссылке по шапке? Точнее в чем заключается зашквар?
Аноним 31/03/17 Птн 23:39:28 #765 №13975 
Черт знает, к вам ли в раздел, но есть литература по МКЭ for dummies для даунов, из разряда, решаем статическую задачу на растяжение, вот плоское сечение, бьём/делаем сетку таким-то и таким-то образом с таким-то шагом, получаем узлы такие-то и такие-то, тут берём граничные условия и решаем в рукопашную системы линейных уравнений.
Аноним 31/03/17 Птн 23:46:33 #766 №13976 
>>13974
Ну это такое направление в основаниях математики, что некий объект существует, только тогда когда есть алгоритм, его строющий. Конструктивист обвиняет всех в вере, поскольку мы не пользуемся такими алгоритмами.
Аноним 31/03/17 Птн 23:47:05 #767 №13977 
>>13974
Зашквар в форсе и обзывания всех верунами, причем, он сам верит в некоторые аксиомы.
Аноним 31/03/17 Птн 23:49:22 #768 №13978 
>>13976
А что такое модули и пучки в двух словах, в чём суть этого эпичного конфликта между старообрядцами и модернистами?
Аноним 31/03/17 Птн 23:49:36 #769 №13979 
>>13976
При этом существование алгоритма никак не определяется, лол.
Аноним 01/04/17 Суб 00:06:13 #770 №13980 
>>13978
Модуль эта такая хуйня, типа векторного пространства. А пучки невъёбно абстрактная херня для алгебраических геометров.
Суть конфликта в том, что местный форсер-дед начал продвигать идею о том, что математика это наука о модулях над кольцами. Очень жестко он это форсил, в результате появились Пыни-Пучкисты, которые не знали определение модуля, но кричали про их важность. Один из таких дебилов пытался форум создать, но у него ничего не вышло. Далее, мудульнику было определено место у параши и его адептов стали называть пучкистами. Вот и всё. Финита ля ПУЧК.
Аноним 01/04/17 Суб 00:07:17 #771 №13981 
>>13978
Вот мемовые пикчи с пучканами.
>>9958
>>9968
>>9983
>>9986
>>10000
>>10744
>>11356
>>11710
>>11824
Аноним 01/04/17 Суб 00:09:36 #772 №13982 
>>13980
>>13981
Лол, понятно. Спасибо.
Аноним 01/04/17 Суб 00:22:21 #773 №13983 
Capture.PNG
Аноны, поясните, начал читать Гашкова Современная элементарная алгебра, а там обозначения какие-то, я не понимаю их. Как прочитать пик?
Аноним 01/04/17 Суб 00:22:44 #774 №13984 
>>13978
Модуль - это обобщение векторного пространства, когда вместо поля используется произвольное кольцо. У модулей появляются необычные свойства - например, не у всякого модуля есть базис.

Топологическим пространством называется множество, в котором выделено семейство подмножеств, замкнутое относительно произвольных объединений и конечных пересечений. Эти подмножества называются открытыми. Топологическое пространство формализует, что такое "быть близко расположенными друг к другу".

Категорией называется совокупность однотипных объектов такая, что для любых двух объектов указано множество морфизмов между ними. Обычно объекты - множества с какой-то математической структурой, морфизмы - сохраняющие эту структуру функции. Например, в категории групп объекты - группы, морфизмы - гомоморфизмы групп. В категории множеств объекты - множества, морфизмы - отображения. Для морфизмов введена ассоциативная операция композиции по аналогии с композицией функций. Причем для каждого объекта есть тождественный морфизм, нейтральный относительно композиции.

Пусть T - топологическое пространство, C - категория групп/колец/небес/Аллахов. Каждому открытому множеству U сопоставляем объект FU категории C. Требуем, чтобы для любого открытого подмножества V данного открытого подмножества U был бы указан морфизм склейки rVU = FU→FV. Причем rUU - тождественный морфизм U, и, кроме того, для любой тройки W⊂V⊂U выполняется равенство rWU = rWV rVU. Это называется предпучок.

Формальнее, рассмотрим категорию T, объектами которой являются открытые подмножества T, а морфизмами - обыкновенные отображения вложения. Предпучок - это контравариантный функтор из T в C.

Пучок - это способ ввести топологию на произвольной категории математических структур. Пусть есть предпучок категории T в категорию C. Мы говорим, что он является пучком, если для любого открытого покрытия произвольного открытого множества U предел функтора F над подкатегорией T, объектами которой являются элементы покрытия, изоморфен FU.

Если хорошо поломать мозг, то станет очевидно, что любую область человеческой деятельности можно рассматривать как теорию когомологий специально подобранных пучков. Но это юмор, а не серьёзный факт.
Аноним 01/04/17 Суб 00:23:58 #775 №13985 
>>13984
Спасибо, хоть и сложно, но сейчас заварю чай и постараюсь понять.
Аноним 01/04/17 Суб 00:24:54 #776 №13986 
>>13983
p|a означает, что p - делитель a.
Аноним 01/04/17 Суб 00:26:22 #777 №13987 
>>13985
Тут главное с категорными понятиями разобраться. Особенно что такое предел функтора вдоль категории.
Аноним 01/04/17 Суб 00:31:15 #778 №13988 
>>13984
>являются элементы покрытия
И их конечные пересечения.
Аноним 01/04/17 Суб 01:43:04 #779 №13989 
>>13986
Спасибо, я думал что-то серьезное означает. Заодно ещё спрошу, так как нихуя не понял. Сейчас прочел про алгоритм Евклида, ну базовую вещь, что НОД находится с помощью a / b, b / r1 -> r1 / r2 и т.д. Дальше про коэффициенты Безу (расширенный алгоритм Евклида). Вот зачем они нужны? Просто ноль информации для понимания, а в заданиях меня просят порассуждать на эту тему.
Аноним 01/04/17 Суб 01:45:18 #780 №13990 
>>13989
Мошу ошибаться, но погугли про цепные дроби, вроде бы расширенный это оно и есть.
Аноним 01/04/17 Суб 11:30:34 #781 №13998 
>>13921
Что такое "Теория Всего"?
01/04/17 Суб 11:41:03 #782 №13999 
>>13998
В ней изучаются ящики и возможность их постройки.
Аноним 01/04/17 Суб 12:09:40 #783 №14001 
>>13999
что? ссылки где?
Аноним 01/04/17 Суб 17:22:29 #784 №14037 
Вот силлогизмы Аристотеля я понял очень легко. А вот в случае с логикой высказываний, а особенно с modus ponens мне намного сложнее. Почему так? Как исправить сие недоразумение?
Аноним 01/04/17 Суб 17:36:57 #785 №14039 
>>14037
У тебя есть какие-то ожидания, которые мешают тебе читать то, что написано. Вместо этого ты пытаешься прочитать то, что ждёшь увидеть; у тебя не получается.
Аноним 01/04/17 Суб 18:18:02 #786 №14040 
>>14039
как почистить сознание тогда?
Аноним 01/04/17 Суб 18:46:23 #787 №14043 
>>14040
No idea.
Аноним 01/04/17 Суб 19:29:26 #788 №14046 
Посоны, нужно нарисовать пару графов. В какой программе можно это сделать?
Нагуглил Gephi, но она отказывается запускаться с ошибкой "Can´t find the name of Intel ICD openGL driver".
Аноним 01/04/17 Суб 19:31:07 #789 №14047 
>>14037
Примеров больше читай, пока один не попадет в цель.
Аноним 01/04/17 Суб 20:22:51 #790 №14051 
https://www.youtube.com/watch?v=Ix7qTRQvU6o
Лол, Михайлов драматург?
Аноним 01/04/17 Суб 21:20:27 #791 №14053 
Вы помните все нужные вам определение наизусть (ну, может не совсем но можете спокойно дать близкое определение, свое) или позволяете себе чувствовать интуитивно?
Аноним 01/04/17 Суб 21:31:01 #792 №14054 
>>14053
А зачем их помнить наизусть? Их суть легко запоминается, если их просто использовать.
Аноним 01/04/17 Суб 21:34:40 #793 №14055 
>>14053
Нет, это в корне неверный подход, определения, а также прочий символизм+формализм нужны лишь для того, чтобы как Диофант всё словами в несколько страниц не записывать. Мало того, даже доказательства никто не запоминает, однажды убедившись в истинности утверждения ты легко сможешь сам вывести доказательство.
Аноним 01/04/17 Суб 21:38:41 #794 №14057 
>>14055
>однажды убедившись в истинности утверждения ты легко сможешь сам
Лол, ну-ну. Отчасти это так, конечно, но в основном для простых теорем.
Аноним 01/04/17 Суб 22:01:03 #795 №14058 
>>14051
Да хуй знает, он очень странный чувак, мне временами кажется, что он на самом деле вообще никакой не математик, придумал себе несколько легенд и живёт несколько разных жизней, и зовут его совсем не так, короче типа дона Хуана.
https://www.youtube.com/watch?v=1FdY01KDfc0
https://www.youtube.com/watch?v=t28DwaZv4eQ
https://www.youtube.com/watch?v=_818e4Sbn7M
https://www.youtube.com/watch?v=jNeRYQSD6mE
Аноним 01/04/17 Суб 23:04:11 #796 №14061 
Правда ли, что каждый шестой матлогик сходит с ума?
Аноним 01/04/17 Суб 23:08:06 #797 №14062 
>>14061
Правда. Но выглядеть более странным сошедший с ума матлогик не начинает.
Аноним 01/04/17 Суб 23:16:26 #798 №14063 
>>14062
>Но выглядеть более странным сошедший с ума матлогик не начинает.
Теорема Эскобара во всей красе.
Аноним 01/04/17 Суб 23:26:15 #799 №14064 
>>14063
Аксиома.
Аноним 02/04/17 Вск 04:24:11 #800 №14066 
Посоветуйте матлогику для чайников.
Аноним 02/04/17 Вск 06:11:46 #801 №14068 
Hitler logics.webm
>>14066
Манин, Доказуемое и недоказуемое. Клини. Мендельсон. Шенфилд. Для говноедов - Колмогоров, Драгалин.
Аноним 02/04/17 Вск 12:25:51 #802 №14069 
Screenshot2017-04-02-12-24-48.png
Народ, вы же математики, а значит и в системах компьютерной алгебры шарите? Гоняю я тут максиму под андроид, а она не интегрирует. Объясните дебилу, в чем ошибка? Мануал читал, скрин прилагаю
Аноним 02/04/17 Вск 12:44:19 #803 №14070 
>>14069
Где пучки? Не вижу пучков. Ничем ни я, ни я не можем помочь. Чао!
Аноним 02/04/17 Вск 12:47:27 #804 №14071 
>>14070
> ни я, ни я
Окей, ты не сможешь помочь. Надеюсь, что на этой доске не только ты да я да мы с тобой
Аноним 02/04/17 Вск 12:53:33 #805 №14072 
>>14070
ПУЧК ПУЧК ПУЧК
Вот тебе пучки, анон.
Аноним 02/04/17 Вск 12:58:53 #806 №14073 
>>14069
Пиздец, блядь. Простите дауна, анончики, я понял. Все потому что не 3х, а 3*х, блин
Аноним 02/04/17 Вск 13:03:14 #807 №14074 
>>14073
Следующий вопрос, ребята: Максима может выводить пошаговые решения? Интересует такая возможность как под ондроедом, так и под вендой
Аноним 02/04/17 Вск 14:55:25 #808 №14075 
>>12775 (OP)
Аноны, помогите с задачей за 4 класс.
3 Бублика=6 батонам. Дополните задачу чтобы решить её.
Я придумал такое условие: всего за покупку заплатили 60 рублей.
А т.к батоны в 2 раза дешевле в рублях чем бублики, то выходит 40 и 20 заплатили за все по отдельность.
Теперь вопрос что будет если поделить 60/40? Мы найдем кол-во всех купленных бубликов?
Аноним 02/04/17 Вск 15:10:07 #809 №14076 
>>14075
Подпиши к числам единицы измерения, как в физике например, и узнаешь что у тебя получится.
Аноним 02/04/17 Вск 16:01:46 #810 №14077 
>>14075
>всего за покупку заплатили 60 рублей.
И чо? Может, они один бублик купили.
Аноним 02/04/17 Вск 16:14:55 #811 №14078 
>>14074
Нет, не может.
Аноним 02/04/17 Вск 17:10:02 #812 №14080 
>>14078
Однако здесь symbolab.com предусмотрено пошаговое решение.
Аноним 02/04/17 Вск 17:32:24 #813 №14081 
РЕБЯТКИ! а поле p-адических чисел алгебраически замкнуто или не???
Число е (Эйлера) Аноним 02/04/17 Вск 18:42:49 #814 №14082 
Сап двач ^^
Есть среди вас действительно мощные математики?
Реквестирую формулу для нахождения N-ой цифры числа e.
Для числа Pi изи найти https://habrahabr.ru/post/179829/ , а вот для е почему-то нету :c
Выручайте ананасы!
Аноним 02/04/17 Вск 20:38:32 #815 №14083 
>>12775 (OP)
Я понял почему местные таполаги не смогли ящик решить. Они читают книжки и на задачках из книжек только согласительно кивают и не решают их.
Сам такой был пока не решил решить всего демидовича и на первой же задачке обосрался.
Аноним 02/04/17 Вск 21:04:03 #816 №14084 
>>14083
Что тебе мешает решать задачи по тапалогиям и гамалогиям, игнорируя при этом прикладные задачи?
Аноним 02/04/17 Вск 21:19:00 #817 №14085 
24.png
Суп, матемач. Объясни дауну, как получается пикрел. Я неделю не могу понять это. Как найти область изменения ро?
Аноним 02/04/17 Вск 21:45:38 #818 №14086 
>>14083
Ну они же типа теорию учать, хотя сами едва её понимают, а когда доходит дела до выёбонов на дваче, то просто переписывают определения из книги.

>>14084
Пынька, ты путаешь прикладные и математические задачи. Демидович - книга для математика а не для прикладника.
Аноним 02/04/17 Вск 21:47:31 #819 №14087 
>>14086
> Демидович - книга для математика а не для прикладника.
> Демидович - книга для математика
Мань, плиз...
02/04/17 Вск 22:11:15 #820 №14088 
>>14087
А вот и безаргументный даун с манямирком. Для кого же она?
Аноним 02/04/17 Вск 22:19:09 #821 №14089 
>>14088
Для дедов, которые в универах ебут студентов неактуальным материалом.
Аноним 02/04/17 Вск 22:20:03 #822 №14090 
>>14089
>>14088
Алсо, в 50е годы, когда книга была написана, она была актуальна, потому что тогда компов еще не было.
02/04/17 Вск 22:21:41 #823 №14091 
>>14090
>>14089
А теперь скажи что конкретно в ней устарело, даун.
Аноним 02/04/17 Вск 22:32:48 #824 №14092 
>>14091
Здравствуйте робяты. Дед на связи.
А о какой книге Демидовича вы говорите? У него их очень много:

Сборник задач и упражнений по математическому анализу
Задачи и упражнения по математическому анализу для ВТУЗов
Краткий курс высшей математики
Основы вычислительной математики
Дифференциальные уравнения
Лекции по математической теории устойчивости
Математические основы квантовой механики
Численные методы анализа
* Элементы теории множеств в курсе математического анализа
02/04/17 Вск 22:36:07 #825 №14093 
>>14092
>Сборник задач и упражнений по математическому анализу
Я об этом.
Аноним 02/04/17 Вск 22:36:27 #826 №14094 
>>14085
Берешь полярные координаты дроченые и вставляешь их в область точенную.
Аноним 02/04/17 Вск 22:38:09 #827 №14095 
>>14092
Так так, а картофанчик принёс?
Аноним 02/04/17 Вск 22:38:59 #828 №14096 
>>14095
А вот и мемасы подъехали.
Аноним 02/04/17 Вск 22:41:55 #829 №14097 
>>14096
Но если картофанчика не принёс, о каких мемесах может идти речь?
Аноним 02/04/17 Вск 22:42:44 #830 №14098 
>>14097
Ты же говоришь о мемасах, пучнутый.
Аноним 02/04/17 Вск 23:17:46 #831 №14099 
Щас тут выясним что демидович не мотиматика нихуя.
ТАк же как и математики не считаю а устанавливают изоморфизмы.
Вконец уже ебанулись.
Аноним 02/04/17 Вск 23:20:46 #832 №14100 
>>14095
Да, и водовку.

>>14096
А ты не встревай, когда деда спрашивают.
Аноним 02/04/17 Вск 23:21:33 #833 №14101 
Selection025.png
>>14091
Чисто наугад открыл
http://www.wolframalpha.com/input/?i=100-th+derivative+of+x+sinh+x
Аноним 02/04/17 Вск 23:24:28 #834 №14103 
Selection026.png
>>14101
http://www.wolframalpha.com/input/?i=integral+(xe%5Ex)%2F(x%2B1)%5E2
Аноним 02/04/17 Вск 23:42:58 #835 №14104 
>>14103
>>14101
Типа вольфрам по другому посчитает или чё?
мимодаунизб
Аноним 02/04/17 Вск 23:57:20 #836 №14105 
>>14101
>>14103
А теперь полистай дальше, там есть и полезные задачи на доказательства.
Аноним 03/04/17 Пнд 00:08:32 #837 №14106 
>>14077
Ну, очевидно что заплатили всего 60 рублей за обе покупки.
>>14076
Ни понимат. Звучит как бред, я просто хочу в мат логику вникнуть.
Если общую сумму покупки поделить на общую сумму покупки для одного предмета этой суммы то что выйдет?
Аноним 03/04/17 Пнд 00:10:24 #838 №14107 
>>14106
> Звучит как бред
>вникнуть в матлогику
С таким мышлением не выйдет.
Аноним 03/04/17 Пнд 00:15:23 #839 №14108 
>>14107
У тебя если только.
Аноним 03/04/17 Пнд 00:25:49 #840 №14110 
>>14105
Возможно, но зачем нужна книга, где полезных задач процентов 10? Почему бы не читать книгу, где все задачи нормальные?
Аноним 03/04/17 Пнд 00:27:11 #841 №14111 
>>14110
Типа вычисление интегралов это ненормально? Лол, твои слова да моей бы преподше в уши, я всё через вольфрам всегда решал, кек. Правда так нихуя и не помню по матеше, но пох.
Аноним 03/04/17 Пнд 00:32:40 #842 №14112 
>>14110
Что значит полезных?
Аноним 03/04/17 Пнд 00:54:53 #843 №14113 
>>14100
>Да, и водовку.
Вообще от души, накрывай.
Аноним 03/04/17 Пнд 01:14:54 #844 №14114 
>>14112
Значит полезных для меня.
Аноним 03/04/17 Пнд 01:20:02 #845 №14115 
>>14114
Охуеть просто, он значит решает, что будет полезно другим только по себе.
Вот ты и обосрался. У тебя тупые придирки, но по факту ты не можешь сказать, почему Демидович книга не для математиков.
Аноним 03/04/17 Пнд 01:33:00 #846 №14116 
>>14115
Сложно, конечно, понять, что полезность - это крайне субъективное понятие, но ты постарайся.

Если ты считаешь взятие интегралов полезным хоть для кого-то, например, для себя, то приведи пример ситуации, в которой тебе нельзя было пользоваться математическим по и знание кучи дедовских трюков по вычислению неопределенных интегралов тебе помогло.
03/04/17 Пнд 01:57:03 #847 №14117 
>>14116
Тогда зачем ты вообще упоминаешь субъективщину, когда мы тут объективно её оцениваем?

Спор был о том, для математиков это книга или нет, не пытайся сворачивать с темы. И как я вижу, у тебя нет не одного огрумента в свою пользу, кроме визжания о дидах и трюках.
Видимо ты из тех поехавших, для которых вычисления это не математика а инжинерное дело.
Аноним 03/04/17 Пнд 02:24:09 #848 №14118 
>>14117
> Тогда зачем ты вообще упоминаешь субъективщину, когда мы тут объективно её оцениваем?
Ясно.

> И как я вижу, у тебя нет не одного огрумента в свою пользу, кроме визжания о дидах и трюках.
Вполне конкретный вопрос тебе задал. Пример ты привести не смог.

> Видимо ты из тех поехавших, для которых вычисления это не математика
Ааа, то есть ты с этим не согласен? Ну ясно, я-то думал, что ты не полный даун, а оказался полный.
03/04/17 Пнд 02:36:13 #849 №14119 
>>14118
>Ясно.
Мне вот категории не нужны, значит они бесполезны для математика? Всё, категории теперь - computer science а не часть алгебры.

>Вполне конкретный вопрос тебе задал. Пример ты привести не смог.
Ты не смог привести то, что устарело в Демидовиче, поскольку интегралы не устрели, манька.

>Ааа, то есть ты с этим не согласен? Ну ясно, я-то думал, что ты не полный даун, а оказался полный.
ПУЧК ПУЧК ПУЧК ТОЛЬКО МУДУЛИ НАД КОЛЬЦАМИ ИНТЕГРАЛЫ ДЛЯ ДИДОВ КОЛЬЦА КОГОМОЛОГИИ СИМПЛЕКСОВ ТОПОЛОГИЧЕСКИЕ ПРОСТРАНСТВА ПУЧК
ДАУН СЧИТАЕТ ЧТО ВЫЧИСЛЕНИЕ НЕ ИНЖИНЕРНОЕ ДЕЛО НЕ НУ ВЫ ВИДЕЛИ ПУЧК? ЗА ТАКОЕ НА ТИФАРЕТИ УБИВАЮТ
Вообще чего ожидать от дегенерата, который считает, что умение считать не часть математики? Типичный мошеник, лжеч и урод, который только и делает, что вводит других людей в заблуждение. Лживая манька, пытается объявить всё что ей не нравится и что она не понимает не математикой.

Аноним 03/04/17 Пнд 12:48:54 #850 №14120 
изображение.png
>>14119
>Мне вот категории не нужны, значит они бесполезны для математика? Всё, категории теперь - computer science а не часть алгебры.
Самое простое: категории нужны для топологии, топология нужна для много чего.

>>14083
>Сам такой был пока не решил решить всего демидовича и на первой же задачке обосрался.
Ты серьезно? Ты тот анон, который чуть ранее плакался, что не понимает мат. индукцию?
Аноним 03/04/17 Пнд 12:55:34 #851 №14121 
>>14120
а и да
>Сам такой был
По себе людей не судят.
Аноним 03/04/17 Пнд 14:02:02 #852 №14122 
>>14120
>Самое простое: категории нужны для топологии, топология нужна для много чего.
НУ ДЛЯ МИНЯ ЖЕ ОНИ БЕСПОСЛЕЗНЫ ЗНАЧИТ НЕ МАТЕМАТИКА РЯЯЯЯ КАТЕГОРИИ КОМПЬЮТЕР САЕНС А НЕ МАТЕМАТИКА
Аноним 03/04/17 Пнд 17:09:43 #853 №14124 
>>14081
БАМП епта вопросу, че как дауны не можете ответить мне
Аноним 03/04/17 Пнд 20:42:04 #854 №14127 
cgsFtrocFCk.jpg
Аноны, нужна помощь с 3 заданием. Как построить метрический тензор? В интернете информации мало, в методичке тоже непонятно. Пробовал задать его скалярным произведением, но препод сказал что это тавтология и метрический тензор сам определяет скалярное произведение. Еще проверка на существование дискриминантного, с этим тоже есть проблемы. Если есть ответ на хотя бы один из этих вопросов, напишите пожалуйста.
Аноним 03/04/17 Пнд 22:00:09 #855 №14130 
>>14127
Метрический тензор иногде представляют как оператор из пространства в его сопряжённое. Видимо, это и имеется в виду.
С дискриминантным - хуй знает. Я не представляю, как он может не существовать, и при чём тут "заданный базис", если тензор - это по определению ковариантная вещь.
Аноним 03/04/17 Пнд 22:56:50 #856 №14131 
>>14127
g_i,j = e_i * e_j
Аноним 03/04/17 Пнд 23:37:33 #857 №14132 
>>14124
Нет.
Аноним 04/04/17 Втр 00:42:51 #858 №14133 DELETED
Срочно нужно. Дайте формулу для пикрил функции.
Аноним 04/04/17 Втр 00:47:25 #859 №14134 
>>14133
sqrt(abs(x))
Аноним 04/04/17 Втр 00:49:07 #860 №14135 DELETED
>>14134
Еще есть варианты? Этот не подходит.
Аноним 04/04/17 Втр 00:56:18 #861 №14136 
>>14135
Чем он не подходит?
Аноним 04/04/17 Втр 01:10:17 #862 №14137 DELETED
>>14136
>Чем он не подходит?
Мне он не подходит прост))) Решил через кусочно-линейную.
Аноним 04/04/17 Втр 07:50:12 #863 №14139 
>>14133
циклоиду гугли
Аноним 04/04/17 Втр 08:11:38 #864 №14141 
>>14121
>По себе людей не судят.
>теорию учат, хотя сами едва её понимают, а когда доходит дела до выёбонов на дваче, то просто переписывают определения из книги.
Аноним 04/04/17 Втр 09:04:10 #865 №14142 
Сап. Подскажите материал по дка и их минимизации.
Аноним 04/04/17 Втр 12:13:40 #866 №14146 
>>14122
Так а при чём тут "для меня"? И при чём тут компьютер саенс? Речь вроде как была о том, что матпакеты давно считают интегралы лучше людей, поэтому с научной точки зрения это вопрос построения эффективного алгоритма, а не каких-то устаревших кустарных методов. Что такое в категориях компьютер умеет делать лучше человека? Он вообще в них хоть что-то может? Взятие интегралов сейчас - это примерно как строевая подготовка в современной армии. Вещь заведомо рудиментарная, остающаяся в программе в основном потому что "диды считали и мы будем", плюс если их убирать, то нужно их чем-то заменять, и переобучать легион 80-летних дедов, которые в своей жизни ничего кроме интегралов и водовки с картофаном не видели.

Я олимпиадник, так что интегралы считать умею, тут без этого далеко не уедешь. Но олимпиады - это всё-таки спорт, а не наука. Учёному важно уметь не СЧИТАТЬ интегралы, а исследовать их. Куча разных спецфункций определяются или могут быть выражены в интегральной форме, и из неё нужно как-то извлечь их свойства. Но на то это и спецфункции, что такие интегралы ПОСЧИТАТЬ не получится, хоть усрись.
Аноним 04/04/17 Втр 12:32:36 #867 №14147 
Ну почему /math/ такой мёртвый? Хоть нихуя не смыслю в математике, но уже месяца два тут ридонли сижу, единственная вменяемая доска. Не хочу опять в б.
Аноним 04/04/17 Втр 12:33:48 #868 №14148 
>>14146
Пиздишь ты и про олимпиады, и про то, что можешь решить. Унылое говно.
Аноним 04/04/17 Втр 12:39:17 #869 №14149 
14909053874080.jpg
>>14148
Ну кто бы мог подумать.
Я бы, конечно, мог сдеанониться и предоставить пруфы, но не вижу смысла. Заблуждайся дальше, если хочешь.
Аноним 04/04/17 Втр 12:39:30 #870 №14150 
>>14147
Сколько листочков ты сегодня решил?
Ни одного? А вот решал бы - было бы о чем поговорить.
Аноним 04/04/17 Втр 12:40:20 #871 №14151 
>>14149
А я мимоанон, просто демагога за версту чую.
Аноним 04/04/17 Втр 12:43:10 #872 №14152 
>>14149
Александр Шень
В редакцию журнала "Математическое просвещение"

Олимпиадный культ: основные принципы

1. Каждому школьнику ставится в соответствие некоторый элемент
линейно упорядоченного множества, называемый его олимпиадностью.

2. Каждой задаче ставится в соответствие некоторый элемент
линейно упорядоченного множества, называемый её олимпиадностью.

3. Значения О. школьников и задач не даны нам непосредственно в
ощущениях. О них можно судить по косвенным признакам:

а. О. школьника связана с умением решать задачи высокой О.

б. О. задачи связана с тем, насколько её решение коррелирует
с высокой О. школьника.

(Взаимная рекурсивность а и б приводит к интересным задачам в
области линейной алгебры)

4. Для применения критериев п.3 проводятся массовые ритуальные
жертвоприношения (олимпиады); О. школьника связана с числом О.,
в которых он участвовал, и с характером повреждений ("диплом",
"похвальная грамота" и др.)

5. Пройдя достаточное число ритуалов, участник О. посвящается в
сан Организатора О., имеющего дар судить об О. задач. Наиболее
просветлённые О. О. могут судить также об О. нерешённых ими
задач и незнакомых им школьников.

6. Признаком высокой О. школьника и задачи является её появление
на Вышестоящей О., независимо от обстоятельств такого появления.

Настоящие принципы открыты группой этнографов, в научных целях
принимавших участие в отправлении О. культа в течение ряда лет.
Ими установлено также, что

- нет статистически достоверных результатов о вреде О. культа
для участников О., хотя поклонники О. вовлекают в ритуалы
тысячи школьников; данные о вреде культа для О.О. более
достоверны, но также требуют уточнения;

- хотя поклонники О. и презирают не участвующих в нём, а также
отступников и лиц с малой О., они их не преследуют.

Тем самым нет оснований считать культ О. опасным и
препятствовать его отправлению; напротив, следует отметить
разнообразие и загадочность ритуалов, благодаря которым он
представляет несомненный интерес для этнографии.

Статья представлена 01.04.2002
Аноним 04/04/17 Втр 12:49:02 #873 №14153 
>>14151
Лучше бы к себе принюхался, друже.
>>14152
Ну и кого ты хочешь удивить этим баяном? С Шенем я, кстати, общался лично, это всё не более чем шутка. Что и так довольно очевидно
Аноним 04/04/17 Втр 12:52:01 #874 №14154 DELETED
>>14152
Ты тратишь время, это кочующий битард, он ничего не смыслит в том, что он говорит, да это и не важно, важно как он говорит, форма, которая его и палит. Вангую что у него аналогичные треды открыты в других тематиках и он одними и теми же общими словами пользуется, при том сам цепляется к синонимам, например, пытаясь на этом выезжать и так далее. Зачем ты его кормишь? Его знания основаны на википедии, причём гуглит он это прямо сейчас, чтобы тебе ответить.
>Я бы, конечно, мог сдеанониться и предоставить пруфы, но не вижу смысла.
>олимпиады это спорт
>врёти
Или ты это понимаешь и всё равно продолжаешь? Или не понимаешь?
Аноним 04/04/17 Втр 12:57:32 #875 №14155 DELETED
>>14154
>гринтекст, агрессивные наезды и ни слова по существу
>обвиняет других в демагогии
Забавный ты зверёк.
Аноним 04/04/17 Втр 12:57:59 #876 №14156 
>>14152
Если попросишь его вычислить определённый интеграл, который Вольфрам не посчитает, он скажет, что это можно сделать на любом языке программирования, но он не какое-то быдлопрограммист, а тополог, если попросишь неопределённый скажет, что в неопределённых смысла нет, так как на практике всё считается только с конкретными значениями, а кроме практики нигде этот картофан под водовку не используется, в отличие от метафизическитеоретической топологии, пучков, категорий и что там ещё в википедии близко расположено. Это ж бред, зачем ты тратишь своё время?
Аноним 04/04/17 Втр 12:59:02 #877 №14157 DELETED
>>14155
Ещё бы!
Аноним 04/04/17 Втр 13:06:58 #878 №14158 DELETED
И ещё одно: причина, по которой быдло вроде меня и этого унтерка наводнила этот раздел только одна. Попросите Абу убрать с видного места ссылку на доску и проблема решится сама собой.
Аноним 04/04/17 Втр 13:14:43 #879 №14159 DELETED
>>14153
Эй, ты тут?
Аноним 04/04/17 Втр 13:17:14 #880 №14160 DELETED
>>14159
Здесь тебе не чатик, чтобы своих друзьяшек окликать. Хочешь написать что-то содержательно - берёшь и пишешь.
Аноним 04/04/17 Втр 13:20:04 #881 №14161 DELETED
>>14160
Вот скажи, тебе что, /b/ с тематиками мало? Зачем срёшь тут? Тут же годно, да и сам стопроцентов много нового здесь узнал, вряд ли бы ты тут надолго остался, если бы не затянуло. Зачем срать там, где питаешься? Скоро вообще негде отдохнуть от бесконечных срачей будет.
Аноним 04/04/17 Втр 13:32:25 #882 №14162 
>>14153
Зачем удивлять? Просто недолюбливаю олимпиады.
Аноним 04/04/17 Втр 13:43:00 #883 №14163 
>>14162
Ездишь по стране (а иногда и за её пределами) за казённый счёт, решаешь интересные задачи, выигрываешь призы. В универе тебя за это любят, физру и всякие психологии с правоведениями закрывают автоматом, стипендию повышенную платят. Годно же.
Аноним 04/04/17 Втр 13:53:53 #884 №14164 
>>14163
А жить когда?
Аноним 04/04/17 Втр 13:55:50 #885 №14165 
>>14164
Оставшиеся 330 дней в году, полагаю.
Аноним 04/04/17 Втр 14:07:03 #886 №14166 
Котятки, запилите перекат, будьняшками, я с калькулятора
Аноним 04/04/17 Втр 14:10:52 #887 №14167 
>>14166
Переката раньше выходных можно не ждать.
Аноним 04/04/17 Втр 14:17:57 #888 №14168 
>>14167
Почему? Я оп, если что, просто флажок давно отвалился. Ты на выходных хочешь запилить?
Аноним 04/04/17 Втр 14:18:28 #889 №14169 DELETED
>>14168
>>14158
Аноним 04/04/17 Втр 14:19:44 #890 №14170 DELETED
>>14169
А я тут при чем?
Аноним 04/04/17 Втр 14:21:06 #891 №14171 DELETED
>>14170
Написали бы Абу и стало бы чище. Или я хуйню написал? Просто реально на самом видном месте, так бы я сюда никогда не попал.
Аноним 04/04/17 Втр 14:58:52 #892 №14176 
>>14166
https://2ch.hk/math/res/14175.html
Перекат
comments powered by Disqus

Отзывы и предложения